Feat: DM pour les tst sti2d

This commit is contained in:
Bertrand Benjamin 2021-02-06 11:33:33 +01:00
parent 3fa915e643
commit ae382c87bb
43 changed files with 4993 additions and 0 deletions

View File

@ -0,0 +1,121 @@
\documentclass[a4paper,10pt]{article}
\usepackage{myXsim}
\usepackage{tasks}
% Title Page
\title{DM1 \hfill BAHBAH Zakaria}
\tribe{TST sti2d}
\date{\hfillÀ render pour le jeudi 25 février}
\xsimsetup{
solution/print = false
}
\begin{document}
\maketitle
\begin{exercise}[subtitle={Complexes}]
\begin{enumerate}
\item Mettre le nombre complexe suivant sous forme algébrique $z_1 = \dfrac{7 + 2 i}{-5 + 3 i} $
\item Mettre le complexe suivante sous forme exponentielle $z_2 = -7 + 7 \sqrt{3} i$
\item Mettre le complexe suivante sous forme exponentielle $z_3 = 4 + 4 \sqrt{3} i$
\item Calculer le produit $z_4=z_2\times z_3$ donner le résultat sous forme exponentielle puis algébrique.
\item Calculer le quotient $z_5=\frac{z_2}{z_3}$ donner le résultat sous forme exponentielle puis algébrique.
\end{enumerate}
\end{exercise}
\begin{solution}
\begin{enumerate}
\item $z_1 = - \frac{29}{34} - \frac{31 i}{34}$
\item $z_3 = 14 e^{\frac{2 i \pi}{3}}$
\item $z_4 = 112 e^{i \pi} = -112 = -112.0$
\item $z_5 = \frac{7}{4} e^{\frac{i \pi}{3}} = \frac{7}{8} + \frac{7 \sqrt{3} i}{8} = 0.875 + 1.52 i$
\end{enumerate}
\end{solution}
\begin{exercise}[subtitle={Bassin}]
Le tour d'un bassin au niveau du sol présente deux axes de symétrie : laxe des abscisses et la droite déquation $x=4$. Il est obtenu par symétrie de la courbe $\mathcal{C_f}$ sur $\intFF{0}{4}$$f$ est la fonction définie par
\[
f(x) = \left(- x^{2} + 2.4 x - 6.2\right) e^{- x} + 6.2
\]
On admet que sur $\intFF{0}{4}$ la fonction $f$ est positive.
\begin{enumerate}
\item Sur un repère, tracer l'allure de la courbe $\mathcal{C}_f$, les axes de symétries puis compléter pour dessiner la forme du bassin.
\item Montrer que la fonction $f$ admet comme primitive sur $\R$ la fonction $F$ définie par
\[
F(x) = 6.2 x + \left( x^{2} - 0.4 x + 5.8\right) e^{- x}
\]
\item Calculer la quantité $\ds \int_0^4 f(x) \; dx$, vous donnerez le résultat sous forme exacte. Interpréter le résultat et reportez cette quantité sur le graphique.
\item On considère que l'échelle de votre graphique est de 1unité pour 15m. Calculer l'aire du bassin. Vous donnerez un résultat arrondi au $m^2$ près.
\end{enumerate}
\end{exercise}
\begin{solution}
\begin{enumerate}
\item
\begin{tikzpicture}[baseline=(a.north), xscale=1, yscale=0.5]
\tkzInit[xmin=0,xmax=5,xstep=1,
ymin=0,ymax=10,ystep=1]
\tkzGrid
\tkzAxeXY
\tkzFct[domain=0:10,color=red,very thick]%
{ (-x**2 + 2.4*x - 6.2)*exp(-x) + 6.2 };
\end{tikzpicture}
\item Il faut dériver $F(x)$ et vérifier que $F'(x) = f(x)$.
\item $\ds \int_0^4 f(x) \; dx = F(4) - F(0) = \frac{20.2}{e^{4}} + 19.0$
\item La quantité calculée à la question précédente se retrouve 4fois pour former le bassin. Il faut ensuite prendre en compte l'échelle, comme 1unité de longueur correspond à 15m, une unité d'air correspond à $15\times15 = 225m^2$. Ainsi l'aire du bassin est égale à
\[
(\frac{20.2}{e^{4}} + 19.0)\times 4 \times 15^2 = 17433.00000
\]
\end{enumerate}
\end{solution}
\begin{exercise}[subtitle={Bassin}]
Le clinker est un constituant du ciment qui résulte de la cuisson d'un mélange composé de calcaire et d'argile. La fabrication du clinker nécessite des fours à très haute température qui libèrent dans l'air une grande quantité de dioxyde de carbone (CO$_2$).
Dans une cimenterie, la fabrication du clinker s'effectue de 7 h 30 à 20 h, dans une pièce de volume \np{900000}~dm$^3$.
À 20 h, après une journée de travail, le taux volumique de CO$_2$ dans la pièce est de 0.9\,\%.
\begin{enumerate}
\item Justifier que le volume de CO$_2$ présent dans cette pièce à 20 h est de \np{8100}~dm$^3$ .
\item On modélise le volume de CO$_2$ présent dans la pièce par une fonction du temps $t$ écoulé après 20h (exprimé en minutes) qui pour formule $V(t) = V_0e^{-0.06t} + 560$
\begin{enumerate}
\item Démontrer que $V_0$ est égale à \np{7540}.
\item Quel sera, au dm$^3$ près, le volume de CO$_2$ dans cette pièce à 23 h ?
\item Démontrer que $V'(t) = - 452.4 e^{- 0.06 t}$.
\item Étudier le signe de $V'(t)$ puis en déduire le sens de variation de $V(t)$.
\item Que peut-on dire du volume de CO$_2$ quand $t$ devient grand?
\end{enumerate}
\end{enumerate}
\end{exercise}
\begin{solution}
\begin{enumerate}
\item Volume à 20h: $900000\times 0.009000000000000001 = 8100$
\item
\begin{enumerate}
\item $t=0$ correspond à 20h.
Donc $V(0) = 8100 = V_0e^{-0.06\times 0} + 560 = V_0 + 560$
Donc $V_0 = 8100 - 560 = 7540$
\item Il faut calculer $V(t)$ pour $t = 3$ donc
\[
V(3) = 6857.94
\]
\item Pas de correction pour cette question.
\item Pas de correction pour cette question.
\item Pas de correction pour cette question.
\end{enumerate}
\end{enumerate}
\end{solution}
\end{document}
%%% Local Variables:
%%% mode: latex
%%% TeX-master: "master"
%%% End:

View File

@ -0,0 +1,121 @@
\documentclass[a4paper,10pt]{article}
\usepackage{myXsim}
\usepackage{tasks}
% Title Page
\title{DM1 \hfill BENALI Ilyas}
\tribe{TST sti2d}
\date{\hfillÀ render pour le jeudi 25 février}
\xsimsetup{
solution/print = false
}
\begin{document}
\maketitle
\begin{exercise}[subtitle={Complexes}]
\begin{enumerate}
\item Mettre le nombre complexe suivant sous forme algébrique $z_1 = \dfrac{9 + 3 i}{-9 + 8 i} $
\item Mettre le complexe suivante sous forme exponentielle $z_2 = 2 \sqrt{2} + 2 \sqrt{2} i$
\item Mettre le complexe suivante sous forme exponentielle $z_3 = 4 \sqrt{2} - 4 \sqrt{2} i$
\item Calculer le produit $z_4=z_2\times z_3$ donner le résultat sous forme exponentielle puis algébrique.
\item Calculer le quotient $z_5=\frac{z_2}{z_3}$ donner le résultat sous forme exponentielle puis algébrique.
\end{enumerate}
\end{exercise}
\begin{solution}
\begin{enumerate}
\item $z_1 = - \frac{57}{145} - \frac{99 i}{145}$
\item $z_3 = 4 e^{\frac{i \pi}{4}}$
\item $z_4 = 32 e^{0} = 32 = 32.0$
\item $z_5 = \frac{1}{2} e^{\frac{i \pi}{2}} = \frac{i}{2} = 0.5 i$
\end{enumerate}
\end{solution}
\begin{exercise}[subtitle={Bassin}]
Le tour d'un bassin au niveau du sol présente deux axes de symétrie : laxe des abscisses et la droite déquation $x=4$. Il est obtenu par symétrie de la courbe $\mathcal{C_f}$ sur $\intFF{0}{4}$$f$ est la fonction définie par
\[
f(x) = \left(- x^{2} + 8.7 x - 5.0\right) e^{- x} + 5.0
\]
On admet que sur $\intFF{0}{4}$ la fonction $f$ est positive.
\begin{enumerate}
\item Sur un repère, tracer l'allure de la courbe $\mathcal{C}_f$, les axes de symétries puis compléter pour dessiner la forme du bassin.
\item Montrer que la fonction $f$ admet comme primitive sur $\R$ la fonction $F$ définie par
\[
F(x) = 5.0 x + \left( x^{2} - 6.7 x - 1.7\right) e^{- x}
\]
\item Calculer la quantité $\ds \int_0^4 f(x) \; dx$, vous donnerez le résultat sous forme exacte. Interpréter le résultat et reportez cette quantité sur le graphique.
\item On considère que l'échelle de votre graphique est de 1unité pour 15m. Calculer l'aire du bassin. Vous donnerez un résultat arrondi au $m^2$ près.
\end{enumerate}
\end{exercise}
\begin{solution}
\begin{enumerate}
\item
\begin{tikzpicture}[baseline=(a.north), xscale=1, yscale=0.5]
\tkzInit[xmin=0,xmax=5,xstep=1,
ymin=0,ymax=10,ystep=1]
\tkzGrid
\tkzAxeXY
\tkzFct[domain=0:10,color=red,very thick]%
{ (-x**2 + 8.7*x - 5.0)*exp(-x) + 5.0 };
\end{tikzpicture}
\item Il faut dériver $F(x)$ et vérifier que $F'(x) = f(x)$.
\item $\ds \int_0^4 f(x) \; dx = F(4) - F(0) = 21.7 - \frac{12.5}{e^{4}}$
\item La quantité calculée à la question précédente se retrouve 4fois pour former le bassin. Il faut ensuite prendre en compte l'échelle, comme 1unité de longueur correspond à 15m, une unité d'air correspond à $15\times15 = 225m^2$. Ainsi l'aire du bassin est égale à
\[
(21.7 - \frac{12.5}{e^{4}})\times 4 \times 15^2 = 19324.00000
\]
\end{enumerate}
\end{solution}
\begin{exercise}[subtitle={Bassin}]
Le clinker est un constituant du ciment qui résulte de la cuisson d'un mélange composé de calcaire et d'argile. La fabrication du clinker nécessite des fours à très haute température qui libèrent dans l'air une grande quantité de dioxyde de carbone (CO$_2$).
Dans une cimenterie, la fabrication du clinker s'effectue de 7 h 30 à 20 h, dans une pièce de volume \np{700000}~dm$^3$.
À 20 h, après une journée de travail, le taux volumique de CO$_2$ dans la pièce est de 0.8\,\%.
\begin{enumerate}
\item Justifier que le volume de CO$_2$ présent dans cette pièce à 20 h est de \np{5600}~dm$^3$ .
\item On modélise le volume de CO$_2$ présent dans la pièce par une fonction du temps $t$ écoulé après 20h (exprimé en minutes) qui pour formule $V(t) = V_0e^{-0.09t} + 360$
\begin{enumerate}
\item Démontrer que $V_0$ est égale à \np{5240}.
\item Quel sera, au dm$^3$ près, le volume de CO$_2$ dans cette pièce à 21 h ?
\item Démontrer que $V'(t) = - 471.6 e^{- 0.09 t}$.
\item Étudier le signe de $V'(t)$ puis en déduire le sens de variation de $V(t)$.
\item Que peut-on dire du volume de CO$_2$ quand $t$ devient grand?
\end{enumerate}
\end{enumerate}
\end{exercise}
\begin{solution}
\begin{enumerate}
\item Volume à 20h: $700000\times 0.008 = 5600$
\item
\begin{enumerate}
\item $t=0$ correspond à 20h.
Donc $V(0) = 5600 = V_0e^{-0.09\times 0} + 360 = V_0 + 360$
Donc $V_0 = 5600 - 360 = 5240$
\item Il faut calculer $V(t)$ pour $t = 1$ donc
\[
V(1) = 5149.00000000000
\]
\item Pas de correction pour cette question.
\item Pas de correction pour cette question.
\item Pas de correction pour cette question.
\end{enumerate}
\end{enumerate}
\end{solution}
\end{document}
%%% Local Variables:
%%% mode: latex
%%% TeX-master: "master"
%%% End:

View File

@ -0,0 +1,121 @@
\documentclass[a4paper,10pt]{article}
\usepackage{myXsim}
\usepackage{tasks}
% Title Page
\title{DM1 \hfill BERNADAT Noah}
\tribe{TST sti2d}
\date{\hfillÀ render pour le jeudi 25 février}
\xsimsetup{
solution/print = false
}
\begin{document}
\maketitle
\begin{exercise}[subtitle={Complexes}]
\begin{enumerate}
\item Mettre le nombre complexe suivant sous forme algébrique $z_1 = \dfrac{2 + 3 i}{-7 + 10 i} $
\item Mettre le complexe suivante sous forme exponentielle $z_2 = - 5 \sqrt{3} + 5 i$
\item Mettre le complexe suivante sous forme exponentielle $z_3 = 3 \sqrt{3} - 3 i$
\item Calculer le produit $z_4=z_2\times z_3$ donner le résultat sous forme exponentielle puis algébrique.
\item Calculer le quotient $z_5=\frac{z_2}{z_3}$ donner le résultat sous forme exponentielle puis algébrique.
\end{enumerate}
\end{exercise}
\begin{solution}
\begin{enumerate}
\item $z_1 = \frac{16}{149} - \frac{41 i}{149}$
\item $z_3 = 10 e^{\frac{5 i \pi}{6}}$
\item $z_4 = 60 e^{\frac{2 i \pi}{3}} = -30 + 30 \sqrt{3} i = -30.0 + 52.0 i$
\item $z_5 = \frac{5}{3} e^{i \pi} = - \frac{5}{3} = -1.67$
\end{enumerate}
\end{solution}
\begin{exercise}[subtitle={Bassin}]
Le tour d'un bassin au niveau du sol présente deux axes de symétrie : laxe des abscisses et la droite déquation $x=4$. Il est obtenu par symétrie de la courbe $\mathcal{C_f}$ sur $\intFF{0}{4}$$f$ est la fonction définie par
\[
f(x) = \left(- x^{2} + 4.9 x - 9.7\right) e^{- x} + 9.7
\]
On admet que sur $\intFF{0}{4}$ la fonction $f$ est positive.
\begin{enumerate}
\item Sur un repère, tracer l'allure de la courbe $\mathcal{C}_f$, les axes de symétries puis compléter pour dessiner la forme du bassin.
\item Montrer que la fonction $f$ admet comme primitive sur $\R$ la fonction $F$ définie par
\[
F(x) = 9.7 x + \left( x^{2} - 2.9 x + 6.8\right) e^{- x}
\]
\item Calculer la quantité $\ds \int_0^4 f(x) \; dx$, vous donnerez le résultat sous forme exacte. Interpréter le résultat et reportez cette quantité sur le graphique.
\item On considère que l'échelle de votre graphique est de 1unité pour 15m. Calculer l'aire du bassin. Vous donnerez un résultat arrondi au $m^2$ près.
\end{enumerate}
\end{exercise}
\begin{solution}
\begin{enumerate}
\item
\begin{tikzpicture}[baseline=(a.north), xscale=1, yscale=0.5]
\tkzInit[xmin=0,xmax=5,xstep=1,
ymin=0,ymax=10,ystep=1]
\tkzGrid
\tkzAxeXY
\tkzFct[domain=0:10,color=red,very thick]%
{ (-x**2 + 4.9*x - 9.7)*exp(-x) + 9.7 };
\end{tikzpicture}
\item Il faut dériver $F(x)$ et vérifier que $F'(x) = f(x)$.
\item $\ds \int_0^4 f(x) \; dx = F(4) - F(0) = \frac{11.2}{e^{4}} + 32.0$
\item La quantité calculée à la question précédente se retrouve 4fois pour former le bassin. Il faut ensuite prendre en compte l'échelle, comme 1unité de longueur correspond à 15m, une unité d'air correspond à $15\times15 = 225m^2$. Ainsi l'aire du bassin est égale à
\[
(\frac{11.2}{e^{4}} + 32.0)\times 4 \times 15^2 = 28985.00000
\]
\end{enumerate}
\end{solution}
\begin{exercise}[subtitle={Bassin}]
Le clinker est un constituant du ciment qui résulte de la cuisson d'un mélange composé de calcaire et d'argile. La fabrication du clinker nécessite des fours à très haute température qui libèrent dans l'air une grande quantité de dioxyde de carbone (CO$_2$).
Dans une cimenterie, la fabrication du clinker s'effectue de 7 h 30 à 20 h, dans une pièce de volume \np{400000}~dm$^3$.
À 20 h, après une journée de travail, le taux volumique de CO$_2$ dans la pièce est de 0.7\,\%.
\begin{enumerate}
\item Justifier que le volume de CO$_2$ présent dans cette pièce à 20 h est de \np{2800}~dm$^3$ .
\item On modélise le volume de CO$_2$ présent dans la pièce par une fonction du temps $t$ écoulé après 20h (exprimé en minutes) qui pour formule $V(t) = V_0e^{-0.06t} + 400$
\begin{enumerate}
\item Démontrer que $V_0$ est égale à \np{2400}.
\item Quel sera, au dm$^3$ près, le volume de CO$_2$ dans cette pièce à 23 h ?
\item Démontrer que $V'(t) = - 144.0 e^{- 0.06 t}$.
\item Étudier le signe de $V'(t)$ puis en déduire le sens de variation de $V(t)$.
\item Que peut-on dire du volume de CO$_2$ quand $t$ devient grand?
\end{enumerate}
\end{enumerate}
\end{exercise}
\begin{solution}
\begin{enumerate}
\item Volume à 20h: $400000\times 0.006999999999999999 = 2800$
\item
\begin{enumerate}
\item $t=0$ correspond à 20h.
Donc $V(0) = 2800 = V_0e^{-0.06\times 0} + 400 = V_0 + 400$
Donc $V_0 = 2800 - 400 = 2400$
\item Il faut calculer $V(t)$ pour $t = 3$ donc
\[
V(3) = 2404.65
\]
\item Pas de correction pour cette question.
\item Pas de correction pour cette question.
\item Pas de correction pour cette question.
\end{enumerate}
\end{enumerate}
\end{solution}
\end{document}
%%% Local Variables:
%%% mode: latex
%%% TeX-master: "master"
%%% End:

View File

@ -0,0 +1,121 @@
\documentclass[a4paper,10pt]{article}
\usepackage{myXsim}
\usepackage{tasks}
% Title Page
\title{DM1 \hfill BUDIN Nathan}
\tribe{TST sti2d}
\date{\hfillÀ render pour le jeudi 25 février}
\xsimsetup{
solution/print = false
}
\begin{document}
\maketitle
\begin{exercise}[subtitle={Complexes}]
\begin{enumerate}
\item Mettre le nombre complexe suivant sous forme algébrique $z_1 = \dfrac{7 + 2 i}{-2 + 3 i} $
\item Mettre le complexe suivante sous forme exponentielle $z_2 = - 6 \sqrt{3} - 6 i$
\item Mettre le complexe suivante sous forme exponentielle $z_3 = - 4 \sqrt{3} + 4 i$
\item Calculer le produit $z_4=z_2\times z_3$ donner le résultat sous forme exponentielle puis algébrique.
\item Calculer le quotient $z_5=\frac{z_2}{z_3}$ donner le résultat sous forme exponentielle puis algébrique.
\end{enumerate}
\end{exercise}
\begin{solution}
\begin{enumerate}
\item $z_1 = - \frac{8}{13} - \frac{25 i}{13}$
\item $z_3 = 12 e^{- \frac{5 i \pi}{6}}$
\item $z_4 = 96 e^{0} = 96 = 96.0$
\item $z_5 = \frac{3}{2} e^{- \frac{5 i \pi}{3}} = \frac{3}{4} + \frac{3 \sqrt{3} i}{4} = 0.75 + 1.3 i$
\end{enumerate}
\end{solution}
\begin{exercise}[subtitle={Bassin}]
Le tour d'un bassin au niveau du sol présente deux axes de symétrie : laxe des abscisses et la droite déquation $x=4$. Il est obtenu par symétrie de la courbe $\mathcal{C_f}$ sur $\intFF{0}{4}$$f$ est la fonction définie par
\[
f(x) = \left(- x^{2} + 4.9 x - 6.0\right) e^{- x} + 6.0
\]
On admet que sur $\intFF{0}{4}$ la fonction $f$ est positive.
\begin{enumerate}
\item Sur un repère, tracer l'allure de la courbe $\mathcal{C}_f$, les axes de symétries puis compléter pour dessiner la forme du bassin.
\item Montrer que la fonction $f$ admet comme primitive sur $\R$ la fonction $F$ définie par
\[
F(x) = 6.0 x + \left( x^{2} - 2.9 x + 3.1\right) e^{- x}
\]
\item Calculer la quantité $\ds \int_0^4 f(x) \; dx$, vous donnerez le résultat sous forme exacte. Interpréter le résultat et reportez cette quantité sur le graphique.
\item On considère que l'échelle de votre graphique est de 1unité pour 15m. Calculer l'aire du bassin. Vous donnerez un résultat arrondi au $m^2$ près.
\end{enumerate}
\end{exercise}
\begin{solution}
\begin{enumerate}
\item
\begin{tikzpicture}[baseline=(a.north), xscale=1, yscale=0.5]
\tkzInit[xmin=0,xmax=5,xstep=1,
ymin=0,ymax=10,ystep=1]
\tkzGrid
\tkzAxeXY
\tkzFct[domain=0:10,color=red,very thick]%
{ (-x**2 + 4.9*x - 6.0)*exp(-x) + 6.0 };
\end{tikzpicture}
\item Il faut dériver $F(x)$ et vérifier que $F'(x) = f(x)$.
\item $\ds \int_0^4 f(x) \; dx = F(4) - F(0) = \frac{7.5}{e^{4}} + 20.9$
\item La quantité calculée à la question précédente se retrouve 4fois pour former le bassin. Il faut ensuite prendre en compte l'échelle, comme 1unité de longueur correspond à 15m, une unité d'air correspond à $15\times15 = 225m^2$. Ainsi l'aire du bassin est égale à
\[
(\frac{7.5}{e^{4}} + 20.9)\times 4 \times 15^2 = 18934.00000
\]
\end{enumerate}
\end{solution}
\begin{exercise}[subtitle={Bassin}]
Le clinker est un constituant du ciment qui résulte de la cuisson d'un mélange composé de calcaire et d'argile. La fabrication du clinker nécessite des fours à très haute température qui libèrent dans l'air une grande quantité de dioxyde de carbone (CO$_2$).
Dans une cimenterie, la fabrication du clinker s'effectue de 7 h 30 à 20 h, dans une pièce de volume \np{400000}~dm$^3$.
À 20 h, après une journée de travail, le taux volumique de CO$_2$ dans la pièce est de 0.6\,\%.
\begin{enumerate}
\item Justifier que le volume de CO$_2$ présent dans cette pièce à 20 h est de \np{2400}~dm$^3$ .
\item On modélise le volume de CO$_2$ présent dans la pièce par une fonction du temps $t$ écoulé après 20h (exprimé en minutes) qui pour formule $V(t) = V_0e^{-0.0t} + 580$
\begin{enumerate}
\item Démontrer que $V_0$ est égale à \np{1820}.
\item Quel sera, au dm$^3$ près, le volume de CO$_2$ dans cette pièce à 22 h ?
\item Démontrer que $V'(t) = 0$.
\item Étudier le signe de $V'(t)$ puis en déduire le sens de variation de $V(t)$.
\item Que peut-on dire du volume de CO$_2$ quand $t$ devient grand?
\end{enumerate}
\end{enumerate}
\end{exercise}
\begin{solution}
\begin{enumerate}
\item Volume à 20h: $400000\times 0.006 = 2400$
\item
\begin{enumerate}
\item $t=0$ correspond à 20h.
Donc $V(0) = 2400 = V_0e^{-0.0\times 0} + 580 = V_0 + 580$
Donc $V_0 = 2400 - 580 = 1820$
\item Il faut calculer $V(t)$ pour $t = 2$ donc
\[
V(2) = 2400
\]
\item Pas de correction pour cette question.
\item Pas de correction pour cette question.
\item Pas de correction pour cette question.
\end{enumerate}
\end{enumerate}
\end{solution}
\end{document}
%%% Local Variables:
%%% mode: latex
%%% TeX-master: "master"
%%% End:

View File

@ -0,0 +1,121 @@
\documentclass[a4paper,10pt]{article}
\usepackage{myXsim}
\usepackage{tasks}
% Title Page
\title{DM1 \hfill CHION Léa}
\tribe{TST sti2d}
\date{\hfillÀ render pour le jeudi 25 février}
\xsimsetup{
solution/print = false
}
\begin{document}
\maketitle
\begin{exercise}[subtitle={Complexes}]
\begin{enumerate}
\item Mettre le nombre complexe suivant sous forme algébrique $z_1 = \dfrac{9 + 6 i}{-3 + 5 i} $
\item Mettre le complexe suivante sous forme exponentielle $z_2 = -7 + 7 \sqrt{3} i$
\item Mettre le complexe suivante sous forme exponentielle $z_3 = -5 - 5 \sqrt{3} i$
\item Calculer le produit $z_4=z_2\times z_3$ donner le résultat sous forme exponentielle puis algébrique.
\item Calculer le quotient $z_5=\frac{z_2}{z_3}$ donner le résultat sous forme exponentielle puis algébrique.
\end{enumerate}
\end{exercise}
\begin{solution}
\begin{enumerate}
\item $z_1 = \frac{3}{34} - \frac{63 i}{34}$
\item $z_3 = 14 e^{\frac{2 i \pi}{3}}$
\item $z_4 = 140 e^{0} = 140 = 140.0$
\item $z_5 = \frac{7}{5} e^{\frac{4 i \pi}{3}} = - \frac{7}{10} - \frac{7 \sqrt{3} i}{10} = -0.7 - 1.21 i$
\end{enumerate}
\end{solution}
\begin{exercise}[subtitle={Bassin}]
Le tour d'un bassin au niveau du sol présente deux axes de symétrie : laxe des abscisses et la droite déquation $x=4$. Il est obtenu par symétrie de la courbe $\mathcal{C_f}$ sur $\intFF{0}{4}$$f$ est la fonction définie par
\[
f(x) = \left(- x^{2} + 6.2 x - 7.3\right) e^{- x} + 7.3
\]
On admet que sur $\intFF{0}{4}$ la fonction $f$ est positive.
\begin{enumerate}
\item Sur un repère, tracer l'allure de la courbe $\mathcal{C}_f$, les axes de symétries puis compléter pour dessiner la forme du bassin.
\item Montrer que la fonction $f$ admet comme primitive sur $\R$ la fonction $F$ définie par
\[
F(x) = 7.3 x + \left( x^{2} - 4.2 x + 3.1\right) e^{- x}
\]
\item Calculer la quantité $\ds \int_0^4 f(x) \; dx$, vous donnerez le résultat sous forme exacte. Interpréter le résultat et reportez cette quantité sur le graphique.
\item On considère que l'échelle de votre graphique est de 1unité pour 15m. Calculer l'aire du bassin. Vous donnerez un résultat arrondi au $m^2$ près.
\end{enumerate}
\end{exercise}
\begin{solution}
\begin{enumerate}
\item
\begin{tikzpicture}[baseline=(a.north), xscale=1, yscale=0.5]
\tkzInit[xmin=0,xmax=5,xstep=1,
ymin=0,ymax=10,ystep=1]
\tkzGrid
\tkzAxeXY
\tkzFct[domain=0:10,color=red,very thick]%
{ (-x**2 + 6.2*x - 7.3)*exp(-x) + 7.3 };
\end{tikzpicture}
\item Il faut dériver $F(x)$ et vérifier que $F'(x) = f(x)$.
\item $\ds \int_0^4 f(x) \; dx = F(4) - F(0) = \frac{2.3}{e^{4}} + 26.1$
\item La quantité calculée à la question précédente se retrouve 4fois pour former le bassin. Il faut ensuite prendre en compte l'échelle, comme 1unité de longueur correspond à 15m, une unité d'air correspond à $15\times15 = 225m^2$. Ainsi l'aire du bassin est égale à
\[
(\frac{2.3}{e^{4}} + 26.1)\times 4 \times 15^2 = 23528.00000
\]
\end{enumerate}
\end{solution}
\begin{exercise}[subtitle={Bassin}]
Le clinker est un constituant du ciment qui résulte de la cuisson d'un mélange composé de calcaire et d'argile. La fabrication du clinker nécessite des fours à très haute température qui libèrent dans l'air une grande quantité de dioxyde de carbone (CO$_2$).
Dans une cimenterie, la fabrication du clinker s'effectue de 7 h 30 à 20 h, dans une pièce de volume \np{300000}~dm$^3$.
À 20 h, après une journée de travail, le taux volumique de CO$_2$ dans la pièce est de 0.6\,\%.
\begin{enumerate}
\item Justifier que le volume de CO$_2$ présent dans cette pièce à 20 h est de \np{1800}~dm$^3$ .
\item On modélise le volume de CO$_2$ présent dans la pièce par une fonction du temps $t$ écoulé après 20h (exprimé en minutes) qui pour formule $V(t) = V_0e^{-0.01t} + 260$
\begin{enumerate}
\item Démontrer que $V_0$ est égale à \np{1540}.
\item Quel sera, au dm$^3$ près, le volume de CO$_2$ dans cette pièce à 22 h ?
\item Démontrer que $V'(t) = - 15.4 e^{- 0.01 t}$.
\item Étudier le signe de $V'(t)$ puis en déduire le sens de variation de $V(t)$.
\item Que peut-on dire du volume de CO$_2$ quand $t$ devient grand?
\end{enumerate}
\end{enumerate}
\end{exercise}
\begin{solution}
\begin{enumerate}
\item Volume à 20h: $300000\times 0.006 = 1800$
\item
\begin{enumerate}
\item $t=0$ correspond à 20h.
Donc $V(0) = 1800 = V_0e^{-0.01\times 0} + 260 = V_0 + 260$
Donc $V_0 = 1800 - 260 = 1540$
\item Il faut calculer $V(t)$ pour $t = 2$ donc
\[
V(2) = 1769.51
\]
\item Pas de correction pour cette question.
\item Pas de correction pour cette question.
\item Pas de correction pour cette question.
\end{enumerate}
\end{enumerate}
\end{solution}
\end{document}
%%% Local Variables:
%%% mode: latex
%%% TeX-master: "master"
%%% End:

View File

@ -0,0 +1,121 @@
\documentclass[a4paper,10pt]{article}
\usepackage{myXsim}
\usepackage{tasks}
% Title Page
\title{DM1 \hfill CLAIN Avinash}
\tribe{TST sti2d}
\date{\hfillÀ render pour le jeudi 25 février}
\xsimsetup{
solution/print = false
}
\begin{document}
\maketitle
\begin{exercise}[subtitle={Complexes}]
\begin{enumerate}
\item Mettre le nombre complexe suivant sous forme algébrique $z_1 = \dfrac{4 + 10 i}{-8 + 3 i} $
\item Mettre le complexe suivante sous forme exponentielle $z_2 = - 4 \sqrt{3} + 4 i$
\item Mettre le complexe suivante sous forme exponentielle $z_3 = 2 \sqrt{2} - 2 \sqrt{2} i$
\item Calculer le produit $z_4=z_2\times z_3$ donner le résultat sous forme exponentielle puis algébrique.
\item Calculer le quotient $z_5=\frac{z_2}{z_3}$ donner le résultat sous forme exponentielle puis algébrique.
\end{enumerate}
\end{exercise}
\begin{solution}
\begin{enumerate}
\item $z_1 = - \frac{2}{73} - \frac{92 i}{73}$
\item $z_3 = 8 e^{\frac{5 i \pi}{6}}$
\item $z_4 = 32 e^{\frac{7 i \pi}{12}} = - 8 \sqrt{6} + 8 \sqrt{2} + i \left(8 \sqrt{2} + 8 \sqrt{6}\right) = -8.28 + 30.9 i$
\item $z_5 = 2 e^{\frac{13 i \pi}{12}} = - \frac{\sqrt{6}}{2} - \frac{\sqrt{2}}{2} + i \left(- \frac{\sqrt{6}}{2} + \frac{\sqrt{2}}{2}\right) = -1.93 - 0.518 i$
\end{enumerate}
\end{solution}
\begin{exercise}[subtitle={Bassin}]
Le tour d'un bassin au niveau du sol présente deux axes de symétrie : laxe des abscisses et la droite déquation $x=4$. Il est obtenu par symétrie de la courbe $\mathcal{C_f}$ sur $\intFF{0}{4}$$f$ est la fonction définie par
\[
f(x) = \left(- x^{2} + 1.8 x - 4.9\right) e^{- x} + 4.9
\]
On admet que sur $\intFF{0}{4}$ la fonction $f$ est positive.
\begin{enumerate}
\item Sur un repère, tracer l'allure de la courbe $\mathcal{C}_f$, les axes de symétries puis compléter pour dessiner la forme du bassin.
\item Montrer que la fonction $f$ admet comme primitive sur $\R$ la fonction $F$ définie par
\[
F(x) = 4.9 x + \left( x^{2} + 0.2 x + 5.1\right) e^{- x}
\]
\item Calculer la quantité $\ds \int_0^4 f(x) \; dx$, vous donnerez le résultat sous forme exacte. Interpréter le résultat et reportez cette quantité sur le graphique.
\item On considère que l'échelle de votre graphique est de 1unité pour 15m. Calculer l'aire du bassin. Vous donnerez un résultat arrondi au $m^2$ près.
\end{enumerate}
\end{exercise}
\begin{solution}
\begin{enumerate}
\item
\begin{tikzpicture}[baseline=(a.north), xscale=1, yscale=0.5]
\tkzInit[xmin=0,xmax=5,xstep=1,
ymin=0,ymax=10,ystep=1]
\tkzGrid
\tkzAxeXY
\tkzFct[domain=0:10,color=red,very thick]%
{ (-x**2 + 1.8*x - 4.9)*exp(-x) + 4.9 };
\end{tikzpicture}
\item Il faut dériver $F(x)$ et vérifier que $F'(x) = f(x)$.
\item $\ds \int_0^4 f(x) \; dx = F(4) - F(0) = \frac{21.9}{e^{4}} + 14.5$
\item La quantité calculée à la question précédente se retrouve 4fois pour former le bassin. Il faut ensuite prendre en compte l'échelle, comme 1unité de longueur correspond à 15m, une unité d'air correspond à $15\times15 = 225m^2$. Ainsi l'aire du bassin est égale à
\[
(\frac{21.9}{e^{4}} + 14.5)\times 4 \times 15^2 = 13411.00000
\]
\end{enumerate}
\end{solution}
\begin{exercise}[subtitle={Bassin}]
Le clinker est un constituant du ciment qui résulte de la cuisson d'un mélange composé de calcaire et d'argile. La fabrication du clinker nécessite des fours à très haute température qui libèrent dans l'air une grande quantité de dioxyde de carbone (CO$_2$).
Dans une cimenterie, la fabrication du clinker s'effectue de 7 h 30 à 20 h, dans une pièce de volume \np{1000000}~dm$^3$.
À 20 h, après une journée de travail, le taux volumique de CO$_2$ dans la pièce est de 0.8\,\%.
\begin{enumerate}
\item Justifier que le volume de CO$_2$ présent dans cette pièce à 20 h est de \np{8000}~dm$^3$ .
\item On modélise le volume de CO$_2$ présent dans la pièce par une fonction du temps $t$ écoulé après 20h (exprimé en minutes) qui pour formule $V(t) = V_0e^{-0.09t} + 580$
\begin{enumerate}
\item Démontrer que $V_0$ est égale à \np{7420}.
\item Quel sera, au dm$^3$ près, le volume de CO$_2$ dans cette pièce à 21 h ?
\item Démontrer que $V'(t) = - 667.8 e^{- 0.09 t}$.
\item Étudier le signe de $V'(t)$ puis en déduire le sens de variation de $V(t)$.
\item Que peut-on dire du volume de CO$_2$ quand $t$ devient grand?
\end{enumerate}
\end{enumerate}
\end{exercise}
\begin{solution}
\begin{enumerate}
\item Volume à 20h: $1000000\times 0.008 = 8000$
\item
\begin{enumerate}
\item $t=0$ correspond à 20h.
Donc $V(0) = 8000 = V_0e^{-0.09\times 0} + 580 = V_0 + 580$
Donc $V_0 = 8000 - 580 = 7420$
\item Il faut calculer $V(t)$ pour $t = 1$ donc
\[
V(1) = 7361.37
\]
\item Pas de correction pour cette question.
\item Pas de correction pour cette question.
\item Pas de correction pour cette question.
\end{enumerate}
\end{enumerate}
\end{solution}
\end{document}
%%% Local Variables:
%%% mode: latex
%%% TeX-master: "master"
%%% End:

View File

@ -0,0 +1,121 @@
\documentclass[a4paper,10pt]{article}
\usepackage{myXsim}
\usepackage{tasks}
% Title Page
\title{DM1 \hfill COUBAT Alexis}
\tribe{TST sti2d}
\date{\hfillÀ render pour le jeudi 25 février}
\xsimsetup{
solution/print = false
}
\begin{document}
\maketitle
\begin{exercise}[subtitle={Complexes}]
\begin{enumerate}
\item Mettre le nombre complexe suivant sous forme algébrique $z_1 = \dfrac{7 + 3 i}{-2 + 2 i} $
\item Mettre le complexe suivante sous forme exponentielle $z_2 = 1 - \sqrt{3} i$
\item Mettre le complexe suivante sous forme exponentielle $z_3 = - 2 \sqrt{3} + 2 i$
\item Calculer le produit $z_4=z_2\times z_3$ donner le résultat sous forme exponentielle puis algébrique.
\item Calculer le quotient $z_5=\frac{z_2}{z_3}$ donner le résultat sous forme exponentielle puis algébrique.
\end{enumerate}
\end{exercise}
\begin{solution}
\begin{enumerate}
\item $z_1 = -1 - \frac{5 i}{2}$
\item $z_3 = 2 e^{- \frac{i \pi}{3}}$
\item $z_4 = 8 e^{\frac{i \pi}{2}} = 8 i = 8.0 i$
\item $z_5 = \frac{1}{2} e^{- \frac{7 i \pi}{6}} = - \frac{\sqrt{3}}{4} + \frac{i}{4} = -0.433 + 0.25 i$
\end{enumerate}
\end{solution}
\begin{exercise}[subtitle={Bassin}]
Le tour d'un bassin au niveau du sol présente deux axes de symétrie : laxe des abscisses et la droite déquation $x=4$. Il est obtenu par symétrie de la courbe $\mathcal{C_f}$ sur $\intFF{0}{4}$$f$ est la fonction définie par
\[
f(x) = \left(- x^{2} + 5.6 x - 4.5\right) e^{- x} + 4.5
\]
On admet que sur $\intFF{0}{4}$ la fonction $f$ est positive.
\begin{enumerate}
\item Sur un repère, tracer l'allure de la courbe $\mathcal{C}_f$, les axes de symétries puis compléter pour dessiner la forme du bassin.
\item Montrer que la fonction $f$ admet comme primitive sur $\R$ la fonction $F$ définie par
\[
F(x) = 4.5 x + \left( x^{2} - 3.6 x + 0.9\right) e^{- x}
\]
\item Calculer la quantité $\ds \int_0^4 f(x) \; dx$, vous donnerez le résultat sous forme exacte. Interpréter le résultat et reportez cette quantité sur le graphique.
\item On considère que l'échelle de votre graphique est de 1unité pour 15m. Calculer l'aire du bassin. Vous donnerez un résultat arrondi au $m^2$ près.
\end{enumerate}
\end{exercise}
\begin{solution}
\begin{enumerate}
\item
\begin{tikzpicture}[baseline=(a.north), xscale=1, yscale=0.5]
\tkzInit[xmin=0,xmax=5,xstep=1,
ymin=0,ymax=10,ystep=1]
\tkzGrid
\tkzAxeXY
\tkzFct[domain=0:10,color=red,very thick]%
{ (-x**2 + 5.6*x - 4.5)*exp(-x) + 4.5 };
\end{tikzpicture}
\item Il faut dériver $F(x)$ et vérifier que $F'(x) = f(x)$.
\item $\ds \int_0^4 f(x) \; dx = F(4) - F(0) = \frac{2.5}{e^{4}} + 17.1$
\item La quantité calculée à la question précédente se retrouve 4fois pour former le bassin. Il faut ensuite prendre en compte l'échelle, comme 1unité de longueur correspond à 15m, une unité d'air correspond à $15\times15 = 225m^2$. Ainsi l'aire du bassin est égale à
\[
(\frac{2.5}{e^{4}} + 17.1)\times 4 \times 15^2 = 15431.00000
\]
\end{enumerate}
\end{solution}
\begin{exercise}[subtitle={Bassin}]
Le clinker est un constituant du ciment qui résulte de la cuisson d'un mélange composé de calcaire et d'argile. La fabrication du clinker nécessite des fours à très haute température qui libèrent dans l'air une grande quantité de dioxyde de carbone (CO$_2$).
Dans une cimenterie, la fabrication du clinker s'effectue de 7 h 30 à 20 h, dans une pièce de volume \np{800000}~dm$^3$.
À 20 h, après une journée de travail, le taux volumique de CO$_2$ dans la pièce est de 0.7\,\%.
\begin{enumerate}
\item Justifier que le volume de CO$_2$ présent dans cette pièce à 20 h est de \np{5600}~dm$^3$ .
\item On modélise le volume de CO$_2$ présent dans la pièce par une fonction du temps $t$ écoulé après 20h (exprimé en minutes) qui pour formule $V(t) = V_0e^{-0.02t} + 340$
\begin{enumerate}
\item Démontrer que $V_0$ est égale à \np{5260}.
\item Quel sera, au dm$^3$ près, le volume de CO$_2$ dans cette pièce à 24 h ?
\item Démontrer que $V'(t) = - 105.2 e^{- 0.02 t}$.
\item Étudier le signe de $V'(t)$ puis en déduire le sens de variation de $V(t)$.
\item Que peut-on dire du volume de CO$_2$ quand $t$ devient grand?
\end{enumerate}
\end{enumerate}
\end{exercise}
\begin{solution}
\begin{enumerate}
\item Volume à 20h: $800000\times 0.006999999999999999 = 5600$
\item
\begin{enumerate}
\item $t=0$ correspond à 20h.
Donc $V(0) = 5600 = V_0e^{-0.02\times 0} + 340 = V_0 + 340$
Donc $V_0 = 5600 - 340 = 5260$
\item Il faut calculer $V(t)$ pour $t = 4$ donc
\[
V(4) = 5195.59
\]
\item Pas de correction pour cette question.
\item Pas de correction pour cette question.
\item Pas de correction pour cette question.
\end{enumerate}
\end{enumerate}
\end{solution}
\end{document}
%%% Local Variables:
%%% mode: latex
%%% TeX-master: "master"
%%% End:

View File

@ -0,0 +1,121 @@
\documentclass[a4paper,10pt]{article}
\usepackage{myXsim}
\usepackage{tasks}
% Title Page
\title{DM1 \hfill EVRARD Jules}
\tribe{TST sti2d}
\date{\hfillÀ render pour le jeudi 25 février}
\xsimsetup{
solution/print = false
}
\begin{document}
\maketitle
\begin{exercise}[subtitle={Complexes}]
\begin{enumerate}
\item Mettre le nombre complexe suivant sous forme algébrique $z_1 = \dfrac{8 + 3 i}{-9 + 3 i} $
\item Mettre le complexe suivante sous forme exponentielle $z_2 = 3 - 3 \sqrt{3} i$
\item Mettre le complexe suivante sous forme exponentielle $z_3 = \sqrt{2} - \sqrt{2} i$
\item Calculer le produit $z_4=z_2\times z_3$ donner le résultat sous forme exponentielle puis algébrique.
\item Calculer le quotient $z_5=\frac{z_2}{z_3}$ donner le résultat sous forme exponentielle puis algébrique.
\end{enumerate}
\end{exercise}
\begin{solution}
\begin{enumerate}
\item $z_1 = - \frac{7}{10} - \frac{17 i}{30}$
\item $z_3 = 6 e^{- \frac{i \pi}{3}}$
\item $z_4 = 12 e^{- \frac{7 i \pi}{12}} = - 3 \sqrt{6} + 3 \sqrt{2} + i \left(- 3 \sqrt{6} - 3 \sqrt{2}\right) = -3.11 - 11.6 i$
\item $z_5 = 3 e^{- \frac{i \pi}{12}} = \frac{3 \sqrt{2}}{4} + \frac{3 \sqrt{6}}{4} + i \left(- \frac{3 \sqrt{6}}{4} + \frac{3 \sqrt{2}}{4}\right) = 2.9 - 0.776 i$
\end{enumerate}
\end{solution}
\begin{exercise}[subtitle={Bassin}]
Le tour d'un bassin au niveau du sol présente deux axes de symétrie : laxe des abscisses et la droite déquation $x=4$. Il est obtenu par symétrie de la courbe $\mathcal{C_f}$ sur $\intFF{0}{4}$$f$ est la fonction définie par
\[
f(x) = \left(- x^{2} + 0.9 x - 3.2\right) e^{- x} + 3.2
\]
On admet que sur $\intFF{0}{4}$ la fonction $f$ est positive.
\begin{enumerate}
\item Sur un repère, tracer l'allure de la courbe $\mathcal{C}_f$, les axes de symétries puis compléter pour dessiner la forme du bassin.
\item Montrer que la fonction $f$ admet comme primitive sur $\R$ la fonction $F$ définie par
\[
F(x) = 3.2 x + \left( x^{2} + 1.1 x + 4.3\right) e^{- x}
\]
\item Calculer la quantité $\ds \int_0^4 f(x) \; dx$, vous donnerez le résultat sous forme exacte. Interpréter le résultat et reportez cette quantité sur le graphique.
\item On considère que l'échelle de votre graphique est de 1unité pour 15m. Calculer l'aire du bassin. Vous donnerez un résultat arrondi au $m^2$ près.
\end{enumerate}
\end{exercise}
\begin{solution}
\begin{enumerate}
\item
\begin{tikzpicture}[baseline=(a.north), xscale=1, yscale=0.5]
\tkzInit[xmin=0,xmax=5,xstep=1,
ymin=0,ymax=10,ystep=1]
\tkzGrid
\tkzAxeXY
\tkzFct[domain=0:10,color=red,very thick]%
{ (-x**2 + 0.9*x - 3.2)*exp(-x) + 3.2 };
\end{tikzpicture}
\item Il faut dériver $F(x)$ et vérifier que $F'(x) = f(x)$.
\item $\ds \int_0^4 f(x) \; dx = F(4) - F(0) = \frac{24.7}{e^{4}} + 8.5$
\item La quantité calculée à la question précédente se retrouve 4fois pour former le bassin. Il faut ensuite prendre en compte l'échelle, comme 1unité de longueur correspond à 15m, une unité d'air correspond à $15\times15 = 225m^2$. Ainsi l'aire du bassin est égale à
\[
(\frac{24.7}{e^{4}} + 8.5)\times 4 \times 15^2 = 8057.000000
\]
\end{enumerate}
\end{solution}
\begin{exercise}[subtitle={Bassin}]
Le clinker est un constituant du ciment qui résulte de la cuisson d'un mélange composé de calcaire et d'argile. La fabrication du clinker nécessite des fours à très haute température qui libèrent dans l'air une grande quantité de dioxyde de carbone (CO$_2$).
Dans une cimenterie, la fabrication du clinker s'effectue de 7 h 30 à 20 h, dans une pièce de volume \np{800000}~dm$^3$.
À 20 h, après une journée de travail, le taux volumique de CO$_2$ dans la pièce est de 1.0\,\%.
\begin{enumerate}
\item Justifier que le volume de CO$_2$ présent dans cette pièce à 20 h est de \np{8000}~dm$^3$ .
\item On modélise le volume de CO$_2$ présent dans la pièce par une fonction du temps $t$ écoulé après 20h (exprimé en minutes) qui pour formule $V(t) = V_0e^{-0.02t} + 560$
\begin{enumerate}
\item Démontrer que $V_0$ est égale à \np{7440}.
\item Quel sera, au dm$^3$ près, le volume de CO$_2$ dans cette pièce à 23 h ?
\item Démontrer que $V'(t) = - 148.8 e^{- 0.02 t}$.
\item Étudier le signe de $V'(t)$ puis en déduire le sens de variation de $V(t)$.
\item Que peut-on dire du volume de CO$_2$ quand $t$ devient grand?
\end{enumerate}
\end{enumerate}
\end{exercise}
\begin{solution}
\begin{enumerate}
\item Volume à 20h: $800000\times 0.01 = 8000$
\item
\begin{enumerate}
\item $t=0$ correspond à 20h.
Donc $V(0) = 8000 = V_0e^{-0.02\times 0} + 560 = V_0 + 560$
Donc $V_0 = 8000 - 560 = 7440$
\item Il faut calculer $V(t)$ pour $t = 3$ donc
\[
V(3) = 7566.73
\]
\item Pas de correction pour cette question.
\item Pas de correction pour cette question.
\item Pas de correction pour cette question.
\end{enumerate}
\end{enumerate}
\end{solution}
\end{document}
%%% Local Variables:
%%% mode: latex
%%% TeX-master: "master"
%%% End:

View File

@ -0,0 +1,121 @@
\documentclass[a4paper,10pt]{article}
\usepackage{myXsim}
\usepackage{tasks}
% Title Page
\title{DM1 \hfill HADJRAS Mohcine}
\tribe{TST sti2d}
\date{\hfillÀ render pour le jeudi 25 février}
\xsimsetup{
solution/print = false
}
\begin{document}
\maketitle
\begin{exercise}[subtitle={Complexes}]
\begin{enumerate}
\item Mettre le nombre complexe suivant sous forme algébrique $z_1 = \dfrac{5 + 3 i}{-5 + 7 i} $
\item Mettre le complexe suivante sous forme exponentielle $z_2 = 2 \sqrt{3} + 2 i$
\item Mettre le complexe suivante sous forme exponentielle $z_3 = -10 - 10 \sqrt{3} i$
\item Calculer le produit $z_4=z_2\times z_3$ donner le résultat sous forme exponentielle puis algébrique.
\item Calculer le quotient $z_5=\frac{z_2}{z_3}$ donner le résultat sous forme exponentielle puis algébrique.
\end{enumerate}
\end{exercise}
\begin{solution}
\begin{enumerate}
\item $z_1 = - \frac{2}{37} - \frac{25 i}{37}$
\item $z_3 = 4 e^{\frac{i \pi}{6}}$
\item $z_4 = 80 e^{- \frac{i \pi}{2}} = - 80 i = - 80.0 i$
\item $z_5 = \frac{1}{5} e^{\frac{5 i \pi}{6}} = - \frac{\sqrt{3}}{10} + \frac{i}{10} = -0.173 + 0.1 i$
\end{enumerate}
\end{solution}
\begin{exercise}[subtitle={Bassin}]
Le tour d'un bassin au niveau du sol présente deux axes de symétrie : laxe des abscisses et la droite déquation $x=4$. Il est obtenu par symétrie de la courbe $\mathcal{C_f}$ sur $\intFF{0}{4}$$f$ est la fonction définie par
\[
f(x) = \left(- x^{2} + 7.5 x - 9.0\right) e^{- x} + 9.0
\]
On admet que sur $\intFF{0}{4}$ la fonction $f$ est positive.
\begin{enumerate}
\item Sur un repère, tracer l'allure de la courbe $\mathcal{C}_f$, les axes de symétries puis compléter pour dessiner la forme du bassin.
\item Montrer que la fonction $f$ admet comme primitive sur $\R$ la fonction $F$ définie par
\[
F(x) = 9.0 x + \left( x^{2} - 5.5 x + 3.5\right) e^{- x}
\]
\item Calculer la quantité $\ds \int_0^4 f(x) \; dx$, vous donnerez le résultat sous forme exacte. Interpréter le résultat et reportez cette quantité sur le graphique.
\item On considère que l'échelle de votre graphique est de 1unité pour 15m. Calculer l'aire du bassin. Vous donnerez un résultat arrondi au $m^2$ près.
\end{enumerate}
\end{exercise}
\begin{solution}
\begin{enumerate}
\item
\begin{tikzpicture}[baseline=(a.north), xscale=1, yscale=0.5]
\tkzInit[xmin=0,xmax=5,xstep=1,
ymin=0,ymax=10,ystep=1]
\tkzGrid
\tkzAxeXY
\tkzFct[domain=0:10,color=red,very thick]%
{ (-x**2 + 7.5*x - 9.0)*exp(-x) + 9.0 };
\end{tikzpicture}
\item Il faut dériver $F(x)$ et vérifier que $F'(x) = f(x)$.
\item $\ds \int_0^4 f(x) \; dx = F(4) - F(0) = 32.5 - \frac{2.5}{e^{4}}$
\item La quantité calculée à la question précédente se retrouve 4fois pour former le bassin. Il faut ensuite prendre en compte l'échelle, comme 1unité de longueur correspond à 15m, une unité d'air correspond à $15\times15 = 225m^2$. Ainsi l'aire du bassin est égale à
\[
(32.5 - \frac{2.5}{e^{4}})\times 4 \times 15^2 = 29209.00000
\]
\end{enumerate}
\end{solution}
\begin{exercise}[subtitle={Bassin}]
Le clinker est un constituant du ciment qui résulte de la cuisson d'un mélange composé de calcaire et d'argile. La fabrication du clinker nécessite des fours à très haute température qui libèrent dans l'air une grande quantité de dioxyde de carbone (CO$_2$).
Dans une cimenterie, la fabrication du clinker s'effectue de 7 h 30 à 20 h, dans une pièce de volume \np{1000000}~dm$^3$.
À 20 h, après une journée de travail, le taux volumique de CO$_2$ dans la pièce est de 0.9\,\%.
\begin{enumerate}
\item Justifier que le volume de CO$_2$ présent dans cette pièce à 20 h est de \np{9000}~dm$^3$ .
\item On modélise le volume de CO$_2$ présent dans la pièce par une fonction du temps $t$ écoulé après 20h (exprimé en minutes) qui pour formule $V(t) = V_0e^{-0.01t} + 330$
\begin{enumerate}
\item Démontrer que $V_0$ est égale à \np{8670}.
\item Quel sera, au dm$^3$ près, le volume de CO$_2$ dans cette pièce à 24 h ?
\item Démontrer que $V'(t) = - 86.7 e^{- 0.01 t}$.
\item Étudier le signe de $V'(t)$ puis en déduire le sens de variation de $V(t)$.
\item Que peut-on dire du volume de CO$_2$ quand $t$ devient grand?
\end{enumerate}
\end{enumerate}
\end{exercise}
\begin{solution}
\begin{enumerate}
\item Volume à 20h: $1000000\times 0.009000000000000001 = 9000$
\item
\begin{enumerate}
\item $t=0$ correspond à 20h.
Donc $V(0) = 9000 = V_0e^{-0.01\times 0} + 330 = V_0 + 330$
Donc $V_0 = 9000 - 330 = 8670$
\item Il faut calculer $V(t)$ pour $t = 4$ donc
\[
V(4) = 8660.04
\]
\item Pas de correction pour cette question.
\item Pas de correction pour cette question.
\item Pas de correction pour cette question.
\end{enumerate}
\end{enumerate}
\end{solution}
\end{document}
%%% Local Variables:
%%% mode: latex
%%% TeX-master: "master"
%%% End:

View File

@ -0,0 +1,121 @@
\documentclass[a4paper,10pt]{article}
\usepackage{myXsim}
\usepackage{tasks}
% Title Page
\title{DM1 \hfill HENRIST Maxime}
\tribe{TST sti2d}
\date{\hfillÀ render pour le jeudi 25 février}
\xsimsetup{
solution/print = false
}
\begin{document}
\maketitle
\begin{exercise}[subtitle={Complexes}]
\begin{enumerate}
\item Mettre le nombre complexe suivant sous forme algébrique $z_1 = \dfrac{6 + 6 i}{-2 + 7 i} $
\item Mettre le complexe suivante sous forme exponentielle $z_2 = 1 - \sqrt{3} i$
\item Mettre le complexe suivante sous forme exponentielle $z_3 = 3 \sqrt{2} + 3 \sqrt{2} i$
\item Calculer le produit $z_4=z_2\times z_3$ donner le résultat sous forme exponentielle puis algébrique.
\item Calculer le quotient $z_5=\frac{z_2}{z_3}$ donner le résultat sous forme exponentielle puis algébrique.
\end{enumerate}
\end{exercise}
\begin{solution}
\begin{enumerate}
\item $z_1 = \frac{30}{53} - \frac{54 i}{53}$
\item $z_3 = 2 e^{- \frac{i \pi}{3}}$
\item $z_4 = 12 e^{- \frac{i \pi}{12}} = 3 \sqrt{2} + 3 \sqrt{6} + i \left(- 3 \sqrt{6} + 3 \sqrt{2}\right) = 11.6 - 3.11 i$
\item $z_5 = \frac{1}{3} e^{- \frac{7 i \pi}{12}} = - \frac{\sqrt{6}}{12} + \frac{\sqrt{2}}{12} + i \left(- \frac{\sqrt{6}}{12} - \frac{\sqrt{2}}{12}\right) = -0.0863 - 0.322 i$
\end{enumerate}
\end{solution}
\begin{exercise}[subtitle={Bassin}]
Le tour d'un bassin au niveau du sol présente deux axes de symétrie : laxe des abscisses et la droite déquation $x=4$. Il est obtenu par symétrie de la courbe $\mathcal{C_f}$ sur $\intFF{0}{4}$$f$ est la fonction définie par
\[
f(x) = \left(- x^{2} + 1.3 x - 8.0\right) e^{- x} + 8.0
\]
On admet que sur $\intFF{0}{4}$ la fonction $f$ est positive.
\begin{enumerate}
\item Sur un repère, tracer l'allure de la courbe $\mathcal{C}_f$, les axes de symétries puis compléter pour dessiner la forme du bassin.
\item Montrer que la fonction $f$ admet comme primitive sur $\R$ la fonction $F$ définie par
\[
F(x) = 8.0 x + \left( x^{2} + 0.7 x + 8.7\right) e^{- x}
\]
\item Calculer la quantité $\ds \int_0^4 f(x) \; dx$, vous donnerez le résultat sous forme exacte. Interpréter le résultat et reportez cette quantité sur le graphique.
\item On considère que l'échelle de votre graphique est de 1unité pour 15m. Calculer l'aire du bassin. Vous donnerez un résultat arrondi au $m^2$ près.
\end{enumerate}
\end{exercise}
\begin{solution}
\begin{enumerate}
\item
\begin{tikzpicture}[baseline=(a.north), xscale=1, yscale=0.5]
\tkzInit[xmin=0,xmax=5,xstep=1,
ymin=0,ymax=10,ystep=1]
\tkzGrid
\tkzAxeXY
\tkzFct[domain=0:10,color=red,very thick]%
{ (-x**2 + 1.3*x - 8.0)*exp(-x) + 8.0 };
\end{tikzpicture}
\item Il faut dériver $F(x)$ et vérifier que $F'(x) = f(x)$.
\item $\ds \int_0^4 f(x) \; dx = F(4) - F(0) = \frac{27.5}{e^{4}} + 23.3$
\item La quantité calculée à la question précédente se retrouve 4fois pour former le bassin. Il faut ensuite prendre en compte l'échelle, comme 1unité de longueur correspond à 15m, une unité d'air correspond à $15\times15 = 225m^2$. Ainsi l'aire du bassin est égale à
\[
(\frac{27.5}{e^{4}} + 23.3)\times 4 \times 15^2 = 21423.00000
\]
\end{enumerate}
\end{solution}
\begin{exercise}[subtitle={Bassin}]
Le clinker est un constituant du ciment qui résulte de la cuisson d'un mélange composé de calcaire et d'argile. La fabrication du clinker nécessite des fours à très haute température qui libèrent dans l'air une grande quantité de dioxyde de carbone (CO$_2$).
Dans une cimenterie, la fabrication du clinker s'effectue de 7 h 30 à 20 h, dans une pièce de volume \np{900000}~dm$^3$.
À 20 h, après une journée de travail, le taux volumique de CO$_2$ dans la pièce est de 0.7\,\%.
\begin{enumerate}
\item Justifier que le volume de CO$_2$ présent dans cette pièce à 20 h est de \np{6300}~dm$^3$ .
\item On modélise le volume de CO$_2$ présent dans la pièce par une fonction du temps $t$ écoulé après 20h (exprimé en minutes) qui pour formule $V(t) = V_0e^{-0.1t} + 390$
\begin{enumerate}
\item Démontrer que $V_0$ est égale à \np{5910}.
\item Quel sera, au dm$^3$ près, le volume de CO$_2$ dans cette pièce à 22 h ?
\item Démontrer que $V'(t) = - 591.0 e^{- 0.1 t}$.
\item Étudier le signe de $V'(t)$ puis en déduire le sens de variation de $V(t)$.
\item Que peut-on dire du volume de CO$_2$ quand $t$ devient grand?
\end{enumerate}
\end{enumerate}
\end{exercise}
\begin{solution}
\begin{enumerate}
\item Volume à 20h: $900000\times 0.006999999999999999 = 6300$
\item
\begin{enumerate}
\item $t=0$ correspond à 20h.
Donc $V(0) = 6300 = V_0e^{-0.1\times 0} + 390 = V_0 + 390$
Donc $V_0 = 6300 - 390 = 5910$
\item Il faut calculer $V(t)$ pour $t = 2$ donc
\[
V(2) = 5228.70
\]
\item Pas de correction pour cette question.
\item Pas de correction pour cette question.
\item Pas de correction pour cette question.
\end{enumerate}
\end{enumerate}
\end{solution}
\end{document}
%%% Local Variables:
%%% mode: latex
%%% TeX-master: "master"
%%% End:

View File

@ -0,0 +1,121 @@
\documentclass[a4paper,10pt]{article}
\usepackage{myXsim}
\usepackage{tasks}
% Title Page
\title{DM1 \hfill HUMBERT Rayan}
\tribe{TST sti2d}
\date{\hfillÀ render pour le jeudi 25 février}
\xsimsetup{
solution/print = false
}
\begin{document}
\maketitle
\begin{exercise}[subtitle={Complexes}]
\begin{enumerate}
\item Mettre le nombre complexe suivant sous forme algébrique $z_1 = \dfrac{5 + 10 i}{-5 + 2 i} $
\item Mettre le complexe suivante sous forme exponentielle $z_2 = - 5 \sqrt{3} - 5 i$
\item Mettre le complexe suivante sous forme exponentielle $z_3 = 9 - 9 \sqrt{3} i$
\item Calculer le produit $z_4=z_2\times z_3$ donner le résultat sous forme exponentielle puis algébrique.
\item Calculer le quotient $z_5=\frac{z_2}{z_3}$ donner le résultat sous forme exponentielle puis algébrique.
\end{enumerate}
\end{exercise}
\begin{solution}
\begin{enumerate}
\item $z_1 = - \frac{5}{29} - \frac{60 i}{29}$
\item $z_3 = 10 e^{- \frac{5 i \pi}{6}}$
\item $z_4 = 180 e^{- \frac{7 i \pi}{6}} = - 90 \sqrt{3} + 90 i = -156.0 + 90.0 i$
\item $z_5 = \frac{5}{9} e^{- \frac{i \pi}{2}} = - \frac{5 i}{9} = - 0.556 i$
\end{enumerate}
\end{solution}
\begin{exercise}[subtitle={Bassin}]
Le tour d'un bassin au niveau du sol présente deux axes de symétrie : laxe des abscisses et la droite déquation $x=4$. Il est obtenu par symétrie de la courbe $\mathcal{C_f}$ sur $\intFF{0}{4}$$f$ est la fonction définie par
\[
f(x) = \left(- x^{2} + 0.3 x - 9.7\right) e^{- x} + 9.7
\]
On admet que sur $\intFF{0}{4}$ la fonction $f$ est positive.
\begin{enumerate}
\item Sur un repère, tracer l'allure de la courbe $\mathcal{C}_f$, les axes de symétries puis compléter pour dessiner la forme du bassin.
\item Montrer que la fonction $f$ admet comme primitive sur $\R$ la fonction $F$ définie par
\[
F(x) = 9.7 x + \left( x^{2} + 1.7 x + 11.4\right) e^{- x}
\]
\item Calculer la quantité $\ds \int_0^4 f(x) \; dx$, vous donnerez le résultat sous forme exacte. Interpréter le résultat et reportez cette quantité sur le graphique.
\item On considère que l'échelle de votre graphique est de 1unité pour 15m. Calculer l'aire du bassin. Vous donnerez un résultat arrondi au $m^2$ près.
\end{enumerate}
\end{exercise}
\begin{solution}
\begin{enumerate}
\item
\begin{tikzpicture}[baseline=(a.north), xscale=1, yscale=0.5]
\tkzInit[xmin=0,xmax=5,xstep=1,
ymin=0,ymax=10,ystep=1]
\tkzGrid
\tkzAxeXY
\tkzFct[domain=0:10,color=red,very thick]%
{ (-x**2 + 0.3*x - 9.7)*exp(-x) + 9.7 };
\end{tikzpicture}
\item Il faut dériver $F(x)$ et vérifier que $F'(x) = f(x)$.
\item $\ds \int_0^4 f(x) \; dx = F(4) - F(0) = \frac{34.2}{e^{4}} + 27.4$
\item La quantité calculée à la question précédente se retrouve 4fois pour former le bassin. Il faut ensuite prendre en compte l'échelle, comme 1unité de longueur correspond à 15m, une unité d'air correspond à $15\times15 = 225m^2$. Ainsi l'aire du bassin est égale à
\[
(\frac{34.2}{e^{4}} + 27.4)\times 4 \times 15^2 = 25224.00000
\]
\end{enumerate}
\end{solution}
\begin{exercise}[subtitle={Bassin}]
Le clinker est un constituant du ciment qui résulte de la cuisson d'un mélange composé de calcaire et d'argile. La fabrication du clinker nécessite des fours à très haute température qui libèrent dans l'air une grande quantité de dioxyde de carbone (CO$_2$).
Dans une cimenterie, la fabrication du clinker s'effectue de 7 h 30 à 20 h, dans une pièce de volume \np{600000}~dm$^3$.
À 20 h, après une journée de travail, le taux volumique de CO$_2$ dans la pièce est de 0.6\,\%.
\begin{enumerate}
\item Justifier que le volume de CO$_2$ présent dans cette pièce à 20 h est de \np{3600}~dm$^3$ .
\item On modélise le volume de CO$_2$ présent dans la pièce par une fonction du temps $t$ écoulé après 20h (exprimé en minutes) qui pour formule $V(t) = V_0e^{-0.06t} + 360$
\begin{enumerate}
\item Démontrer que $V_0$ est égale à \np{3240}.
\item Quel sera, au dm$^3$ près, le volume de CO$_2$ dans cette pièce à 24 h ?
\item Démontrer que $V'(t) = - 194.4 e^{- 0.06 t}$.
\item Étudier le signe de $V'(t)$ puis en déduire le sens de variation de $V(t)$.
\item Que peut-on dire du volume de CO$_2$ quand $t$ devient grand?
\end{enumerate}
\end{enumerate}
\end{exercise}
\begin{solution}
\begin{enumerate}
\item Volume à 20h: $600000\times 0.006 = 3600$
\item
\begin{enumerate}
\item $t=0$ correspond à 20h.
Donc $V(0) = 3600 = V_0e^{-0.06\times 0} + 360 = V_0 + 360$
Donc $V_0 = 3600 - 360 = 3240$
\item Il faut calculer $V(t)$ pour $t = 4$ donc
\[
V(4) = 2908.67
\]
\item Pas de correction pour cette question.
\item Pas de correction pour cette question.
\item Pas de correction pour cette question.
\end{enumerate}
\end{enumerate}
\end{solution}
\end{document}
%%% Local Variables:
%%% mode: latex
%%% TeX-master: "master"
%%% End:

View File

@ -0,0 +1,121 @@
\documentclass[a4paper,10pt]{article}
\usepackage{myXsim}
\usepackage{tasks}
% Title Page
\title{DM1 \hfill KILINC Suleyman}
\tribe{TST sti2d}
\date{\hfillÀ render pour le jeudi 25 février}
\xsimsetup{
solution/print = false
}
\begin{document}
\maketitle
\begin{exercise}[subtitle={Complexes}]
\begin{enumerate}
\item Mettre le nombre complexe suivant sous forme algébrique $z_1 = \dfrac{2 + 7 i}{-8 + 6 i} $
\item Mettre le complexe suivante sous forme exponentielle $z_2 = \sqrt{2} + \sqrt{2} i$
\item Mettre le complexe suivante sous forme exponentielle $z_3 = 8 \sqrt{3} - 8 i$
\item Calculer le produit $z_4=z_2\times z_3$ donner le résultat sous forme exponentielle puis algébrique.
\item Calculer le quotient $z_5=\frac{z_2}{z_3}$ donner le résultat sous forme exponentielle puis algébrique.
\end{enumerate}
\end{exercise}
\begin{solution}
\begin{enumerate}
\item $z_1 = \frac{13}{50} - \frac{17 i}{25}$
\item $z_3 = 2 e^{\frac{i \pi}{4}}$
\item $z_4 = 32 e^{\frac{i \pi}{12}} = 8 \sqrt{2} + 8 \sqrt{6} + i \left(- 8 \sqrt{2} + 8 \sqrt{6}\right) = 30.9 + 8.28 i$
\item $z_5 = \frac{1}{8} e^{\frac{5 i \pi}{12}} = - \frac{\sqrt{2}}{32} + \frac{\sqrt{6}}{32} + i \left(\frac{\sqrt{2}}{32} + \frac{\sqrt{6}}{32}\right) = 0.0323 + 0.121 i$
\end{enumerate}
\end{solution}
\begin{exercise}[subtitle={Bassin}]
Le tour d'un bassin au niveau du sol présente deux axes de symétrie : laxe des abscisses et la droite déquation $x=4$. Il est obtenu par symétrie de la courbe $\mathcal{C_f}$ sur $\intFF{0}{4}$$f$ est la fonction définie par
\[
f(x) = \left(- x^{2} + 5.8 x - 4.6\right) e^{- x} + 4.6
\]
On admet que sur $\intFF{0}{4}$ la fonction $f$ est positive.
\begin{enumerate}
\item Sur un repère, tracer l'allure de la courbe $\mathcal{C}_f$, les axes de symétries puis compléter pour dessiner la forme du bassin.
\item Montrer que la fonction $f$ admet comme primitive sur $\R$ la fonction $F$ définie par
\[
F(x) = 4.6 x + \left( x^{2} - 3.8 x + 0.8\right) e^{- x}
\]
\item Calculer la quantité $\ds \int_0^4 f(x) \; dx$, vous donnerez le résultat sous forme exacte. Interpréter le résultat et reportez cette quantité sur le graphique.
\item On considère que l'échelle de votre graphique est de 1unité pour 15m. Calculer l'aire du bassin. Vous donnerez un résultat arrondi au $m^2$ près.
\end{enumerate}
\end{exercise}
\begin{solution}
\begin{enumerate}
\item
\begin{tikzpicture}[baseline=(a.north), xscale=1, yscale=0.5]
\tkzInit[xmin=0,xmax=5,xstep=1,
ymin=0,ymax=10,ystep=1]
\tkzGrid
\tkzAxeXY
\tkzFct[domain=0:10,color=red,very thick]%
{ (-x**2 + 5.8*x - 4.6)*exp(-x) + 4.6 };
\end{tikzpicture}
\item Il faut dériver $F(x)$ et vérifier que $F'(x) = f(x)$.
\item $\ds \int_0^4 f(x) \; dx = F(4) - F(0) = \frac{1.6}{e^{4}} + 17.6$
\item La quantité calculée à la question précédente se retrouve 4fois pour former le bassin. Il faut ensuite prendre en compte l'échelle, comme 1unité de longueur correspond à 15m, une unité d'air correspond à $15\times15 = 225m^2$. Ainsi l'aire du bassin est égale à
\[
(\frac{1.6}{e^{4}} + 17.6)\times 4 \times 15^2 = 15866.00000
\]
\end{enumerate}
\end{solution}
\begin{exercise}[subtitle={Bassin}]
Le clinker est un constituant du ciment qui résulte de la cuisson d'un mélange composé de calcaire et d'argile. La fabrication du clinker nécessite des fours à très haute température qui libèrent dans l'air une grande quantité de dioxyde de carbone (CO$_2$).
Dans une cimenterie, la fabrication du clinker s'effectue de 7 h 30 à 20 h, dans une pièce de volume \np{700000}~dm$^3$.
À 20 h, après une journée de travail, le taux volumique de CO$_2$ dans la pièce est de 1.0\,\%.
\begin{enumerate}
\item Justifier que le volume de CO$_2$ présent dans cette pièce à 20 h est de \np{7000}~dm$^3$ .
\item On modélise le volume de CO$_2$ présent dans la pièce par une fonction du temps $t$ écoulé après 20h (exprimé en minutes) qui pour formule $V(t) = V_0e^{-0.07t} + 520$
\begin{enumerate}
\item Démontrer que $V_0$ est égale à \np{6480}.
\item Quel sera, au dm$^3$ près, le volume de CO$_2$ dans cette pièce à 24 h ?
\item Démontrer que $V'(t) = - 453.6 e^{- 0.07 t}$.
\item Étudier le signe de $V'(t)$ puis en déduire le sens de variation de $V(t)$.
\item Que peut-on dire du volume de CO$_2$ quand $t$ devient grand?
\end{enumerate}
\end{enumerate}
\end{exercise}
\begin{solution}
\begin{enumerate}
\item Volume à 20h: $700000\times 0.01 = 7000$
\item
\begin{enumerate}
\item $t=0$ correspond à 20h.
Donc $V(0) = 7000 = V_0e^{-0.07\times 0} + 520 = V_0 + 520$
Donc $V_0 = 7000 - 520 = 6480$
\item Il faut calculer $V(t)$ pour $t = 4$ donc
\[
V(4) = 5417.48
\]
\item Pas de correction pour cette question.
\item Pas de correction pour cette question.
\item Pas de correction pour cette question.
\end{enumerate}
\end{enumerate}
\end{solution}
\end{document}
%%% Local Variables:
%%% mode: latex
%%% TeX-master: "master"
%%% End:

View File

@ -0,0 +1,121 @@
\documentclass[a4paper,10pt]{article}
\usepackage{myXsim}
\usepackage{tasks}
% Title Page
\title{DM1 \hfill M'BAREK HASNAOUI Bilal}
\tribe{TST sti2d}
\date{\hfillÀ render pour le jeudi 25 février}
\xsimsetup{
solution/print = false
}
\begin{document}
\maketitle
\begin{exercise}[subtitle={Complexes}]
\begin{enumerate}
\item Mettre le nombre complexe suivant sous forme algébrique $z_1 = \dfrac{4 + 7 i}{-10 + 7 i} $
\item Mettre le complexe suivante sous forme exponentielle $z_2 = 6 - 6 \sqrt{3} i$
\item Mettre le complexe suivante sous forme exponentielle $z_3 = 6 \sqrt{2} + 6 \sqrt{2} i$
\item Calculer le produit $z_4=z_2\times z_3$ donner le résultat sous forme exponentielle puis algébrique.
\item Calculer le quotient $z_5=\frac{z_2}{z_3}$ donner le résultat sous forme exponentielle puis algébrique.
\end{enumerate}
\end{exercise}
\begin{solution}
\begin{enumerate}
\item $z_1 = \frac{9}{149} - \frac{98 i}{149}$
\item $z_3 = 12 e^{- \frac{i \pi}{3}}$
\item $z_4 = 144 e^{- \frac{i \pi}{12}} = 36 \sqrt{2} + 36 \sqrt{6} + i \left(- 36 \sqrt{6} + 36 \sqrt{2}\right) = 139.0 - 37.3 i$
\item $z_5 = 1 e^{- \frac{7 i \pi}{12}} = - \frac{\sqrt{6}}{4} + \frac{\sqrt{2}}{4} + i \left(- \frac{\sqrt{6}}{4} - \frac{\sqrt{2}}{4}\right) = -0.259 - 0.966 i$
\end{enumerate}
\end{solution}
\begin{exercise}[subtitle={Bassin}]
Le tour d'un bassin au niveau du sol présente deux axes de symétrie : laxe des abscisses et la droite déquation $x=4$. Il est obtenu par symétrie de la courbe $\mathcal{C_f}$ sur $\intFF{0}{4}$$f$ est la fonction définie par
\[
f(x) = \left(- x^{2} + 2.2 x - 9.7\right) e^{- x} + 9.7
\]
On admet que sur $\intFF{0}{4}$ la fonction $f$ est positive.
\begin{enumerate}
\item Sur un repère, tracer l'allure de la courbe $\mathcal{C}_f$, les axes de symétries puis compléter pour dessiner la forme du bassin.
\item Montrer que la fonction $f$ admet comme primitive sur $\R$ la fonction $F$ définie par
\[
F(x) = 9.7 x + \left( x^{2} - 0.2 x + 9.5\right) e^{- x}
\]
\item Calculer la quantité $\ds \int_0^4 f(x) \; dx$, vous donnerez le résultat sous forme exacte. Interpréter le résultat et reportez cette quantité sur le graphique.
\item On considère que l'échelle de votre graphique est de 1unité pour 15m. Calculer l'aire du bassin. Vous donnerez un résultat arrondi au $m^2$ près.
\end{enumerate}
\end{exercise}
\begin{solution}
\begin{enumerate}
\item
\begin{tikzpicture}[baseline=(a.north), xscale=1, yscale=0.5]
\tkzInit[xmin=0,xmax=5,xstep=1,
ymin=0,ymax=10,ystep=1]
\tkzGrid
\tkzAxeXY
\tkzFct[domain=0:10,color=red,very thick]%
{ (-x**2 + 2.2*x - 9.7)*exp(-x) + 9.7 };
\end{tikzpicture}
\item Il faut dériver $F(x)$ et vérifier que $F'(x) = f(x)$.
\item $\ds \int_0^4 f(x) \; dx = F(4) - F(0) = \frac{24.7}{e^{4}} + 29.3$
\item La quantité calculée à la question précédente se retrouve 4fois pour former le bassin. Il faut ensuite prendre en compte l'échelle, comme 1unité de longueur correspond à 15m, une unité d'air correspond à $15\times15 = 225m^2$. Ainsi l'aire du bassin est égale à
\[
(\frac{24.7}{e^{4}} + 29.3)\times 4 \times 15^2 = 26777.00000
\]
\end{enumerate}
\end{solution}
\begin{exercise}[subtitle={Bassin}]
Le clinker est un constituant du ciment qui résulte de la cuisson d'un mélange composé de calcaire et d'argile. La fabrication du clinker nécessite des fours à très haute température qui libèrent dans l'air une grande quantité de dioxyde de carbone (CO$_2$).
Dans une cimenterie, la fabrication du clinker s'effectue de 7 h 30 à 20 h, dans une pièce de volume \np{300000}~dm$^3$.
À 20 h, après une journée de travail, le taux volumique de CO$_2$ dans la pièce est de 0.6\,\%.
\begin{enumerate}
\item Justifier que le volume de CO$_2$ présent dans cette pièce à 20 h est de \np{1800}~dm$^3$ .
\item On modélise le volume de CO$_2$ présent dans la pièce par une fonction du temps $t$ écoulé après 20h (exprimé en minutes) qui pour formule $V(t) = V_0e^{-0.1t} + 430$
\begin{enumerate}
\item Démontrer que $V_0$ est égale à \np{1370}.
\item Quel sera, au dm$^3$ près, le volume de CO$_2$ dans cette pièce à 22 h ?
\item Démontrer que $V'(t) = - 137.0 e^{- 0.1 t}$.
\item Étudier le signe de $V'(t)$ puis en déduire le sens de variation de $V(t)$.
\item Que peut-on dire du volume de CO$_2$ quand $t$ devient grand?
\end{enumerate}
\end{enumerate}
\end{exercise}
\begin{solution}
\begin{enumerate}
\item Volume à 20h: $300000\times 0.006 = 1800$
\item
\begin{enumerate}
\item $t=0$ correspond à 20h.
Donc $V(0) = 1800 = V_0e^{-0.1\times 0} + 430 = V_0 + 430$
Donc $V_0 = 1800 - 430 = 1370$
\item Il faut calculer $V(t)$ pour $t = 2$ donc
\[
V(2) = 1551.66
\]
\item Pas de correction pour cette question.
\item Pas de correction pour cette question.
\item Pas de correction pour cette question.
\end{enumerate}
\end{enumerate}
\end{solution}
\end{document}
%%% Local Variables:
%%% mode: latex
%%% TeX-master: "master"
%%% End:

View File

@ -0,0 +1,121 @@
\documentclass[a4paper,10pt]{article}
\usepackage{myXsim}
\usepackage{tasks}
% Title Page
\title{DM1 \hfill MERCIER Almandin}
\tribe{TST sti2d}
\date{\hfillÀ render pour le jeudi 25 février}
\xsimsetup{
solution/print = false
}
\begin{document}
\maketitle
\begin{exercise}[subtitle={Complexes}]
\begin{enumerate}
\item Mettre le nombre complexe suivant sous forme algébrique $z_1 = \dfrac{2 + 4 i}{-6 + 6 i} $
\item Mettre le complexe suivante sous forme exponentielle $z_2 = 4 \sqrt{3} - 4 i$
\item Mettre le complexe suivante sous forme exponentielle $z_3 = - 10 \sqrt{2} + 10 \sqrt{2} i$
\item Calculer le produit $z_4=z_2\times z_3$ donner le résultat sous forme exponentielle puis algébrique.
\item Calculer le quotient $z_5=\frac{z_2}{z_3}$ donner le résultat sous forme exponentielle puis algébrique.
\end{enumerate}
\end{exercise}
\begin{solution}
\begin{enumerate}
\item $z_1 = \frac{1}{6} - \frac{i}{2}$
\item $z_3 = 8 e^{- \frac{i \pi}{6}}$
\item $z_4 = 160 e^{\frac{7 i \pi}{12}} = - 40 \sqrt{6} + 40 \sqrt{2} + i \left(40 \sqrt{2} + 40 \sqrt{6}\right) = -41.4 + 155.0 i$
\item $z_5 = \frac{2}{5} e^{- \frac{11 i \pi}{12}} = - \frac{\sqrt{6}}{10} - \frac{\sqrt{2}}{10} + i \left(- \frac{\sqrt{6}}{10} + \frac{\sqrt{2}}{10}\right) = -0.386 - 0.104 i$
\end{enumerate}
\end{solution}
\begin{exercise}[subtitle={Bassin}]
Le tour d'un bassin au niveau du sol présente deux axes de symétrie : laxe des abscisses et la droite déquation $x=4$. Il est obtenu par symétrie de la courbe $\mathcal{C_f}$ sur $\intFF{0}{4}$$f$ est la fonction définie par
\[
f(x) = \left(- x^{2} + 1.4 x - 4.9\right) e^{- x} + 4.9
\]
On admet que sur $\intFF{0}{4}$ la fonction $f$ est positive.
\begin{enumerate}
\item Sur un repère, tracer l'allure de la courbe $\mathcal{C}_f$, les axes de symétries puis compléter pour dessiner la forme du bassin.
\item Montrer que la fonction $f$ admet comme primitive sur $\R$ la fonction $F$ définie par
\[
F(x) = 4.9 x + \left( x^{2} + 0.6 x + 5.5\right) e^{- x}
\]
\item Calculer la quantité $\ds \int_0^4 f(x) \; dx$, vous donnerez le résultat sous forme exacte. Interpréter le résultat et reportez cette quantité sur le graphique.
\item On considère que l'échelle de votre graphique est de 1unité pour 15m. Calculer l'aire du bassin. Vous donnerez un résultat arrondi au $m^2$ près.
\end{enumerate}
\end{exercise}
\begin{solution}
\begin{enumerate}
\item
\begin{tikzpicture}[baseline=(a.north), xscale=1, yscale=0.5]
\tkzInit[xmin=0,xmax=5,xstep=1,
ymin=0,ymax=10,ystep=1]
\tkzGrid
\tkzAxeXY
\tkzFct[domain=0:10,color=red,very thick]%
{ (-x**2 + 1.4*x - 4.9)*exp(-x) + 4.9 };
\end{tikzpicture}
\item Il faut dériver $F(x)$ et vérifier que $F'(x) = f(x)$.
\item $\ds \int_0^4 f(x) \; dx = F(4) - F(0) = \frac{23.9}{e^{4}} + 14.1$
\item La quantité calculée à la question précédente se retrouve 4fois pour former le bassin. Il faut ensuite prendre en compte l'échelle, comme 1unité de longueur correspond à 15m, une unité d'air correspond à $15\times15 = 225m^2$. Ainsi l'aire du bassin est égale à
\[
(\frac{23.9}{e^{4}} + 14.1)\times 4 \times 15^2 = 13084.00000
\]
\end{enumerate}
\end{solution}
\begin{exercise}[subtitle={Bassin}]
Le clinker est un constituant du ciment qui résulte de la cuisson d'un mélange composé de calcaire et d'argile. La fabrication du clinker nécessite des fours à très haute température qui libèrent dans l'air une grande quantité de dioxyde de carbone (CO$_2$).
Dans une cimenterie, la fabrication du clinker s'effectue de 7 h 30 à 20 h, dans une pièce de volume \np{400000}~dm$^3$.
À 20 h, après une journée de travail, le taux volumique de CO$_2$ dans la pièce est de 0.7\,\%.
\begin{enumerate}
\item Justifier que le volume de CO$_2$ présent dans cette pièce à 20 h est de \np{2800}~dm$^3$ .
\item On modélise le volume de CO$_2$ présent dans la pièce par une fonction du temps $t$ écoulé après 20h (exprimé en minutes) qui pour formule $V(t) = V_0e^{-0.07t} + 360$
\begin{enumerate}
\item Démontrer que $V_0$ est égale à \np{2440}.
\item Quel sera, au dm$^3$ près, le volume de CO$_2$ dans cette pièce à 23 h ?
\item Démontrer que $V'(t) = - 170.8 e^{- 0.07 t}$.
\item Étudier le signe de $V'(t)$ puis en déduire le sens de variation de $V(t)$.
\item Que peut-on dire du volume de CO$_2$ quand $t$ devient grand?
\end{enumerate}
\end{enumerate}
\end{exercise}
\begin{solution}
\begin{enumerate}
\item Volume à 20h: $400000\times 0.006999999999999999 = 2800$
\item
\begin{enumerate}
\item $t=0$ correspond à 20h.
Donc $V(0) = 2800 = V_0e^{-0.07\times 0} + 360 = V_0 + 360$
Donc $V_0 = 2800 - 360 = 2440$
\item Il faut calculer $V(t)$ pour $t = 3$ donc
\[
V(3) = 2337.83
\]
\item Pas de correction pour cette question.
\item Pas de correction pour cette question.
\item Pas de correction pour cette question.
\end{enumerate}
\end{enumerate}
\end{solution}
\end{document}
%%% Local Variables:
%%% mode: latex
%%% TeX-master: "master"
%%% End:

View File

@ -0,0 +1,121 @@
\documentclass[a4paper,10pt]{article}
\usepackage{myXsim}
\usepackage{tasks}
% Title Page
\title{DM1 \hfill MOUFAQ Amine}
\tribe{TST sti2d}
\date{\hfillÀ render pour le jeudi 25 février}
\xsimsetup{
solution/print = false
}
\begin{document}
\maketitle
\begin{exercise}[subtitle={Complexes}]
\begin{enumerate}
\item Mettre le nombre complexe suivant sous forme algébrique $z_1 = \dfrac{6 + 9 i}{-9 + 10 i} $
\item Mettre le complexe suivante sous forme exponentielle $z_2 = 10 \sqrt{3} + 10 i$
\item Mettre le complexe suivante sous forme exponentielle $z_3 = 9 \sqrt{2} + 9 \sqrt{2} i$
\item Calculer le produit $z_4=z_2\times z_3$ donner le résultat sous forme exponentielle puis algébrique.
\item Calculer le quotient $z_5=\frac{z_2}{z_3}$ donner le résultat sous forme exponentielle puis algébrique.
\end{enumerate}
\end{exercise}
\begin{solution}
\begin{enumerate}
\item $z_1 = \frac{36}{181} - \frac{141 i}{181}$
\item $z_3 = 20 e^{\frac{i \pi}{6}}$
\item $z_4 = 360 e^{\frac{5 i \pi}{12}} = - 90 \sqrt{2} + 90 \sqrt{6} + i \left(90 \sqrt{2} + 90 \sqrt{6}\right) = 93.2 + 348.0 i$
\item $z_5 = \frac{10}{9} e^{- \frac{i \pi}{12}} = \frac{5 \sqrt{2}}{18} + \frac{5 \sqrt{6}}{18} + i \left(- \frac{5 \sqrt{6}}{18} + \frac{5 \sqrt{2}}{18}\right) = 1.07 - 0.288 i$
\end{enumerate}
\end{solution}
\begin{exercise}[subtitle={Bassin}]
Le tour d'un bassin au niveau du sol présente deux axes de symétrie : laxe des abscisses et la droite déquation $x=4$. Il est obtenu par symétrie de la courbe $\mathcal{C_f}$ sur $\intFF{0}{4}$$f$ est la fonction définie par
\[
f(x) = \left(- x^{2} + 5.5 x - 9.8\right) e^{- x} + 9.8
\]
On admet que sur $\intFF{0}{4}$ la fonction $f$ est positive.
\begin{enumerate}
\item Sur un repère, tracer l'allure de la courbe $\mathcal{C}_f$, les axes de symétries puis compléter pour dessiner la forme du bassin.
\item Montrer que la fonction $f$ admet comme primitive sur $\R$ la fonction $F$ définie par
\[
F(x) = 9.8 x + \left( x^{2} - 3.5 x + 6.3\right) e^{- x}
\]
\item Calculer la quantité $\ds \int_0^4 f(x) \; dx$, vous donnerez le résultat sous forme exacte. Interpréter le résultat et reportez cette quantité sur le graphique.
\item On considère que l'échelle de votre graphique est de 1unité pour 15m. Calculer l'aire du bassin. Vous donnerez un résultat arrondi au $m^2$ près.
\end{enumerate}
\end{exercise}
\begin{solution}
\begin{enumerate}
\item
\begin{tikzpicture}[baseline=(a.north), xscale=1, yscale=0.5]
\tkzInit[xmin=0,xmax=5,xstep=1,
ymin=0,ymax=10,ystep=1]
\tkzGrid
\tkzAxeXY
\tkzFct[domain=0:10,color=red,very thick]%
{ (-x**2 + 5.5*x - 9.8)*exp(-x) + 9.8 };
\end{tikzpicture}
\item Il faut dériver $F(x)$ et vérifier que $F'(x) = f(x)$.
\item $\ds \int_0^4 f(x) \; dx = F(4) - F(0) = \frac{8.3}{e^{4}} + 32.9$
\item La quantité calculée à la question précédente se retrouve 4fois pour former le bassin. Il faut ensuite prendre en compte l'échelle, comme 1unité de longueur correspond à 15m, une unité d'air correspond à $15\times15 = 225m^2$. Ainsi l'aire du bassin est égale à
\[
(\frac{8.3}{e^{4}} + 32.9)\times 4 \times 15^2 = 29747.00000
\]
\end{enumerate}
\end{solution}
\begin{exercise}[subtitle={Bassin}]
Le clinker est un constituant du ciment qui résulte de la cuisson d'un mélange composé de calcaire et d'argile. La fabrication du clinker nécessite des fours à très haute température qui libèrent dans l'air une grande quantité de dioxyde de carbone (CO$_2$).
Dans une cimenterie, la fabrication du clinker s'effectue de 7 h 30 à 20 h, dans une pièce de volume \np{600000}~dm$^3$.
À 20 h, après une journée de travail, le taux volumique de CO$_2$ dans la pièce est de 0.9\,\%.
\begin{enumerate}
\item Justifier que le volume de CO$_2$ présent dans cette pièce à 20 h est de \np{5400}~dm$^3$ .
\item On modélise le volume de CO$_2$ présent dans la pièce par une fonction du temps $t$ écoulé après 20h (exprimé en minutes) qui pour formule $V(t) = V_0e^{-0.06t} + 340$
\begin{enumerate}
\item Démontrer que $V_0$ est égale à \np{5060}.
\item Quel sera, au dm$^3$ près, le volume de CO$_2$ dans cette pièce à 24 h ?
\item Démontrer que $V'(t) = - 303.6 e^{- 0.06 t}$.
\item Étudier le signe de $V'(t)$ puis en déduire le sens de variation de $V(t)$.
\item Que peut-on dire du volume de CO$_2$ quand $t$ devient grand?
\end{enumerate}
\end{enumerate}
\end{exercise}
\begin{solution}
\begin{enumerate}
\item Volume à 20h: $600000\times 0.009000000000000001 = 5400$
\item
\begin{enumerate}
\item $t=0$ correspond à 20h.
Donc $V(0) = 5400 = V_0e^{-0.06\times 0} + 340 = V_0 + 340$
Donc $V_0 = 5400 - 340 = 5060$
\item Il faut calculer $V(t)$ pour $t = 4$ donc
\[
V(4) = 4320.34
\]
\item Pas de correction pour cette question.
\item Pas de correction pour cette question.
\item Pas de correction pour cette question.
\end{enumerate}
\end{enumerate}
\end{solution}
\end{document}
%%% Local Variables:
%%% mode: latex
%%% TeX-master: "master"
%%% End:

View File

@ -0,0 +1,121 @@
\documentclass[a4paper,10pt]{article}
\usepackage{myXsim}
\usepackage{tasks}
% Title Page
\title{DM1 \hfill NARDINI Kakary}
\tribe{TST sti2d}
\date{\hfillÀ render pour le jeudi 25 février}
\xsimsetup{
solution/print = false
}
\begin{document}
\maketitle
\begin{exercise}[subtitle={Complexes}]
\begin{enumerate}
\item Mettre le nombre complexe suivant sous forme algébrique $z_1 = \dfrac{7 + 5 i}{-4 + 5 i} $
\item Mettre le complexe suivante sous forme exponentielle $z_2 = - 8 \sqrt{2} + 8 \sqrt{2} i$
\item Mettre le complexe suivante sous forme exponentielle $z_3 = \sqrt{3} + i$
\item Calculer le produit $z_4=z_2\times z_3$ donner le résultat sous forme exponentielle puis algébrique.
\item Calculer le quotient $z_5=\frac{z_2}{z_3}$ donner le résultat sous forme exponentielle puis algébrique.
\end{enumerate}
\end{exercise}
\begin{solution}
\begin{enumerate}
\item $z_1 = - \frac{3}{41} - \frac{55 i}{41}$
\item $z_3 = 16 e^{\frac{3 i \pi}{4}}$
\item $z_4 = 32 e^{\frac{11 i \pi}{12}} = - 8 \sqrt{6} - 8 \sqrt{2} + i \left(- 8 \sqrt{2} + 8 \sqrt{6}\right) = -30.9 + 8.28 i$
\item $z_5 = 8 e^{\frac{7 i \pi}{12}} = - 2 \sqrt{6} + 2 \sqrt{2} + i \left(2 \sqrt{2} + 2 \sqrt{6}\right) = -2.07 + 7.73 i$
\end{enumerate}
\end{solution}
\begin{exercise}[subtitle={Bassin}]
Le tour d'un bassin au niveau du sol présente deux axes de symétrie : laxe des abscisses et la droite déquation $x=4$. Il est obtenu par symétrie de la courbe $\mathcal{C_f}$ sur $\intFF{0}{4}$$f$ est la fonction définie par
\[
f(x) = \left(- x^{2} + 2.3 x - 1.7\right) e^{- x} + 1.7
\]
On admet que sur $\intFF{0}{4}$ la fonction $f$ est positive.
\begin{enumerate}
\item Sur un repère, tracer l'allure de la courbe $\mathcal{C}_f$, les axes de symétries puis compléter pour dessiner la forme du bassin.
\item Montrer que la fonction $f$ admet comme primitive sur $\R$ la fonction $F$ définie par
\[
F(x) = 1.7 x + \left( x^{2} - 0.3 x + 1.4\right) e^{- x}
\]
\item Calculer la quantité $\ds \int_0^4 f(x) \; dx$, vous donnerez le résultat sous forme exacte. Interpréter le résultat et reportez cette quantité sur le graphique.
\item On considère que l'échelle de votre graphique est de 1unité pour 15m. Calculer l'aire du bassin. Vous donnerez un résultat arrondi au $m^2$ près.
\end{enumerate}
\end{exercise}
\begin{solution}
\begin{enumerate}
\item
\begin{tikzpicture}[baseline=(a.north), xscale=1, yscale=0.5]
\tkzInit[xmin=0,xmax=5,xstep=1,
ymin=0,ymax=10,ystep=1]
\tkzGrid
\tkzAxeXY
\tkzFct[domain=0:10,color=red,very thick]%
{ (-x**2 + 2.3*x - 1.7)*exp(-x) + 1.7 };
\end{tikzpicture}
\item Il faut dériver $F(x)$ et vérifier que $F'(x) = f(x)$.
\item $\ds \int_0^4 f(x) \; dx = F(4) - F(0) = \frac{16.2}{e^{4}} + 5.4$
\item La quantité calculée à la question précédente se retrouve 4fois pour former le bassin. Il faut ensuite prendre en compte l'échelle, comme 1unité de longueur correspond à 15m, une unité d'air correspond à $15\times15 = 225m^2$. Ainsi l'aire du bassin est égale à
\[
(\frac{16.2}{e^{4}} + 5.4)\times 4 \times 15^2 = 5127.000000
\]
\end{enumerate}
\end{solution}
\begin{exercise}[subtitle={Bassin}]
Le clinker est un constituant du ciment qui résulte de la cuisson d'un mélange composé de calcaire et d'argile. La fabrication du clinker nécessite des fours à très haute température qui libèrent dans l'air une grande quantité de dioxyde de carbone (CO$_2$).
Dans une cimenterie, la fabrication du clinker s'effectue de 7 h 30 à 20 h, dans une pièce de volume \np{600000}~dm$^3$.
À 20 h, après une journée de travail, le taux volumique de CO$_2$ dans la pièce est de 0.9\,\%.
\begin{enumerate}
\item Justifier que le volume de CO$_2$ présent dans cette pièce à 20 h est de \np{5400}~dm$^3$ .
\item On modélise le volume de CO$_2$ présent dans la pièce par une fonction du temps $t$ écoulé après 20h (exprimé en minutes) qui pour formule $V(t) = V_0e^{-0.09t} + 360$
\begin{enumerate}
\item Démontrer que $V_0$ est égale à \np{5040}.
\item Quel sera, au dm$^3$ près, le volume de CO$_2$ dans cette pièce à 22 h ?
\item Démontrer que $V'(t) = - 453.6 e^{- 0.09 t}$.
\item Étudier le signe de $V'(t)$ puis en déduire le sens de variation de $V(t)$.
\item Que peut-on dire du volume de CO$_2$ quand $t$ devient grand?
\end{enumerate}
\end{enumerate}
\end{exercise}
\begin{solution}
\begin{enumerate}
\item Volume à 20h: $600000\times 0.009000000000000001 = 5400$
\item
\begin{enumerate}
\item $t=0$ correspond à 20h.
Donc $V(0) = 5400 = V_0e^{-0.09\times 0} + 360 = V_0 + 360$
Donc $V_0 = 5400 - 360 = 5040$
\item Il faut calculer $V(t)$ pour $t = 2$ donc
\[
V(2) = 4569.76
\]
\item Pas de correction pour cette question.
\item Pas de correction pour cette question.
\item Pas de correction pour cette question.
\end{enumerate}
\end{enumerate}
\end{solution}
\end{document}
%%% Local Variables:
%%% mode: latex
%%% TeX-master: "master"
%%% End:

View File

@ -0,0 +1,121 @@
\documentclass[a4paper,10pt]{article}
\usepackage{myXsim}
\usepackage{tasks}
% Title Page
\title{DM1 \hfill ONAL Yakub}
\tribe{TST sti2d}
\date{\hfillÀ render pour le jeudi 25 février}
\xsimsetup{
solution/print = false
}
\begin{document}
\maketitle
\begin{exercise}[subtitle={Complexes}]
\begin{enumerate}
\item Mettre le nombre complexe suivant sous forme algébrique $z_1 = \dfrac{8 + 7 i}{-5 + 6 i} $
\item Mettre le complexe suivante sous forme exponentielle $z_2 = 7 + 7 \sqrt{3} i$
\item Mettre le complexe suivante sous forme exponentielle $z_3 = \sqrt{3} + i$
\item Calculer le produit $z_4=z_2\times z_3$ donner le résultat sous forme exponentielle puis algébrique.
\item Calculer le quotient $z_5=\frac{z_2}{z_3}$ donner le résultat sous forme exponentielle puis algébrique.
\end{enumerate}
\end{exercise}
\begin{solution}
\begin{enumerate}
\item $z_1 = \frac{2}{61} - \frac{83 i}{61}$
\item $z_3 = 14 e^{\frac{i \pi}{3}}$
\item $z_4 = 28 e^{\frac{i \pi}{2}} = 28 i = 28.0 i$
\item $z_5 = 7 e^{\frac{i \pi}{6}} = \frac{7 \sqrt{3}}{2} + \frac{7 i}{2} = 6.06 + 3.5 i$
\end{enumerate}
\end{solution}
\begin{exercise}[subtitle={Bassin}]
Le tour d'un bassin au niveau du sol présente deux axes de symétrie : laxe des abscisses et la droite déquation $x=4$. Il est obtenu par symétrie de la courbe $\mathcal{C_f}$ sur $\intFF{0}{4}$$f$ est la fonction définie par
\[
f(x) = \left(- x^{2} + 3.6 x - 8.6\right) e^{- x} + 8.6
\]
On admet que sur $\intFF{0}{4}$ la fonction $f$ est positive.
\begin{enumerate}
\item Sur un repère, tracer l'allure de la courbe $\mathcal{C}_f$, les axes de symétries puis compléter pour dessiner la forme du bassin.
\item Montrer que la fonction $f$ admet comme primitive sur $\R$ la fonction $F$ définie par
\[
F(x) = 8.6 x + \left( x^{2} - 1.6 x + 7.0\right) e^{- x}
\]
\item Calculer la quantité $\ds \int_0^4 f(x) \; dx$, vous donnerez le résultat sous forme exacte. Interpréter le résultat et reportez cette quantité sur le graphique.
\item On considère que l'échelle de votre graphique est de 1unité pour 15m. Calculer l'aire du bassin. Vous donnerez un résultat arrondi au $m^2$ près.
\end{enumerate}
\end{exercise}
\begin{solution}
\begin{enumerate}
\item
\begin{tikzpicture}[baseline=(a.north), xscale=1, yscale=0.5]
\tkzInit[xmin=0,xmax=5,xstep=1,
ymin=0,ymax=10,ystep=1]
\tkzGrid
\tkzAxeXY
\tkzFct[domain=0:10,color=red,very thick]%
{ (-x**2 + 3.6*x - 8.6)*exp(-x) + 8.6 };
\end{tikzpicture}
\item Il faut dériver $F(x)$ et vérifier que $F'(x) = f(x)$.
\item $\ds \int_0^4 f(x) \; dx = F(4) - F(0) = \frac{16.6}{e^{4}} + 27.4$
\item La quantité calculée à la question précédente se retrouve 4fois pour former le bassin. Il faut ensuite prendre en compte l'échelle, comme 1unité de longueur correspond à 15m, une unité d'air correspond à $15\times15 = 225m^2$. Ainsi l'aire du bassin est égale à
\[
(\frac{16.6}{e^{4}} + 27.4)\times 4 \times 15^2 = 24934.00000
\]
\end{enumerate}
\end{solution}
\begin{exercise}[subtitle={Bassin}]
Le clinker est un constituant du ciment qui résulte de la cuisson d'un mélange composé de calcaire et d'argile. La fabrication du clinker nécessite des fours à très haute température qui libèrent dans l'air une grande quantité de dioxyde de carbone (CO$_2$).
Dans une cimenterie, la fabrication du clinker s'effectue de 7 h 30 à 20 h, dans une pièce de volume \np{300000}~dm$^3$.
À 20 h, après une journée de travail, le taux volumique de CO$_2$ dans la pièce est de 0.6\,\%.
\begin{enumerate}
\item Justifier que le volume de CO$_2$ présent dans cette pièce à 20 h est de \np{1800}~dm$^3$ .
\item On modélise le volume de CO$_2$ présent dans la pièce par une fonction du temps $t$ écoulé après 20h (exprimé en minutes) qui pour formule $V(t) = V_0e^{-0.08t} + 450$
\begin{enumerate}
\item Démontrer que $V_0$ est égale à \np{1350}.
\item Quel sera, au dm$^3$ près, le volume de CO$_2$ dans cette pièce à 22 h ?
\item Démontrer que $V'(t) = - 108.0 e^{- 0.08 t}$.
\item Étudier le signe de $V'(t)$ puis en déduire le sens de variation de $V(t)$.
\item Que peut-on dire du volume de CO$_2$ quand $t$ devient grand?
\end{enumerate}
\end{enumerate}
\end{exercise}
\begin{solution}
\begin{enumerate}
\item Volume à 20h: $300000\times 0.006 = 1800$
\item
\begin{enumerate}
\item $t=0$ correspond à 20h.
Donc $V(0) = 1800 = V_0e^{-0.08\times 0} + 450 = V_0 + 450$
Donc $V_0 = 1800 - 450 = 1350$
\item Il faut calculer $V(t)$ pour $t = 2$ donc
\[
V(2) = 1600.39
\]
\item Pas de correction pour cette question.
\item Pas de correction pour cette question.
\item Pas de correction pour cette question.
\end{enumerate}
\end{enumerate}
\end{solution}
\end{document}
%%% Local Variables:
%%% mode: latex
%%% TeX-master: "master"
%%% End:

View File

@ -0,0 +1,121 @@
\documentclass[a4paper,10pt]{article}
\usepackage{myXsim}
\usepackage{tasks}
% Title Page
\title{DM1 \hfill RADOUAA Saleh}
\tribe{TST sti2d}
\date{\hfillÀ render pour le jeudi 25 février}
\xsimsetup{
solution/print = false
}
\begin{document}
\maketitle
\begin{exercise}[subtitle={Complexes}]
\begin{enumerate}
\item Mettre le nombre complexe suivant sous forme algébrique $z_1 = \dfrac{4 + 2 i}{-8 + 10 i} $
\item Mettre le complexe suivante sous forme exponentielle $z_2 = 10 - 10 \sqrt{3} i$
\item Mettre le complexe suivante sous forme exponentielle $z_3 = 9 \sqrt{3} + 9 i$
\item Calculer le produit $z_4=z_2\times z_3$ donner le résultat sous forme exponentielle puis algébrique.
\item Calculer le quotient $z_5=\frac{z_2}{z_3}$ donner le résultat sous forme exponentielle puis algébrique.
\end{enumerate}
\end{exercise}
\begin{solution}
\begin{enumerate}
\item $z_1 = - \frac{3}{41} - \frac{14 i}{41}$
\item $z_3 = 20 e^{- \frac{i \pi}{3}}$
\item $z_4 = 360 e^{- \frac{i \pi}{6}} = 180 \sqrt{3} - 180 i = 312.0 - 180.0 i$
\item $z_5 = \frac{10}{9} e^{- \frac{i \pi}{2}} = - \frac{10 i}{9} = - 1.11 i$
\end{enumerate}
\end{solution}
\begin{exercise}[subtitle={Bassin}]
Le tour d'un bassin au niveau du sol présente deux axes de symétrie : laxe des abscisses et la droite déquation $x=4$. Il est obtenu par symétrie de la courbe $\mathcal{C_f}$ sur $\intFF{0}{4}$$f$ est la fonction définie par
\[
f(x) = \left(- x^{2} + 0.6 x - 2.3\right) e^{- x} + 2.3
\]
On admet que sur $\intFF{0}{4}$ la fonction $f$ est positive.
\begin{enumerate}
\item Sur un repère, tracer l'allure de la courbe $\mathcal{C}_f$, les axes de symétries puis compléter pour dessiner la forme du bassin.
\item Montrer que la fonction $f$ admet comme primitive sur $\R$ la fonction $F$ définie par
\[
F(x) = 2.3 x + \left( x^{2} + 1.4 x + 3.7\right) e^{- x}
\]
\item Calculer la quantité $\ds \int_0^4 f(x) \; dx$, vous donnerez le résultat sous forme exacte. Interpréter le résultat et reportez cette quantité sur le graphique.
\item On considère que l'échelle de votre graphique est de 1unité pour 15m. Calculer l'aire du bassin. Vous donnerez un résultat arrondi au $m^2$ près.
\end{enumerate}
\end{exercise}
\begin{solution}
\begin{enumerate}
\item
\begin{tikzpicture}[baseline=(a.north), xscale=1, yscale=0.5]
\tkzInit[xmin=0,xmax=5,xstep=1,
ymin=0,ymax=10,ystep=1]
\tkzGrid
\tkzAxeXY
\tkzFct[domain=0:10,color=red,very thick]%
{ (-x**2 + 0.6*x - 2.3)*exp(-x) + 2.3 };
\end{tikzpicture}
\item Il faut dériver $F(x)$ et vérifier que $F'(x) = f(x)$.
\item $\ds \int_0^4 f(x) \; dx = F(4) - F(0) = \frac{25.3}{e^{4}} + 5.5$
\item La quantité calculée à la question précédente se retrouve 4fois pour former le bassin. Il faut ensuite prendre en compte l'échelle, comme 1unité de longueur correspond à 15m, une unité d'air correspond à $15\times15 = 225m^2$. Ainsi l'aire du bassin est égale à
\[
(\frac{25.3}{e^{4}} + 5.5)\times 4 \times 15^2 = 5367.000000
\]
\end{enumerate}
\end{solution}
\begin{exercise}[subtitle={Bassin}]
Le clinker est un constituant du ciment qui résulte de la cuisson d'un mélange composé de calcaire et d'argile. La fabrication du clinker nécessite des fours à très haute température qui libèrent dans l'air une grande quantité de dioxyde de carbone (CO$_2$).
Dans une cimenterie, la fabrication du clinker s'effectue de 7 h 30 à 20 h, dans une pièce de volume \np{500000}~dm$^3$.
À 20 h, après une journée de travail, le taux volumique de CO$_2$ dans la pièce est de 0.9\,\%.
\begin{enumerate}
\item Justifier que le volume de CO$_2$ présent dans cette pièce à 20 h est de \np{4500}~dm$^3$ .
\item On modélise le volume de CO$_2$ présent dans la pièce par une fonction du temps $t$ écoulé après 20h (exprimé en minutes) qui pour formule $V(t) = V_0e^{-0.06t} + 440$
\begin{enumerate}
\item Démontrer que $V_0$ est égale à \np{4060}.
\item Quel sera, au dm$^3$ près, le volume de CO$_2$ dans cette pièce à 24 h ?
\item Démontrer que $V'(t) = - 243.6 e^{- 0.06 t}$.
\item Étudier le signe de $V'(t)$ puis en déduire le sens de variation de $V(t)$.
\item Que peut-on dire du volume de CO$_2$ quand $t$ devient grand?
\end{enumerate}
\end{enumerate}
\end{exercise}
\begin{solution}
\begin{enumerate}
\item Volume à 20h: $500000\times 0.009000000000000001 = 4500$
\item
\begin{enumerate}
\item $t=0$ correspond à 20h.
Donc $V(0) = 4500 = V_0e^{-0.06\times 0} + 440 = V_0 + 440$
Donc $V_0 = 4500 - 440 = 4060$
\item Il faut calculer $V(t)$ pour $t = 4$ donc
\[
V(4) = 3633.71
\]
\item Pas de correction pour cette question.
\item Pas de correction pour cette question.
\item Pas de correction pour cette question.
\end{enumerate}
\end{enumerate}
\end{solution}
\end{document}
%%% Local Variables:
%%% mode: latex
%%% TeX-master: "master"
%%% End:

View File

@ -0,0 +1,121 @@
\documentclass[a4paper,10pt]{article}
\usepackage{myXsim}
\usepackage{tasks}
% Title Page
\title{DM1 \hfill TAVERNIER Joanny}
\tribe{TST sti2d}
\date{\hfillÀ render pour le jeudi 25 février}
\xsimsetup{
solution/print = false
}
\begin{document}
\maketitle
\begin{exercise}[subtitle={Complexes}]
\begin{enumerate}
\item Mettre le nombre complexe suivant sous forme algébrique $z_1 = \dfrac{10 + 6 i}{-7 + 3 i} $
\item Mettre le complexe suivante sous forme exponentielle $z_2 = 6 - 6 \sqrt{3} i$
\item Mettre le complexe suivante sous forme exponentielle $z_3 = 5 \sqrt{3} - 5 i$
\item Calculer le produit $z_4=z_2\times z_3$ donner le résultat sous forme exponentielle puis algébrique.
\item Calculer le quotient $z_5=\frac{z_2}{z_3}$ donner le résultat sous forme exponentielle puis algébrique.
\end{enumerate}
\end{exercise}
\begin{solution}
\begin{enumerate}
\item $z_1 = - \frac{26}{29} - \frac{36 i}{29}$
\item $z_3 = 12 e^{- \frac{i \pi}{3}}$
\item $z_4 = 120 e^{- \frac{i \pi}{2}} = - 120 i = - 120.0 i$
\item $z_5 = \frac{6}{5} e^{- \frac{i \pi}{6}} = \frac{3 \sqrt{3}}{5} - \frac{3 i}{5} = 1.04 - 0.6 i$
\end{enumerate}
\end{solution}
\begin{exercise}[subtitle={Bassin}]
Le tour d'un bassin au niveau du sol présente deux axes de symétrie : laxe des abscisses et la droite déquation $x=4$. Il est obtenu par symétrie de la courbe $\mathcal{C_f}$ sur $\intFF{0}{4}$$f$ est la fonction définie par
\[
f(x) = \left(- x^{2} + 8.2 x - 3.6\right) e^{- x} + 3.6
\]
On admet que sur $\intFF{0}{4}$ la fonction $f$ est positive.
\begin{enumerate}
\item Sur un repère, tracer l'allure de la courbe $\mathcal{C}_f$, les axes de symétries puis compléter pour dessiner la forme du bassin.
\item Montrer que la fonction $f$ admet comme primitive sur $\R$ la fonction $F$ définie par
\[
F(x) = 3.6 x + \left( x^{2} - 6.2 x - 2.6\right) e^{- x}
\]
\item Calculer la quantité $\ds \int_0^4 f(x) \; dx$, vous donnerez le résultat sous forme exacte. Interpréter le résultat et reportez cette quantité sur le graphique.
\item On considère que l'échelle de votre graphique est de 1unité pour 15m. Calculer l'aire du bassin. Vous donnerez un résultat arrondi au $m^2$ près.
\end{enumerate}
\end{exercise}
\begin{solution}
\begin{enumerate}
\item
\begin{tikzpicture}[baseline=(a.north), xscale=1, yscale=0.5]
\tkzInit[xmin=0,xmax=5,xstep=1,
ymin=0,ymax=10,ystep=1]
\tkzGrid
\tkzAxeXY
\tkzFct[domain=0:10,color=red,very thick]%
{ (-x**2 + 8.2*x - 3.6)*exp(-x) + 3.6 };
\end{tikzpicture}
\item Il faut dériver $F(x)$ et vérifier que $F'(x) = f(x)$.
\item $\ds \int_0^4 f(x) \; dx = F(4) - F(0) = 17.0 - \frac{11.4}{e^{4}}$
\item La quantité calculée à la question précédente se retrouve 4fois pour former le bassin. Il faut ensuite prendre en compte l'échelle, comme 1unité de longueur correspond à 15m, une unité d'air correspond à $15\times15 = 225m^2$. Ainsi l'aire du bassin est égale à
\[
(17.0 - \frac{11.4}{e^{4}})\times 4 \times 15^2 = 15112.00000
\]
\end{enumerate}
\end{solution}
\begin{exercise}[subtitle={Bassin}]
Le clinker est un constituant du ciment qui résulte de la cuisson d'un mélange composé de calcaire et d'argile. La fabrication du clinker nécessite des fours à très haute température qui libèrent dans l'air une grande quantité de dioxyde de carbone (CO$_2$).
Dans une cimenterie, la fabrication du clinker s'effectue de 7 h 30 à 20 h, dans une pièce de volume \np{100000}~dm$^3$.
À 20 h, après une journée de travail, le taux volumique de CO$_2$ dans la pièce est de 0.8\,\%.
\begin{enumerate}
\item Justifier que le volume de CO$_2$ présent dans cette pièce à 20 h est de \np{800}~dm$^3$ .
\item On modélise le volume de CO$_2$ présent dans la pièce par une fonction du temps $t$ écoulé après 20h (exprimé en minutes) qui pour formule $V(t) = V_0e^{-0.01t} + 560$
\begin{enumerate}
\item Démontrer que $V_0$ est égale à \np{240}.
\item Quel sera, au dm$^3$ près, le volume de CO$_2$ dans cette pièce à 22 h ?
\item Démontrer que $V'(t) = - 2.4 e^{- 0.01 t}$.
\item Étudier le signe de $V'(t)$ puis en déduire le sens de variation de $V(t)$.
\item Que peut-on dire du volume de CO$_2$ quand $t$ devient grand?
\end{enumerate}
\end{enumerate}
\end{exercise}
\begin{solution}
\begin{enumerate}
\item Volume à 20h: $100000\times 0.008 = 800$
\item
\begin{enumerate}
\item $t=0$ correspond à 20h.
Donc $V(0) = 800 = V_0e^{-0.01\times 0} + 560 = V_0 + 560$
Donc $V_0 = 800 - 560 = 240$
\item Il faut calculer $V(t)$ pour $t = 2$ donc
\[
V(2) = 795.25
\]
\item Pas de correction pour cette question.
\item Pas de correction pour cette question.
\item Pas de correction pour cette question.
\end{enumerate}
\end{enumerate}
\end{solution}
\end{document}
%%% Local Variables:
%%% mode: latex
%%% TeX-master: "master"
%%% End:

View File

@ -0,0 +1,121 @@
\documentclass[a4paper,10pt]{article}
\usepackage{myXsim}
\usepackage{tasks}
% Title Page
\title{DM1 \hfill ZAHORE Zahiri}
\tribe{TST sti2d}
\date{\hfillÀ render pour le jeudi 25 février}
\xsimsetup{
solution/print = false
}
\begin{document}
\maketitle
\begin{exercise}[subtitle={Complexes}]
\begin{enumerate}
\item Mettre le nombre complexe suivant sous forme algébrique $z_1 = \dfrac{2 + 4 i}{-4 + 9 i} $
\item Mettre le complexe suivante sous forme exponentielle $z_2 = - 2 \sqrt{3} - 2 i$
\item Mettre le complexe suivante sous forme exponentielle $z_3 = 6 + 6 \sqrt{3} i$
\item Calculer le produit $z_4=z_2\times z_3$ donner le résultat sous forme exponentielle puis algébrique.
\item Calculer le quotient $z_5=\frac{z_2}{z_3}$ donner le résultat sous forme exponentielle puis algébrique.
\end{enumerate}
\end{exercise}
\begin{solution}
\begin{enumerate}
\item $z_1 = \frac{28}{97} - \frac{34 i}{97}$
\item $z_3 = 4 e^{- \frac{5 i \pi}{6}}$
\item $z_4 = 48 e^{- \frac{i \pi}{2}} = - 48 i = - 48.0 i$
\item $z_5 = \frac{1}{3} e^{- \frac{7 i \pi}{6}} = - \frac{\sqrt{3}}{6} + \frac{i}{6} = -0.289 + 0.167 i$
\end{enumerate}
\end{solution}
\begin{exercise}[subtitle={Bassin}]
Le tour d'un bassin au niveau du sol présente deux axes de symétrie : laxe des abscisses et la droite déquation $x=4$. Il est obtenu par symétrie de la courbe $\mathcal{C_f}$ sur $\intFF{0}{4}$$f$ est la fonction définie par
\[
f(x) = \left(- x^{2} + 4.8 x - 3.0\right) e^{- x} + 3.0
\]
On admet que sur $\intFF{0}{4}$ la fonction $f$ est positive.
\begin{enumerate}
\item Sur un repère, tracer l'allure de la courbe $\mathcal{C}_f$, les axes de symétries puis compléter pour dessiner la forme du bassin.
\item Montrer que la fonction $f$ admet comme primitive sur $\R$ la fonction $F$ définie par
\[
F(x) = 3.0 x + \left( x^{2} - 2.8 x + 0.2\right) e^{- x}
\]
\item Calculer la quantité $\ds \int_0^4 f(x) \; dx$, vous donnerez le résultat sous forme exacte. Interpréter le résultat et reportez cette quantité sur le graphique.
\item On considère que l'échelle de votre graphique est de 1unité pour 15m. Calculer l'aire du bassin. Vous donnerez un résultat arrondi au $m^2$ près.
\end{enumerate}
\end{exercise}
\begin{solution}
\begin{enumerate}
\item
\begin{tikzpicture}[baseline=(a.north), xscale=1, yscale=0.5]
\tkzInit[xmin=0,xmax=5,xstep=1,
ymin=0,ymax=10,ystep=1]
\tkzGrid
\tkzAxeXY
\tkzFct[domain=0:10,color=red,very thick]%
{ (-x**2 + 4.8*x - 3.0)*exp(-x) + 3.0 };
\end{tikzpicture}
\item Il faut dériver $F(x)$ et vérifier que $F'(x) = f(x)$.
\item $\ds \int_0^4 f(x) \; dx = F(4) - F(0) = \frac{5.0}{e^{4}} + 11.8$
\item La quantité calculée à la question précédente se retrouve 4fois pour former le bassin. Il faut ensuite prendre en compte l'échelle, comme 1unité de longueur correspond à 15m, une unité d'air correspond à $15\times15 = 225m^2$. Ainsi l'aire du bassin est égale à
\[
(\frac{5.0}{e^{4}} + 11.8)\times 4 \times 15^2 = 10702.00000
\]
\end{enumerate}
\end{solution}
\begin{exercise}[subtitle={Bassin}]
Le clinker est un constituant du ciment qui résulte de la cuisson d'un mélange composé de calcaire et d'argile. La fabrication du clinker nécessite des fours à très haute température qui libèrent dans l'air une grande quantité de dioxyde de carbone (CO$_2$).
Dans une cimenterie, la fabrication du clinker s'effectue de 7 h 30 à 20 h, dans une pièce de volume \np{500000}~dm$^3$.
À 20 h, après une journée de travail, le taux volumique de CO$_2$ dans la pièce est de 0.8\,\%.
\begin{enumerate}
\item Justifier que le volume de CO$_2$ présent dans cette pièce à 20 h est de \np{4000}~dm$^3$ .
\item On modélise le volume de CO$_2$ présent dans la pièce par une fonction du temps $t$ écoulé après 20h (exprimé en minutes) qui pour formule $V(t) = V_0e^{-0.05t} + 230$
\begin{enumerate}
\item Démontrer que $V_0$ est égale à \np{3770}.
\item Quel sera, au dm$^3$ près, le volume de CO$_2$ dans cette pièce à 24 h ?
\item Démontrer que $V'(t) = - 188.5 e^{- 0.05 t}$.
\item Étudier le signe de $V'(t)$ puis en déduire le sens de variation de $V(t)$.
\item Que peut-on dire du volume de CO$_2$ quand $t$ devient grand?
\end{enumerate}
\end{enumerate}
\end{exercise}
\begin{solution}
\begin{enumerate}
\item Volume à 20h: $500000\times 0.008 = 4000$
\item
\begin{enumerate}
\item $t=0$ correspond à 20h.
Donc $V(0) = 4000 = V_0e^{-0.05\times 0} + 230 = V_0 + 230$
Donc $V_0 = 4000 - 230 = 3770$
\item Il faut calculer $V(t)$ pour $t = 4$ donc
\[
V(4) = 3316.61
\]
\item Pas de correction pour cette question.
\item Pas de correction pour cette question.
\item Pas de correction pour cette question.
\end{enumerate}
\end{enumerate}
\end{solution}
\end{document}
%%% Local Variables:
%%% mode: latex
%%% TeX-master: "master"
%%% End:

Binary file not shown.

View File

@ -0,0 +1,121 @@
\documentclass[a4paper,10pt]{article}
\usepackage{myXsim}
\usepackage{tasks}
% Title Page
\title{DM1 \hfill BAHBAH Zakaria}
\tribe{TST sti2d}
\date{\hfillÀ render pour le jeudi 25 février}
\xsimsetup{
solution/print = true
}
\begin{document}
\maketitle
\begin{exercise}[subtitle={Complexes}]
\begin{enumerate}
\item Mettre le nombre complexe suivant sous forme algébrique $z_1 = \dfrac{7 + 2 i}{-5 + 3 i} $
\item Mettre le complexe suivante sous forme exponentielle $z_2 = -7 + 7 \sqrt{3} i$
\item Mettre le complexe suivante sous forme exponentielle $z_3 = 4 + 4 \sqrt{3} i$
\item Calculer le produit $z_4=z_2\times z_3$ donner le résultat sous forme exponentielle puis algébrique.
\item Calculer le quotient $z_5=\frac{z_2}{z_3}$ donner le résultat sous forme exponentielle puis algébrique.
\end{enumerate}
\end{exercise}
\begin{solution}
\begin{enumerate}
\item $z_1 = - \frac{29}{34} - \frac{31 i}{34}$
\item $z_3 = 14 e^{\frac{2 i \pi}{3}}$
\item $z_4 = 112 e^{i \pi} = -112 = -112.0$
\item $z_5 = \frac{7}{4} e^{\frac{i \pi}{3}} = \frac{7}{8} + \frac{7 \sqrt{3} i}{8} = 0.875 + 1.52 i$
\end{enumerate}
\end{solution}
\begin{exercise}[subtitle={Bassin}]
Le tour d'un bassin au niveau du sol présente deux axes de symétrie : laxe des abscisses et la droite déquation $x=4$. Il est obtenu par symétrie de la courbe $\mathcal{C_f}$ sur $\intFF{0}{4}$$f$ est la fonction définie par
\[
f(x) = \left(- x^{2} + 2.4 x - 6.2\right) e^{- x} + 6.2
\]
On admet que sur $\intFF{0}{4}$ la fonction $f$ est positive.
\begin{enumerate}
\item Sur un repère, tracer l'allure de la courbe $\mathcal{C}_f$, les axes de symétries puis compléter pour dessiner la forme du bassin.
\item Montrer que la fonction $f$ admet comme primitive sur $\R$ la fonction $F$ définie par
\[
F(x) = 6.2 x + \left( x^{2} - 0.4 x + 5.8\right) e^{- x}
\]
\item Calculer la quantité $\ds \int_0^4 f(x) \; dx$, vous donnerez le résultat sous forme exacte. Interpréter le résultat et reportez cette quantité sur le graphique.
\item On considère que l'échelle de votre graphique est de 1unité pour 15m. Calculer l'aire du bassin. Vous donnerez un résultat arrondi au $m^2$ près.
\end{enumerate}
\end{exercise}
\begin{solution}
\begin{enumerate}
\item
\begin{tikzpicture}[baseline=(a.north), xscale=1, yscale=0.5]
\tkzInit[xmin=0,xmax=5,xstep=1,
ymin=0,ymax=10,ystep=1]
\tkzGrid
\tkzAxeXY
\tkzFct[domain=0:10,color=red,very thick]%
{ (-x**2 + 2.4*x - 6.2)*exp(-x) + 6.2 };
\end{tikzpicture}
\item Il faut dériver $F(x)$ et vérifier que $F'(x) = f(x)$.
\item $\ds \int_0^4 f(x) \; dx = F(4) - F(0) = \frac{20.2}{e^{4}} + 19.0$
\item La quantité calculée à la question précédente se retrouve 4fois pour former le bassin. Il faut ensuite prendre en compte l'échelle, comme 1unité de longueur correspond à 15m, une unité d'air correspond à $15\times15 = 225m^2$. Ainsi l'aire du bassin est égale à
\[
(\frac{20.2}{e^{4}} + 19.0)\times 4 \times 15^2 = 17433.00000
\]
\end{enumerate}
\end{solution}
\begin{exercise}[subtitle={Bassin}]
Le clinker est un constituant du ciment qui résulte de la cuisson d'un mélange composé de calcaire et d'argile. La fabrication du clinker nécessite des fours à très haute température qui libèrent dans l'air une grande quantité de dioxyde de carbone (CO$_2$).
Dans une cimenterie, la fabrication du clinker s'effectue de 7 h 30 à 20 h, dans une pièce de volume \np{900000}~dm$^3$.
À 20 h, après une journée de travail, le taux volumique de CO$_2$ dans la pièce est de 0.9\,\%.
\begin{enumerate}
\item Justifier que le volume de CO$_2$ présent dans cette pièce à 20 h est de \np{8100}~dm$^3$ .
\item On modélise le volume de CO$_2$ présent dans la pièce par une fonction du temps $t$ écoulé après 20h (exprimé en minutes) qui pour formule $V(t) = V_0e^{-0.06t} + 560$
\begin{enumerate}
\item Démontrer que $V_0$ est égale à \np{7540}.
\item Quel sera, au dm$^3$ près, le volume de CO$_2$ dans cette pièce à 23 h ?
\item Démontrer que $V'(t) = - 452.4 e^{- 0.06 t}$.
\item Étudier le signe de $V'(t)$ puis en déduire le sens de variation de $V(t)$.
\item Que peut-on dire du volume de CO$_2$ quand $t$ devient grand?
\end{enumerate}
\end{enumerate}
\end{exercise}
\begin{solution}
\begin{enumerate}
\item Volume à 20h: $900000\times 0.009000000000000001 = 8100$
\item
\begin{enumerate}
\item $t=0$ correspond à 20h.
Donc $V(0) = 8100 = V_0e^{-0.06\times 0} + 560 = V_0 + 560$
Donc $V_0 = 8100 - 560 = 7540$
\item Il faut calculer $V(t)$ pour $t = 3$ donc
\[
V(3) = 6857.94
\]
\item Pas de correction pour cette question.
\item Pas de correction pour cette question.
\item Pas de correction pour cette question.
\end{enumerate}
\end{enumerate}
\end{solution}
\end{document}
%%% Local Variables:
%%% mode: latex
%%% TeX-master: "master"
%%% End:

View File

@ -0,0 +1,121 @@
\documentclass[a4paper,10pt]{article}
\usepackage{myXsim}
\usepackage{tasks}
% Title Page
\title{DM1 \hfill BENALI Ilyas}
\tribe{TST sti2d}
\date{\hfillÀ render pour le jeudi 25 février}
\xsimsetup{
solution/print = true
}
\begin{document}
\maketitle
\begin{exercise}[subtitle={Complexes}]
\begin{enumerate}
\item Mettre le nombre complexe suivant sous forme algébrique $z_1 = \dfrac{9 + 3 i}{-9 + 8 i} $
\item Mettre le complexe suivante sous forme exponentielle $z_2 = 2 \sqrt{2} + 2 \sqrt{2} i$
\item Mettre le complexe suivante sous forme exponentielle $z_3 = 4 \sqrt{2} - 4 \sqrt{2} i$
\item Calculer le produit $z_4=z_2\times z_3$ donner le résultat sous forme exponentielle puis algébrique.
\item Calculer le quotient $z_5=\frac{z_2}{z_3}$ donner le résultat sous forme exponentielle puis algébrique.
\end{enumerate}
\end{exercise}
\begin{solution}
\begin{enumerate}
\item $z_1 = - \frac{57}{145} - \frac{99 i}{145}$
\item $z_3 = 4 e^{\frac{i \pi}{4}}$
\item $z_4 = 32 e^{0} = 32 = 32.0$
\item $z_5 = \frac{1}{2} e^{\frac{i \pi}{2}} = \frac{i}{2} = 0.5 i$
\end{enumerate}
\end{solution}
\begin{exercise}[subtitle={Bassin}]
Le tour d'un bassin au niveau du sol présente deux axes de symétrie : laxe des abscisses et la droite déquation $x=4$. Il est obtenu par symétrie de la courbe $\mathcal{C_f}$ sur $\intFF{0}{4}$$f$ est la fonction définie par
\[
f(x) = \left(- x^{2} + 8.7 x - 5.0\right) e^{- x} + 5.0
\]
On admet que sur $\intFF{0}{4}$ la fonction $f$ est positive.
\begin{enumerate}
\item Sur un repère, tracer l'allure de la courbe $\mathcal{C}_f$, les axes de symétries puis compléter pour dessiner la forme du bassin.
\item Montrer que la fonction $f$ admet comme primitive sur $\R$ la fonction $F$ définie par
\[
F(x) = 5.0 x + \left( x^{2} - 6.7 x - 1.7\right) e^{- x}
\]
\item Calculer la quantité $\ds \int_0^4 f(x) \; dx$, vous donnerez le résultat sous forme exacte. Interpréter le résultat et reportez cette quantité sur le graphique.
\item On considère que l'échelle de votre graphique est de 1unité pour 15m. Calculer l'aire du bassin. Vous donnerez un résultat arrondi au $m^2$ près.
\end{enumerate}
\end{exercise}
\begin{solution}
\begin{enumerate}
\item
\begin{tikzpicture}[baseline=(a.north), xscale=1, yscale=0.5]
\tkzInit[xmin=0,xmax=5,xstep=1,
ymin=0,ymax=10,ystep=1]
\tkzGrid
\tkzAxeXY
\tkzFct[domain=0:10,color=red,very thick]%
{ (-x**2 + 8.7*x - 5.0)*exp(-x) + 5.0 };
\end{tikzpicture}
\item Il faut dériver $F(x)$ et vérifier que $F'(x) = f(x)$.
\item $\ds \int_0^4 f(x) \; dx = F(4) - F(0) = 21.7 - \frac{12.5}{e^{4}}$
\item La quantité calculée à la question précédente se retrouve 4fois pour former le bassin. Il faut ensuite prendre en compte l'échelle, comme 1unité de longueur correspond à 15m, une unité d'air correspond à $15\times15 = 225m^2$. Ainsi l'aire du bassin est égale à
\[
(21.7 - \frac{12.5}{e^{4}})\times 4 \times 15^2 = 19324.00000
\]
\end{enumerate}
\end{solution}
\begin{exercise}[subtitle={Bassin}]
Le clinker est un constituant du ciment qui résulte de la cuisson d'un mélange composé de calcaire et d'argile. La fabrication du clinker nécessite des fours à très haute température qui libèrent dans l'air une grande quantité de dioxyde de carbone (CO$_2$).
Dans une cimenterie, la fabrication du clinker s'effectue de 7 h 30 à 20 h, dans une pièce de volume \np{700000}~dm$^3$.
À 20 h, après une journée de travail, le taux volumique de CO$_2$ dans la pièce est de 0.8\,\%.
\begin{enumerate}
\item Justifier que le volume de CO$_2$ présent dans cette pièce à 20 h est de \np{5600}~dm$^3$ .
\item On modélise le volume de CO$_2$ présent dans la pièce par une fonction du temps $t$ écoulé après 20h (exprimé en minutes) qui pour formule $V(t) = V_0e^{-0.09t} + 360$
\begin{enumerate}
\item Démontrer que $V_0$ est égale à \np{5240}.
\item Quel sera, au dm$^3$ près, le volume de CO$_2$ dans cette pièce à 21 h ?
\item Démontrer que $V'(t) = - 471.6 e^{- 0.09 t}$.
\item Étudier le signe de $V'(t)$ puis en déduire le sens de variation de $V(t)$.
\item Que peut-on dire du volume de CO$_2$ quand $t$ devient grand?
\end{enumerate}
\end{enumerate}
\end{exercise}
\begin{solution}
\begin{enumerate}
\item Volume à 20h: $700000\times 0.008 = 5600$
\item
\begin{enumerate}
\item $t=0$ correspond à 20h.
Donc $V(0) = 5600 = V_0e^{-0.09\times 0} + 360 = V_0 + 360$
Donc $V_0 = 5600 - 360 = 5240$
\item Il faut calculer $V(t)$ pour $t = 1$ donc
\[
V(1) = 5149.00000000000
\]
\item Pas de correction pour cette question.
\item Pas de correction pour cette question.
\item Pas de correction pour cette question.
\end{enumerate}
\end{enumerate}
\end{solution}
\end{document}
%%% Local Variables:
%%% mode: latex
%%% TeX-master: "master"
%%% End:

View File

@ -0,0 +1,121 @@
\documentclass[a4paper,10pt]{article}
\usepackage{myXsim}
\usepackage{tasks}
% Title Page
\title{DM1 \hfill BERNADAT Noah}
\tribe{TST sti2d}
\date{\hfillÀ render pour le jeudi 25 février}
\xsimsetup{
solution/print = true
}
\begin{document}
\maketitle
\begin{exercise}[subtitle={Complexes}]
\begin{enumerate}
\item Mettre le nombre complexe suivant sous forme algébrique $z_1 = \dfrac{2 + 3 i}{-7 + 10 i} $
\item Mettre le complexe suivante sous forme exponentielle $z_2 = - 5 \sqrt{3} + 5 i$
\item Mettre le complexe suivante sous forme exponentielle $z_3 = 3 \sqrt{3} - 3 i$
\item Calculer le produit $z_4=z_2\times z_3$ donner le résultat sous forme exponentielle puis algébrique.
\item Calculer le quotient $z_5=\frac{z_2}{z_3}$ donner le résultat sous forme exponentielle puis algébrique.
\end{enumerate}
\end{exercise}
\begin{solution}
\begin{enumerate}
\item $z_1 = \frac{16}{149} - \frac{41 i}{149}$
\item $z_3 = 10 e^{\frac{5 i \pi}{6}}$
\item $z_4 = 60 e^{\frac{2 i \pi}{3}} = -30 + 30 \sqrt{3} i = -30.0 + 52.0 i$
\item $z_5 = \frac{5}{3} e^{i \pi} = - \frac{5}{3} = -1.67$
\end{enumerate}
\end{solution}
\begin{exercise}[subtitle={Bassin}]
Le tour d'un bassin au niveau du sol présente deux axes de symétrie : laxe des abscisses et la droite déquation $x=4$. Il est obtenu par symétrie de la courbe $\mathcal{C_f}$ sur $\intFF{0}{4}$$f$ est la fonction définie par
\[
f(x) = \left(- x^{2} + 4.9 x - 9.7\right) e^{- x} + 9.7
\]
On admet que sur $\intFF{0}{4}$ la fonction $f$ est positive.
\begin{enumerate}
\item Sur un repère, tracer l'allure de la courbe $\mathcal{C}_f$, les axes de symétries puis compléter pour dessiner la forme du bassin.
\item Montrer que la fonction $f$ admet comme primitive sur $\R$ la fonction $F$ définie par
\[
F(x) = 9.7 x + \left( x^{2} - 2.9 x + 6.8\right) e^{- x}
\]
\item Calculer la quantité $\ds \int_0^4 f(x) \; dx$, vous donnerez le résultat sous forme exacte. Interpréter le résultat et reportez cette quantité sur le graphique.
\item On considère que l'échelle de votre graphique est de 1unité pour 15m. Calculer l'aire du bassin. Vous donnerez un résultat arrondi au $m^2$ près.
\end{enumerate}
\end{exercise}
\begin{solution}
\begin{enumerate}
\item
\begin{tikzpicture}[baseline=(a.north), xscale=1, yscale=0.5]
\tkzInit[xmin=0,xmax=5,xstep=1,
ymin=0,ymax=10,ystep=1]
\tkzGrid
\tkzAxeXY
\tkzFct[domain=0:10,color=red,very thick]%
{ (-x**2 + 4.9*x - 9.7)*exp(-x) + 9.7 };
\end{tikzpicture}
\item Il faut dériver $F(x)$ et vérifier que $F'(x) = f(x)$.
\item $\ds \int_0^4 f(x) \; dx = F(4) - F(0) = \frac{11.2}{e^{4}} + 32.0$
\item La quantité calculée à la question précédente se retrouve 4fois pour former le bassin. Il faut ensuite prendre en compte l'échelle, comme 1unité de longueur correspond à 15m, une unité d'air correspond à $15\times15 = 225m^2$. Ainsi l'aire du bassin est égale à
\[
(\frac{11.2}{e^{4}} + 32.0)\times 4 \times 15^2 = 28985.00000
\]
\end{enumerate}
\end{solution}
\begin{exercise}[subtitle={Bassin}]
Le clinker est un constituant du ciment qui résulte de la cuisson d'un mélange composé de calcaire et d'argile. La fabrication du clinker nécessite des fours à très haute température qui libèrent dans l'air une grande quantité de dioxyde de carbone (CO$_2$).
Dans une cimenterie, la fabrication du clinker s'effectue de 7 h 30 à 20 h, dans une pièce de volume \np{400000}~dm$^3$.
À 20 h, après une journée de travail, le taux volumique de CO$_2$ dans la pièce est de 0.7\,\%.
\begin{enumerate}
\item Justifier que le volume de CO$_2$ présent dans cette pièce à 20 h est de \np{2800}~dm$^3$ .
\item On modélise le volume de CO$_2$ présent dans la pièce par une fonction du temps $t$ écoulé après 20h (exprimé en minutes) qui pour formule $V(t) = V_0e^{-0.06t} + 400$
\begin{enumerate}
\item Démontrer que $V_0$ est égale à \np{2400}.
\item Quel sera, au dm$^3$ près, le volume de CO$_2$ dans cette pièce à 23 h ?
\item Démontrer que $V'(t) = - 144.0 e^{- 0.06 t}$.
\item Étudier le signe de $V'(t)$ puis en déduire le sens de variation de $V(t)$.
\item Que peut-on dire du volume de CO$_2$ quand $t$ devient grand?
\end{enumerate}
\end{enumerate}
\end{exercise}
\begin{solution}
\begin{enumerate}
\item Volume à 20h: $400000\times 0.006999999999999999 = 2800$
\item
\begin{enumerate}
\item $t=0$ correspond à 20h.
Donc $V(0) = 2800 = V_0e^{-0.06\times 0} + 400 = V_0 + 400$
Donc $V_0 = 2800 - 400 = 2400$
\item Il faut calculer $V(t)$ pour $t = 3$ donc
\[
V(3) = 2404.65
\]
\item Pas de correction pour cette question.
\item Pas de correction pour cette question.
\item Pas de correction pour cette question.
\end{enumerate}
\end{enumerate}
\end{solution}
\end{document}
%%% Local Variables:
%%% mode: latex
%%% TeX-master: "master"
%%% End:

View File

@ -0,0 +1,121 @@
\documentclass[a4paper,10pt]{article}
\usepackage{myXsim}
\usepackage{tasks}
% Title Page
\title{DM1 \hfill BUDIN Nathan}
\tribe{TST sti2d}
\date{\hfillÀ render pour le jeudi 25 février}
\xsimsetup{
solution/print = true
}
\begin{document}
\maketitle
\begin{exercise}[subtitle={Complexes}]
\begin{enumerate}
\item Mettre le nombre complexe suivant sous forme algébrique $z_1 = \dfrac{7 + 2 i}{-2 + 3 i} $
\item Mettre le complexe suivante sous forme exponentielle $z_2 = - 6 \sqrt{3} - 6 i$
\item Mettre le complexe suivante sous forme exponentielle $z_3 = - 4 \sqrt{3} + 4 i$
\item Calculer le produit $z_4=z_2\times z_3$ donner le résultat sous forme exponentielle puis algébrique.
\item Calculer le quotient $z_5=\frac{z_2}{z_3}$ donner le résultat sous forme exponentielle puis algébrique.
\end{enumerate}
\end{exercise}
\begin{solution}
\begin{enumerate}
\item $z_1 = - \frac{8}{13} - \frac{25 i}{13}$
\item $z_3 = 12 e^{- \frac{5 i \pi}{6}}$
\item $z_4 = 96 e^{0} = 96 = 96.0$
\item $z_5 = \frac{3}{2} e^{- \frac{5 i \pi}{3}} = \frac{3}{4} + \frac{3 \sqrt{3} i}{4} = 0.75 + 1.3 i$
\end{enumerate}
\end{solution}
\begin{exercise}[subtitle={Bassin}]
Le tour d'un bassin au niveau du sol présente deux axes de symétrie : laxe des abscisses et la droite déquation $x=4$. Il est obtenu par symétrie de la courbe $\mathcal{C_f}$ sur $\intFF{0}{4}$$f$ est la fonction définie par
\[
f(x) = \left(- x^{2} + 4.9 x - 6.0\right) e^{- x} + 6.0
\]
On admet que sur $\intFF{0}{4}$ la fonction $f$ est positive.
\begin{enumerate}
\item Sur un repère, tracer l'allure de la courbe $\mathcal{C}_f$, les axes de symétries puis compléter pour dessiner la forme du bassin.
\item Montrer que la fonction $f$ admet comme primitive sur $\R$ la fonction $F$ définie par
\[
F(x) = 6.0 x + \left( x^{2} - 2.9 x + 3.1\right) e^{- x}
\]
\item Calculer la quantité $\ds \int_0^4 f(x) \; dx$, vous donnerez le résultat sous forme exacte. Interpréter le résultat et reportez cette quantité sur le graphique.
\item On considère que l'échelle de votre graphique est de 1unité pour 15m. Calculer l'aire du bassin. Vous donnerez un résultat arrondi au $m^2$ près.
\end{enumerate}
\end{exercise}
\begin{solution}
\begin{enumerate}
\item
\begin{tikzpicture}[baseline=(a.north), xscale=1, yscale=0.5]
\tkzInit[xmin=0,xmax=5,xstep=1,
ymin=0,ymax=10,ystep=1]
\tkzGrid
\tkzAxeXY
\tkzFct[domain=0:10,color=red,very thick]%
{ (-x**2 + 4.9*x - 6.0)*exp(-x) + 6.0 };
\end{tikzpicture}
\item Il faut dériver $F(x)$ et vérifier que $F'(x) = f(x)$.
\item $\ds \int_0^4 f(x) \; dx = F(4) - F(0) = \frac{7.5}{e^{4}} + 20.9$
\item La quantité calculée à la question précédente se retrouve 4fois pour former le bassin. Il faut ensuite prendre en compte l'échelle, comme 1unité de longueur correspond à 15m, une unité d'air correspond à $15\times15 = 225m^2$. Ainsi l'aire du bassin est égale à
\[
(\frac{7.5}{e^{4}} + 20.9)\times 4 \times 15^2 = 18934.00000
\]
\end{enumerate}
\end{solution}
\begin{exercise}[subtitle={Bassin}]
Le clinker est un constituant du ciment qui résulte de la cuisson d'un mélange composé de calcaire et d'argile. La fabrication du clinker nécessite des fours à très haute température qui libèrent dans l'air une grande quantité de dioxyde de carbone (CO$_2$).
Dans une cimenterie, la fabrication du clinker s'effectue de 7 h 30 à 20 h, dans une pièce de volume \np{400000}~dm$^3$.
À 20 h, après une journée de travail, le taux volumique de CO$_2$ dans la pièce est de 0.6\,\%.
\begin{enumerate}
\item Justifier que le volume de CO$_2$ présent dans cette pièce à 20 h est de \np{2400}~dm$^3$ .
\item On modélise le volume de CO$_2$ présent dans la pièce par une fonction du temps $t$ écoulé après 20h (exprimé en minutes) qui pour formule $V(t) = V_0e^{-0.0t} + 580$
\begin{enumerate}
\item Démontrer que $V_0$ est égale à \np{1820}.
\item Quel sera, au dm$^3$ près, le volume de CO$_2$ dans cette pièce à 22 h ?
\item Démontrer que $V'(t) = 0$.
\item Étudier le signe de $V'(t)$ puis en déduire le sens de variation de $V(t)$.
\item Que peut-on dire du volume de CO$_2$ quand $t$ devient grand?
\end{enumerate}
\end{enumerate}
\end{exercise}
\begin{solution}
\begin{enumerate}
\item Volume à 20h: $400000\times 0.006 = 2400$
\item
\begin{enumerate}
\item $t=0$ correspond à 20h.
Donc $V(0) = 2400 = V_0e^{-0.0\times 0} + 580 = V_0 + 580$
Donc $V_0 = 2400 - 580 = 1820$
\item Il faut calculer $V(t)$ pour $t = 2$ donc
\[
V(2) = 2400
\]
\item Pas de correction pour cette question.
\item Pas de correction pour cette question.
\item Pas de correction pour cette question.
\end{enumerate}
\end{enumerate}
\end{solution}
\end{document}
%%% Local Variables:
%%% mode: latex
%%% TeX-master: "master"
%%% End:

View File

@ -0,0 +1,121 @@
\documentclass[a4paper,10pt]{article}
\usepackage{myXsim}
\usepackage{tasks}
% Title Page
\title{DM1 \hfill CHION Léa}
\tribe{TST sti2d}
\date{\hfillÀ render pour le jeudi 25 février}
\xsimsetup{
solution/print = true
}
\begin{document}
\maketitle
\begin{exercise}[subtitle={Complexes}]
\begin{enumerate}
\item Mettre le nombre complexe suivant sous forme algébrique $z_1 = \dfrac{9 + 6 i}{-3 + 5 i} $
\item Mettre le complexe suivante sous forme exponentielle $z_2 = -7 + 7 \sqrt{3} i$
\item Mettre le complexe suivante sous forme exponentielle $z_3 = -5 - 5 \sqrt{3} i$
\item Calculer le produit $z_4=z_2\times z_3$ donner le résultat sous forme exponentielle puis algébrique.
\item Calculer le quotient $z_5=\frac{z_2}{z_3}$ donner le résultat sous forme exponentielle puis algébrique.
\end{enumerate}
\end{exercise}
\begin{solution}
\begin{enumerate}
\item $z_1 = \frac{3}{34} - \frac{63 i}{34}$
\item $z_3 = 14 e^{\frac{2 i \pi}{3}}$
\item $z_4 = 140 e^{0} = 140 = 140.0$
\item $z_5 = \frac{7}{5} e^{\frac{4 i \pi}{3}} = - \frac{7}{10} - \frac{7 \sqrt{3} i}{10} = -0.7 - 1.21 i$
\end{enumerate}
\end{solution}
\begin{exercise}[subtitle={Bassin}]
Le tour d'un bassin au niveau du sol présente deux axes de symétrie : laxe des abscisses et la droite déquation $x=4$. Il est obtenu par symétrie de la courbe $\mathcal{C_f}$ sur $\intFF{0}{4}$$f$ est la fonction définie par
\[
f(x) = \left(- x^{2} + 6.2 x - 7.3\right) e^{- x} + 7.3
\]
On admet que sur $\intFF{0}{4}$ la fonction $f$ est positive.
\begin{enumerate}
\item Sur un repère, tracer l'allure de la courbe $\mathcal{C}_f$, les axes de symétries puis compléter pour dessiner la forme du bassin.
\item Montrer que la fonction $f$ admet comme primitive sur $\R$ la fonction $F$ définie par
\[
F(x) = 7.3 x + \left( x^{2} - 4.2 x + 3.1\right) e^{- x}
\]
\item Calculer la quantité $\ds \int_0^4 f(x) \; dx$, vous donnerez le résultat sous forme exacte. Interpréter le résultat et reportez cette quantité sur le graphique.
\item On considère que l'échelle de votre graphique est de 1unité pour 15m. Calculer l'aire du bassin. Vous donnerez un résultat arrondi au $m^2$ près.
\end{enumerate}
\end{exercise}
\begin{solution}
\begin{enumerate}
\item
\begin{tikzpicture}[baseline=(a.north), xscale=1, yscale=0.5]
\tkzInit[xmin=0,xmax=5,xstep=1,
ymin=0,ymax=10,ystep=1]
\tkzGrid
\tkzAxeXY
\tkzFct[domain=0:10,color=red,very thick]%
{ (-x**2 + 6.2*x - 7.3)*exp(-x) + 7.3 };
\end{tikzpicture}
\item Il faut dériver $F(x)$ et vérifier que $F'(x) = f(x)$.
\item $\ds \int_0^4 f(x) \; dx = F(4) - F(0) = \frac{2.3}{e^{4}} + 26.1$
\item La quantité calculée à la question précédente se retrouve 4fois pour former le bassin. Il faut ensuite prendre en compte l'échelle, comme 1unité de longueur correspond à 15m, une unité d'air correspond à $15\times15 = 225m^2$. Ainsi l'aire du bassin est égale à
\[
(\frac{2.3}{e^{4}} + 26.1)\times 4 \times 15^2 = 23528.00000
\]
\end{enumerate}
\end{solution}
\begin{exercise}[subtitle={Bassin}]
Le clinker est un constituant du ciment qui résulte de la cuisson d'un mélange composé de calcaire et d'argile. La fabrication du clinker nécessite des fours à très haute température qui libèrent dans l'air une grande quantité de dioxyde de carbone (CO$_2$).
Dans une cimenterie, la fabrication du clinker s'effectue de 7 h 30 à 20 h, dans une pièce de volume \np{300000}~dm$^3$.
À 20 h, après une journée de travail, le taux volumique de CO$_2$ dans la pièce est de 0.6\,\%.
\begin{enumerate}
\item Justifier que le volume de CO$_2$ présent dans cette pièce à 20 h est de \np{1800}~dm$^3$ .
\item On modélise le volume de CO$_2$ présent dans la pièce par une fonction du temps $t$ écoulé après 20h (exprimé en minutes) qui pour formule $V(t) = V_0e^{-0.01t} + 260$
\begin{enumerate}
\item Démontrer que $V_0$ est égale à \np{1540}.
\item Quel sera, au dm$^3$ près, le volume de CO$_2$ dans cette pièce à 22 h ?
\item Démontrer que $V'(t) = - 15.4 e^{- 0.01 t}$.
\item Étudier le signe de $V'(t)$ puis en déduire le sens de variation de $V(t)$.
\item Que peut-on dire du volume de CO$_2$ quand $t$ devient grand?
\end{enumerate}
\end{enumerate}
\end{exercise}
\begin{solution}
\begin{enumerate}
\item Volume à 20h: $300000\times 0.006 = 1800$
\item
\begin{enumerate}
\item $t=0$ correspond à 20h.
Donc $V(0) = 1800 = V_0e^{-0.01\times 0} + 260 = V_0 + 260$
Donc $V_0 = 1800 - 260 = 1540$
\item Il faut calculer $V(t)$ pour $t = 2$ donc
\[
V(2) = 1769.51
\]
\item Pas de correction pour cette question.
\item Pas de correction pour cette question.
\item Pas de correction pour cette question.
\end{enumerate}
\end{enumerate}
\end{solution}
\end{document}
%%% Local Variables:
%%% mode: latex
%%% TeX-master: "master"
%%% End:

View File

@ -0,0 +1,121 @@
\documentclass[a4paper,10pt]{article}
\usepackage{myXsim}
\usepackage{tasks}
% Title Page
\title{DM1 \hfill CLAIN Avinash}
\tribe{TST sti2d}
\date{\hfillÀ render pour le jeudi 25 février}
\xsimsetup{
solution/print = true
}
\begin{document}
\maketitle
\begin{exercise}[subtitle={Complexes}]
\begin{enumerate}
\item Mettre le nombre complexe suivant sous forme algébrique $z_1 = \dfrac{4 + 10 i}{-8 + 3 i} $
\item Mettre le complexe suivante sous forme exponentielle $z_2 = - 4 \sqrt{3} + 4 i$
\item Mettre le complexe suivante sous forme exponentielle $z_3 = 2 \sqrt{2} - 2 \sqrt{2} i$
\item Calculer le produit $z_4=z_2\times z_3$ donner le résultat sous forme exponentielle puis algébrique.
\item Calculer le quotient $z_5=\frac{z_2}{z_3}$ donner le résultat sous forme exponentielle puis algébrique.
\end{enumerate}
\end{exercise}
\begin{solution}
\begin{enumerate}
\item $z_1 = - \frac{2}{73} - \frac{92 i}{73}$
\item $z_3 = 8 e^{\frac{5 i \pi}{6}}$
\item $z_4 = 32 e^{\frac{7 i \pi}{12}} = - 8 \sqrt{6} + 8 \sqrt{2} + i \left(8 \sqrt{2} + 8 \sqrt{6}\right) = -8.28 + 30.9 i$
\item $z_5 = 2 e^{\frac{13 i \pi}{12}} = - \frac{\sqrt{6}}{2} - \frac{\sqrt{2}}{2} + i \left(- \frac{\sqrt{6}}{2} + \frac{\sqrt{2}}{2}\right) = -1.93 - 0.518 i$
\end{enumerate}
\end{solution}
\begin{exercise}[subtitle={Bassin}]
Le tour d'un bassin au niveau du sol présente deux axes de symétrie : laxe des abscisses et la droite déquation $x=4$. Il est obtenu par symétrie de la courbe $\mathcal{C_f}$ sur $\intFF{0}{4}$$f$ est la fonction définie par
\[
f(x) = \left(- x^{2} + 1.8 x - 4.9\right) e^{- x} + 4.9
\]
On admet que sur $\intFF{0}{4}$ la fonction $f$ est positive.
\begin{enumerate}
\item Sur un repère, tracer l'allure de la courbe $\mathcal{C}_f$, les axes de symétries puis compléter pour dessiner la forme du bassin.
\item Montrer que la fonction $f$ admet comme primitive sur $\R$ la fonction $F$ définie par
\[
F(x) = 4.9 x + \left( x^{2} + 0.2 x + 5.1\right) e^{- x}
\]
\item Calculer la quantité $\ds \int_0^4 f(x) \; dx$, vous donnerez le résultat sous forme exacte. Interpréter le résultat et reportez cette quantité sur le graphique.
\item On considère que l'échelle de votre graphique est de 1unité pour 15m. Calculer l'aire du bassin. Vous donnerez un résultat arrondi au $m^2$ près.
\end{enumerate}
\end{exercise}
\begin{solution}
\begin{enumerate}
\item
\begin{tikzpicture}[baseline=(a.north), xscale=1, yscale=0.5]
\tkzInit[xmin=0,xmax=5,xstep=1,
ymin=0,ymax=10,ystep=1]
\tkzGrid
\tkzAxeXY
\tkzFct[domain=0:10,color=red,very thick]%
{ (-x**2 + 1.8*x - 4.9)*exp(-x) + 4.9 };
\end{tikzpicture}
\item Il faut dériver $F(x)$ et vérifier que $F'(x) = f(x)$.
\item $\ds \int_0^4 f(x) \; dx = F(4) - F(0) = \frac{21.9}{e^{4}} + 14.5$
\item La quantité calculée à la question précédente se retrouve 4fois pour former le bassin. Il faut ensuite prendre en compte l'échelle, comme 1unité de longueur correspond à 15m, une unité d'air correspond à $15\times15 = 225m^2$. Ainsi l'aire du bassin est égale à
\[
(\frac{21.9}{e^{4}} + 14.5)\times 4 \times 15^2 = 13411.00000
\]
\end{enumerate}
\end{solution}
\begin{exercise}[subtitle={Bassin}]
Le clinker est un constituant du ciment qui résulte de la cuisson d'un mélange composé de calcaire et d'argile. La fabrication du clinker nécessite des fours à très haute température qui libèrent dans l'air une grande quantité de dioxyde de carbone (CO$_2$).
Dans une cimenterie, la fabrication du clinker s'effectue de 7 h 30 à 20 h, dans une pièce de volume \np{1000000}~dm$^3$.
À 20 h, après une journée de travail, le taux volumique de CO$_2$ dans la pièce est de 0.8\,\%.
\begin{enumerate}
\item Justifier que le volume de CO$_2$ présent dans cette pièce à 20 h est de \np{8000}~dm$^3$ .
\item On modélise le volume de CO$_2$ présent dans la pièce par une fonction du temps $t$ écoulé après 20h (exprimé en minutes) qui pour formule $V(t) = V_0e^{-0.09t} + 580$
\begin{enumerate}
\item Démontrer que $V_0$ est égale à \np{7420}.
\item Quel sera, au dm$^3$ près, le volume de CO$_2$ dans cette pièce à 21 h ?
\item Démontrer que $V'(t) = - 667.8 e^{- 0.09 t}$.
\item Étudier le signe de $V'(t)$ puis en déduire le sens de variation de $V(t)$.
\item Que peut-on dire du volume de CO$_2$ quand $t$ devient grand?
\end{enumerate}
\end{enumerate}
\end{exercise}
\begin{solution}
\begin{enumerate}
\item Volume à 20h: $1000000\times 0.008 = 8000$
\item
\begin{enumerate}
\item $t=0$ correspond à 20h.
Donc $V(0) = 8000 = V_0e^{-0.09\times 0} + 580 = V_0 + 580$
Donc $V_0 = 8000 - 580 = 7420$
\item Il faut calculer $V(t)$ pour $t = 1$ donc
\[
V(1) = 7361.37
\]
\item Pas de correction pour cette question.
\item Pas de correction pour cette question.
\item Pas de correction pour cette question.
\end{enumerate}
\end{enumerate}
\end{solution}
\end{document}
%%% Local Variables:
%%% mode: latex
%%% TeX-master: "master"
%%% End:

View File

@ -0,0 +1,121 @@
\documentclass[a4paper,10pt]{article}
\usepackage{myXsim}
\usepackage{tasks}
% Title Page
\title{DM1 \hfill COUBAT Alexis}
\tribe{TST sti2d}
\date{\hfillÀ render pour le jeudi 25 février}
\xsimsetup{
solution/print = true
}
\begin{document}
\maketitle
\begin{exercise}[subtitle={Complexes}]
\begin{enumerate}
\item Mettre le nombre complexe suivant sous forme algébrique $z_1 = \dfrac{7 + 3 i}{-2 + 2 i} $
\item Mettre le complexe suivante sous forme exponentielle $z_2 = 1 - \sqrt{3} i$
\item Mettre le complexe suivante sous forme exponentielle $z_3 = - 2 \sqrt{3} + 2 i$
\item Calculer le produit $z_4=z_2\times z_3$ donner le résultat sous forme exponentielle puis algébrique.
\item Calculer le quotient $z_5=\frac{z_2}{z_3}$ donner le résultat sous forme exponentielle puis algébrique.
\end{enumerate}
\end{exercise}
\begin{solution}
\begin{enumerate}
\item $z_1 = -1 - \frac{5 i}{2}$
\item $z_3 = 2 e^{- \frac{i \pi}{3}}$
\item $z_4 = 8 e^{\frac{i \pi}{2}} = 8 i = 8.0 i$
\item $z_5 = \frac{1}{2} e^{- \frac{7 i \pi}{6}} = - \frac{\sqrt{3}}{4} + \frac{i}{4} = -0.433 + 0.25 i$
\end{enumerate}
\end{solution}
\begin{exercise}[subtitle={Bassin}]
Le tour d'un bassin au niveau du sol présente deux axes de symétrie : laxe des abscisses et la droite déquation $x=4$. Il est obtenu par symétrie de la courbe $\mathcal{C_f}$ sur $\intFF{0}{4}$$f$ est la fonction définie par
\[
f(x) = \left(- x^{2} + 5.6 x - 4.5\right) e^{- x} + 4.5
\]
On admet que sur $\intFF{0}{4}$ la fonction $f$ est positive.
\begin{enumerate}
\item Sur un repère, tracer l'allure de la courbe $\mathcal{C}_f$, les axes de symétries puis compléter pour dessiner la forme du bassin.
\item Montrer que la fonction $f$ admet comme primitive sur $\R$ la fonction $F$ définie par
\[
F(x) = 4.5 x + \left( x^{2} - 3.6 x + 0.9\right) e^{- x}
\]
\item Calculer la quantité $\ds \int_0^4 f(x) \; dx$, vous donnerez le résultat sous forme exacte. Interpréter le résultat et reportez cette quantité sur le graphique.
\item On considère que l'échelle de votre graphique est de 1unité pour 15m. Calculer l'aire du bassin. Vous donnerez un résultat arrondi au $m^2$ près.
\end{enumerate}
\end{exercise}
\begin{solution}
\begin{enumerate}
\item
\begin{tikzpicture}[baseline=(a.north), xscale=1, yscale=0.5]
\tkzInit[xmin=0,xmax=5,xstep=1,
ymin=0,ymax=10,ystep=1]
\tkzGrid
\tkzAxeXY
\tkzFct[domain=0:10,color=red,very thick]%
{ (-x**2 + 5.6*x - 4.5)*exp(-x) + 4.5 };
\end{tikzpicture}
\item Il faut dériver $F(x)$ et vérifier que $F'(x) = f(x)$.
\item $\ds \int_0^4 f(x) \; dx = F(4) - F(0) = \frac{2.5}{e^{4}} + 17.1$
\item La quantité calculée à la question précédente se retrouve 4fois pour former le bassin. Il faut ensuite prendre en compte l'échelle, comme 1unité de longueur correspond à 15m, une unité d'air correspond à $15\times15 = 225m^2$. Ainsi l'aire du bassin est égale à
\[
(\frac{2.5}{e^{4}} + 17.1)\times 4 \times 15^2 = 15431.00000
\]
\end{enumerate}
\end{solution}
\begin{exercise}[subtitle={Bassin}]
Le clinker est un constituant du ciment qui résulte de la cuisson d'un mélange composé de calcaire et d'argile. La fabrication du clinker nécessite des fours à très haute température qui libèrent dans l'air une grande quantité de dioxyde de carbone (CO$_2$).
Dans une cimenterie, la fabrication du clinker s'effectue de 7 h 30 à 20 h, dans une pièce de volume \np{800000}~dm$^3$.
À 20 h, après une journée de travail, le taux volumique de CO$_2$ dans la pièce est de 0.7\,\%.
\begin{enumerate}
\item Justifier que le volume de CO$_2$ présent dans cette pièce à 20 h est de \np{5600}~dm$^3$ .
\item On modélise le volume de CO$_2$ présent dans la pièce par une fonction du temps $t$ écoulé après 20h (exprimé en minutes) qui pour formule $V(t) = V_0e^{-0.02t} + 340$
\begin{enumerate}
\item Démontrer que $V_0$ est égale à \np{5260}.
\item Quel sera, au dm$^3$ près, le volume de CO$_2$ dans cette pièce à 24 h ?
\item Démontrer que $V'(t) = - 105.2 e^{- 0.02 t}$.
\item Étudier le signe de $V'(t)$ puis en déduire le sens de variation de $V(t)$.
\item Que peut-on dire du volume de CO$_2$ quand $t$ devient grand?
\end{enumerate}
\end{enumerate}
\end{exercise}
\begin{solution}
\begin{enumerate}
\item Volume à 20h: $800000\times 0.006999999999999999 = 5600$
\item
\begin{enumerate}
\item $t=0$ correspond à 20h.
Donc $V(0) = 5600 = V_0e^{-0.02\times 0} + 340 = V_0 + 340$
Donc $V_0 = 5600 - 340 = 5260$
\item Il faut calculer $V(t)$ pour $t = 4$ donc
\[
V(4) = 5195.59
\]
\item Pas de correction pour cette question.
\item Pas de correction pour cette question.
\item Pas de correction pour cette question.
\end{enumerate}
\end{enumerate}
\end{solution}
\end{document}
%%% Local Variables:
%%% mode: latex
%%% TeX-master: "master"
%%% End:

View File

@ -0,0 +1,121 @@
\documentclass[a4paper,10pt]{article}
\usepackage{myXsim}
\usepackage{tasks}
% Title Page
\title{DM1 \hfill EVRARD Jules}
\tribe{TST sti2d}
\date{\hfillÀ render pour le jeudi 25 février}
\xsimsetup{
solution/print = true
}
\begin{document}
\maketitle
\begin{exercise}[subtitle={Complexes}]
\begin{enumerate}
\item Mettre le nombre complexe suivant sous forme algébrique $z_1 = \dfrac{8 + 3 i}{-9 + 3 i} $
\item Mettre le complexe suivante sous forme exponentielle $z_2 = 3 - 3 \sqrt{3} i$
\item Mettre le complexe suivante sous forme exponentielle $z_3 = \sqrt{2} - \sqrt{2} i$
\item Calculer le produit $z_4=z_2\times z_3$ donner le résultat sous forme exponentielle puis algébrique.
\item Calculer le quotient $z_5=\frac{z_2}{z_3}$ donner le résultat sous forme exponentielle puis algébrique.
\end{enumerate}
\end{exercise}
\begin{solution}
\begin{enumerate}
\item $z_1 = - \frac{7}{10} - \frac{17 i}{30}$
\item $z_3 = 6 e^{- \frac{i \pi}{3}}$
\item $z_4 = 12 e^{- \frac{7 i \pi}{12}} = - 3 \sqrt{6} + 3 \sqrt{2} + i \left(- 3 \sqrt{6} - 3 \sqrt{2}\right) = -3.11 - 11.6 i$
\item $z_5 = 3 e^{- \frac{i \pi}{12}} = \frac{3 \sqrt{2}}{4} + \frac{3 \sqrt{6}}{4} + i \left(- \frac{3 \sqrt{6}}{4} + \frac{3 \sqrt{2}}{4}\right) = 2.9 - 0.776 i$
\end{enumerate}
\end{solution}
\begin{exercise}[subtitle={Bassin}]
Le tour d'un bassin au niveau du sol présente deux axes de symétrie : laxe des abscisses et la droite déquation $x=4$. Il est obtenu par symétrie de la courbe $\mathcal{C_f}$ sur $\intFF{0}{4}$$f$ est la fonction définie par
\[
f(x) = \left(- x^{2} + 0.9 x - 3.2\right) e^{- x} + 3.2
\]
On admet que sur $\intFF{0}{4}$ la fonction $f$ est positive.
\begin{enumerate}
\item Sur un repère, tracer l'allure de la courbe $\mathcal{C}_f$, les axes de symétries puis compléter pour dessiner la forme du bassin.
\item Montrer que la fonction $f$ admet comme primitive sur $\R$ la fonction $F$ définie par
\[
F(x) = 3.2 x + \left( x^{2} + 1.1 x + 4.3\right) e^{- x}
\]
\item Calculer la quantité $\ds \int_0^4 f(x) \; dx$, vous donnerez le résultat sous forme exacte. Interpréter le résultat et reportez cette quantité sur le graphique.
\item On considère que l'échelle de votre graphique est de 1unité pour 15m. Calculer l'aire du bassin. Vous donnerez un résultat arrondi au $m^2$ près.
\end{enumerate}
\end{exercise}
\begin{solution}
\begin{enumerate}
\item
\begin{tikzpicture}[baseline=(a.north), xscale=1, yscale=0.5]
\tkzInit[xmin=0,xmax=5,xstep=1,
ymin=0,ymax=10,ystep=1]
\tkzGrid
\tkzAxeXY
\tkzFct[domain=0:10,color=red,very thick]%
{ (-x**2 + 0.9*x - 3.2)*exp(-x) + 3.2 };
\end{tikzpicture}
\item Il faut dériver $F(x)$ et vérifier que $F'(x) = f(x)$.
\item $\ds \int_0^4 f(x) \; dx = F(4) - F(0) = \frac{24.7}{e^{4}} + 8.5$
\item La quantité calculée à la question précédente se retrouve 4fois pour former le bassin. Il faut ensuite prendre en compte l'échelle, comme 1unité de longueur correspond à 15m, une unité d'air correspond à $15\times15 = 225m^2$. Ainsi l'aire du bassin est égale à
\[
(\frac{24.7}{e^{4}} + 8.5)\times 4 \times 15^2 = 8057.000000
\]
\end{enumerate}
\end{solution}
\begin{exercise}[subtitle={Bassin}]
Le clinker est un constituant du ciment qui résulte de la cuisson d'un mélange composé de calcaire et d'argile. La fabrication du clinker nécessite des fours à très haute température qui libèrent dans l'air une grande quantité de dioxyde de carbone (CO$_2$).
Dans une cimenterie, la fabrication du clinker s'effectue de 7 h 30 à 20 h, dans une pièce de volume \np{800000}~dm$^3$.
À 20 h, après une journée de travail, le taux volumique de CO$_2$ dans la pièce est de 1.0\,\%.
\begin{enumerate}
\item Justifier que le volume de CO$_2$ présent dans cette pièce à 20 h est de \np{8000}~dm$^3$ .
\item On modélise le volume de CO$_2$ présent dans la pièce par une fonction du temps $t$ écoulé après 20h (exprimé en minutes) qui pour formule $V(t) = V_0e^{-0.02t} + 560$
\begin{enumerate}
\item Démontrer que $V_0$ est égale à \np{7440}.
\item Quel sera, au dm$^3$ près, le volume de CO$_2$ dans cette pièce à 23 h ?
\item Démontrer que $V'(t) = - 148.8 e^{- 0.02 t}$.
\item Étudier le signe de $V'(t)$ puis en déduire le sens de variation de $V(t)$.
\item Que peut-on dire du volume de CO$_2$ quand $t$ devient grand?
\end{enumerate}
\end{enumerate}
\end{exercise}
\begin{solution}
\begin{enumerate}
\item Volume à 20h: $800000\times 0.01 = 8000$
\item
\begin{enumerate}
\item $t=0$ correspond à 20h.
Donc $V(0) = 8000 = V_0e^{-0.02\times 0} + 560 = V_0 + 560$
Donc $V_0 = 8000 - 560 = 7440$
\item Il faut calculer $V(t)$ pour $t = 3$ donc
\[
V(3) = 7566.73
\]
\item Pas de correction pour cette question.
\item Pas de correction pour cette question.
\item Pas de correction pour cette question.
\end{enumerate}
\end{enumerate}
\end{solution}
\end{document}
%%% Local Variables:
%%% mode: latex
%%% TeX-master: "master"
%%% End:

View File

@ -0,0 +1,121 @@
\documentclass[a4paper,10pt]{article}
\usepackage{myXsim}
\usepackage{tasks}
% Title Page
\title{DM1 \hfill HADJRAS Mohcine}
\tribe{TST sti2d}
\date{\hfillÀ render pour le jeudi 25 février}
\xsimsetup{
solution/print = true
}
\begin{document}
\maketitle
\begin{exercise}[subtitle={Complexes}]
\begin{enumerate}
\item Mettre le nombre complexe suivant sous forme algébrique $z_1 = \dfrac{5 + 3 i}{-5 + 7 i} $
\item Mettre le complexe suivante sous forme exponentielle $z_2 = 2 \sqrt{3} + 2 i$
\item Mettre le complexe suivante sous forme exponentielle $z_3 = -10 - 10 \sqrt{3} i$
\item Calculer le produit $z_4=z_2\times z_3$ donner le résultat sous forme exponentielle puis algébrique.
\item Calculer le quotient $z_5=\frac{z_2}{z_3}$ donner le résultat sous forme exponentielle puis algébrique.
\end{enumerate}
\end{exercise}
\begin{solution}
\begin{enumerate}
\item $z_1 = - \frac{2}{37} - \frac{25 i}{37}$
\item $z_3 = 4 e^{\frac{i \pi}{6}}$
\item $z_4 = 80 e^{- \frac{i \pi}{2}} = - 80 i = - 80.0 i$
\item $z_5 = \frac{1}{5} e^{\frac{5 i \pi}{6}} = - \frac{\sqrt{3}}{10} + \frac{i}{10} = -0.173 + 0.1 i$
\end{enumerate}
\end{solution}
\begin{exercise}[subtitle={Bassin}]
Le tour d'un bassin au niveau du sol présente deux axes de symétrie : laxe des abscisses et la droite déquation $x=4$. Il est obtenu par symétrie de la courbe $\mathcal{C_f}$ sur $\intFF{0}{4}$$f$ est la fonction définie par
\[
f(x) = \left(- x^{2} + 7.5 x - 9.0\right) e^{- x} + 9.0
\]
On admet que sur $\intFF{0}{4}$ la fonction $f$ est positive.
\begin{enumerate}
\item Sur un repère, tracer l'allure de la courbe $\mathcal{C}_f$, les axes de symétries puis compléter pour dessiner la forme du bassin.
\item Montrer que la fonction $f$ admet comme primitive sur $\R$ la fonction $F$ définie par
\[
F(x) = 9.0 x + \left( x^{2} - 5.5 x + 3.5\right) e^{- x}
\]
\item Calculer la quantité $\ds \int_0^4 f(x) \; dx$, vous donnerez le résultat sous forme exacte. Interpréter le résultat et reportez cette quantité sur le graphique.
\item On considère que l'échelle de votre graphique est de 1unité pour 15m. Calculer l'aire du bassin. Vous donnerez un résultat arrondi au $m^2$ près.
\end{enumerate}
\end{exercise}
\begin{solution}
\begin{enumerate}
\item
\begin{tikzpicture}[baseline=(a.north), xscale=1, yscale=0.5]
\tkzInit[xmin=0,xmax=5,xstep=1,
ymin=0,ymax=10,ystep=1]
\tkzGrid
\tkzAxeXY
\tkzFct[domain=0:10,color=red,very thick]%
{ (-x**2 + 7.5*x - 9.0)*exp(-x) + 9.0 };
\end{tikzpicture}
\item Il faut dériver $F(x)$ et vérifier que $F'(x) = f(x)$.
\item $\ds \int_0^4 f(x) \; dx = F(4) - F(0) = 32.5 - \frac{2.5}{e^{4}}$
\item La quantité calculée à la question précédente se retrouve 4fois pour former le bassin. Il faut ensuite prendre en compte l'échelle, comme 1unité de longueur correspond à 15m, une unité d'air correspond à $15\times15 = 225m^2$. Ainsi l'aire du bassin est égale à
\[
(32.5 - \frac{2.5}{e^{4}})\times 4 \times 15^2 = 29209.00000
\]
\end{enumerate}
\end{solution}
\begin{exercise}[subtitle={Bassin}]
Le clinker est un constituant du ciment qui résulte de la cuisson d'un mélange composé de calcaire et d'argile. La fabrication du clinker nécessite des fours à très haute température qui libèrent dans l'air une grande quantité de dioxyde de carbone (CO$_2$).
Dans une cimenterie, la fabrication du clinker s'effectue de 7 h 30 à 20 h, dans une pièce de volume \np{1000000}~dm$^3$.
À 20 h, après une journée de travail, le taux volumique de CO$_2$ dans la pièce est de 0.9\,\%.
\begin{enumerate}
\item Justifier que le volume de CO$_2$ présent dans cette pièce à 20 h est de \np{9000}~dm$^3$ .
\item On modélise le volume de CO$_2$ présent dans la pièce par une fonction du temps $t$ écoulé après 20h (exprimé en minutes) qui pour formule $V(t) = V_0e^{-0.01t} + 330$
\begin{enumerate}
\item Démontrer que $V_0$ est égale à \np{8670}.
\item Quel sera, au dm$^3$ près, le volume de CO$_2$ dans cette pièce à 24 h ?
\item Démontrer que $V'(t) = - 86.7 e^{- 0.01 t}$.
\item Étudier le signe de $V'(t)$ puis en déduire le sens de variation de $V(t)$.
\item Que peut-on dire du volume de CO$_2$ quand $t$ devient grand?
\end{enumerate}
\end{enumerate}
\end{exercise}
\begin{solution}
\begin{enumerate}
\item Volume à 20h: $1000000\times 0.009000000000000001 = 9000$
\item
\begin{enumerate}
\item $t=0$ correspond à 20h.
Donc $V(0) = 9000 = V_0e^{-0.01\times 0} + 330 = V_0 + 330$
Donc $V_0 = 9000 - 330 = 8670$
\item Il faut calculer $V(t)$ pour $t = 4$ donc
\[
V(4) = 8660.04
\]
\item Pas de correction pour cette question.
\item Pas de correction pour cette question.
\item Pas de correction pour cette question.
\end{enumerate}
\end{enumerate}
\end{solution}
\end{document}
%%% Local Variables:
%%% mode: latex
%%% TeX-master: "master"
%%% End:

View File

@ -0,0 +1,121 @@
\documentclass[a4paper,10pt]{article}
\usepackage{myXsim}
\usepackage{tasks}
% Title Page
\title{DM1 \hfill HENRIST Maxime}
\tribe{TST sti2d}
\date{\hfillÀ render pour le jeudi 25 février}
\xsimsetup{
solution/print = true
}
\begin{document}
\maketitle
\begin{exercise}[subtitle={Complexes}]
\begin{enumerate}
\item Mettre le nombre complexe suivant sous forme algébrique $z_1 = \dfrac{6 + 6 i}{-2 + 7 i} $
\item Mettre le complexe suivante sous forme exponentielle $z_2 = 1 - \sqrt{3} i$
\item Mettre le complexe suivante sous forme exponentielle $z_3 = 3 \sqrt{2} + 3 \sqrt{2} i$
\item Calculer le produit $z_4=z_2\times z_3$ donner le résultat sous forme exponentielle puis algébrique.
\item Calculer le quotient $z_5=\frac{z_2}{z_3}$ donner le résultat sous forme exponentielle puis algébrique.
\end{enumerate}
\end{exercise}
\begin{solution}
\begin{enumerate}
\item $z_1 = \frac{30}{53} - \frac{54 i}{53}$
\item $z_3 = 2 e^{- \frac{i \pi}{3}}$
\item $z_4 = 12 e^{- \frac{i \pi}{12}} = 3 \sqrt{2} + 3 \sqrt{6} + i \left(- 3 \sqrt{6} + 3 \sqrt{2}\right) = 11.6 - 3.11 i$
\item $z_5 = \frac{1}{3} e^{- \frac{7 i \pi}{12}} = - \frac{\sqrt{6}}{12} + \frac{\sqrt{2}}{12} + i \left(- \frac{\sqrt{6}}{12} - \frac{\sqrt{2}}{12}\right) = -0.0863 - 0.322 i$
\end{enumerate}
\end{solution}
\begin{exercise}[subtitle={Bassin}]
Le tour d'un bassin au niveau du sol présente deux axes de symétrie : laxe des abscisses et la droite déquation $x=4$. Il est obtenu par symétrie de la courbe $\mathcal{C_f}$ sur $\intFF{0}{4}$$f$ est la fonction définie par
\[
f(x) = \left(- x^{2} + 1.3 x - 8.0\right) e^{- x} + 8.0
\]
On admet que sur $\intFF{0}{4}$ la fonction $f$ est positive.
\begin{enumerate}
\item Sur un repère, tracer l'allure de la courbe $\mathcal{C}_f$, les axes de symétries puis compléter pour dessiner la forme du bassin.
\item Montrer que la fonction $f$ admet comme primitive sur $\R$ la fonction $F$ définie par
\[
F(x) = 8.0 x + \left( x^{2} + 0.7 x + 8.7\right) e^{- x}
\]
\item Calculer la quantité $\ds \int_0^4 f(x) \; dx$, vous donnerez le résultat sous forme exacte. Interpréter le résultat et reportez cette quantité sur le graphique.
\item On considère que l'échelle de votre graphique est de 1unité pour 15m. Calculer l'aire du bassin. Vous donnerez un résultat arrondi au $m^2$ près.
\end{enumerate}
\end{exercise}
\begin{solution}
\begin{enumerate}
\item
\begin{tikzpicture}[baseline=(a.north), xscale=1, yscale=0.5]
\tkzInit[xmin=0,xmax=5,xstep=1,
ymin=0,ymax=10,ystep=1]
\tkzGrid
\tkzAxeXY
\tkzFct[domain=0:10,color=red,very thick]%
{ (-x**2 + 1.3*x - 8.0)*exp(-x) + 8.0 };
\end{tikzpicture}
\item Il faut dériver $F(x)$ et vérifier que $F'(x) = f(x)$.
\item $\ds \int_0^4 f(x) \; dx = F(4) - F(0) = \frac{27.5}{e^{4}} + 23.3$
\item La quantité calculée à la question précédente se retrouve 4fois pour former le bassin. Il faut ensuite prendre en compte l'échelle, comme 1unité de longueur correspond à 15m, une unité d'air correspond à $15\times15 = 225m^2$. Ainsi l'aire du bassin est égale à
\[
(\frac{27.5}{e^{4}} + 23.3)\times 4 \times 15^2 = 21423.00000
\]
\end{enumerate}
\end{solution}
\begin{exercise}[subtitle={Bassin}]
Le clinker est un constituant du ciment qui résulte de la cuisson d'un mélange composé de calcaire et d'argile. La fabrication du clinker nécessite des fours à très haute température qui libèrent dans l'air une grande quantité de dioxyde de carbone (CO$_2$).
Dans une cimenterie, la fabrication du clinker s'effectue de 7 h 30 à 20 h, dans une pièce de volume \np{900000}~dm$^3$.
À 20 h, après une journée de travail, le taux volumique de CO$_2$ dans la pièce est de 0.7\,\%.
\begin{enumerate}
\item Justifier que le volume de CO$_2$ présent dans cette pièce à 20 h est de \np{6300}~dm$^3$ .
\item On modélise le volume de CO$_2$ présent dans la pièce par une fonction du temps $t$ écoulé après 20h (exprimé en minutes) qui pour formule $V(t) = V_0e^{-0.1t} + 390$
\begin{enumerate}
\item Démontrer que $V_0$ est égale à \np{5910}.
\item Quel sera, au dm$^3$ près, le volume de CO$_2$ dans cette pièce à 22 h ?
\item Démontrer que $V'(t) = - 591.0 e^{- 0.1 t}$.
\item Étudier le signe de $V'(t)$ puis en déduire le sens de variation de $V(t)$.
\item Que peut-on dire du volume de CO$_2$ quand $t$ devient grand?
\end{enumerate}
\end{enumerate}
\end{exercise}
\begin{solution}
\begin{enumerate}
\item Volume à 20h: $900000\times 0.006999999999999999 = 6300$
\item
\begin{enumerate}
\item $t=0$ correspond à 20h.
Donc $V(0) = 6300 = V_0e^{-0.1\times 0} + 390 = V_0 + 390$
Donc $V_0 = 6300 - 390 = 5910$
\item Il faut calculer $V(t)$ pour $t = 2$ donc
\[
V(2) = 5228.70
\]
\item Pas de correction pour cette question.
\item Pas de correction pour cette question.
\item Pas de correction pour cette question.
\end{enumerate}
\end{enumerate}
\end{solution}
\end{document}
%%% Local Variables:
%%% mode: latex
%%% TeX-master: "master"
%%% End:

View File

@ -0,0 +1,121 @@
\documentclass[a4paper,10pt]{article}
\usepackage{myXsim}
\usepackage{tasks}
% Title Page
\title{DM1 \hfill HUMBERT Rayan}
\tribe{TST sti2d}
\date{\hfillÀ render pour le jeudi 25 février}
\xsimsetup{
solution/print = true
}
\begin{document}
\maketitle
\begin{exercise}[subtitle={Complexes}]
\begin{enumerate}
\item Mettre le nombre complexe suivant sous forme algébrique $z_1 = \dfrac{5 + 10 i}{-5 + 2 i} $
\item Mettre le complexe suivante sous forme exponentielle $z_2 = - 5 \sqrt{3} - 5 i$
\item Mettre le complexe suivante sous forme exponentielle $z_3 = 9 - 9 \sqrt{3} i$
\item Calculer le produit $z_4=z_2\times z_3$ donner le résultat sous forme exponentielle puis algébrique.
\item Calculer le quotient $z_5=\frac{z_2}{z_3}$ donner le résultat sous forme exponentielle puis algébrique.
\end{enumerate}
\end{exercise}
\begin{solution}
\begin{enumerate}
\item $z_1 = - \frac{5}{29} - \frac{60 i}{29}$
\item $z_3 = 10 e^{- \frac{5 i \pi}{6}}$
\item $z_4 = 180 e^{- \frac{7 i \pi}{6}} = - 90 \sqrt{3} + 90 i = -156.0 + 90.0 i$
\item $z_5 = \frac{5}{9} e^{- \frac{i \pi}{2}} = - \frac{5 i}{9} = - 0.556 i$
\end{enumerate}
\end{solution}
\begin{exercise}[subtitle={Bassin}]
Le tour d'un bassin au niveau du sol présente deux axes de symétrie : laxe des abscisses et la droite déquation $x=4$. Il est obtenu par symétrie de la courbe $\mathcal{C_f}$ sur $\intFF{0}{4}$$f$ est la fonction définie par
\[
f(x) = \left(- x^{2} + 0.3 x - 9.7\right) e^{- x} + 9.7
\]
On admet que sur $\intFF{0}{4}$ la fonction $f$ est positive.
\begin{enumerate}
\item Sur un repère, tracer l'allure de la courbe $\mathcal{C}_f$, les axes de symétries puis compléter pour dessiner la forme du bassin.
\item Montrer que la fonction $f$ admet comme primitive sur $\R$ la fonction $F$ définie par
\[
F(x) = 9.7 x + \left( x^{2} + 1.7 x + 11.4\right) e^{- x}
\]
\item Calculer la quantité $\ds \int_0^4 f(x) \; dx$, vous donnerez le résultat sous forme exacte. Interpréter le résultat et reportez cette quantité sur le graphique.
\item On considère que l'échelle de votre graphique est de 1unité pour 15m. Calculer l'aire du bassin. Vous donnerez un résultat arrondi au $m^2$ près.
\end{enumerate}
\end{exercise}
\begin{solution}
\begin{enumerate}
\item
\begin{tikzpicture}[baseline=(a.north), xscale=1, yscale=0.5]
\tkzInit[xmin=0,xmax=5,xstep=1,
ymin=0,ymax=10,ystep=1]
\tkzGrid
\tkzAxeXY
\tkzFct[domain=0:10,color=red,very thick]%
{ (-x**2 + 0.3*x - 9.7)*exp(-x) + 9.7 };
\end{tikzpicture}
\item Il faut dériver $F(x)$ et vérifier que $F'(x) = f(x)$.
\item $\ds \int_0^4 f(x) \; dx = F(4) - F(0) = \frac{34.2}{e^{4}} + 27.4$
\item La quantité calculée à la question précédente se retrouve 4fois pour former le bassin. Il faut ensuite prendre en compte l'échelle, comme 1unité de longueur correspond à 15m, une unité d'air correspond à $15\times15 = 225m^2$. Ainsi l'aire du bassin est égale à
\[
(\frac{34.2}{e^{4}} + 27.4)\times 4 \times 15^2 = 25224.00000
\]
\end{enumerate}
\end{solution}
\begin{exercise}[subtitle={Bassin}]
Le clinker est un constituant du ciment qui résulte de la cuisson d'un mélange composé de calcaire et d'argile. La fabrication du clinker nécessite des fours à très haute température qui libèrent dans l'air une grande quantité de dioxyde de carbone (CO$_2$).
Dans une cimenterie, la fabrication du clinker s'effectue de 7 h 30 à 20 h, dans une pièce de volume \np{600000}~dm$^3$.
À 20 h, après une journée de travail, le taux volumique de CO$_2$ dans la pièce est de 0.6\,\%.
\begin{enumerate}
\item Justifier que le volume de CO$_2$ présent dans cette pièce à 20 h est de \np{3600}~dm$^3$ .
\item On modélise le volume de CO$_2$ présent dans la pièce par une fonction du temps $t$ écoulé après 20h (exprimé en minutes) qui pour formule $V(t) = V_0e^{-0.06t} + 360$
\begin{enumerate}
\item Démontrer que $V_0$ est égale à \np{3240}.
\item Quel sera, au dm$^3$ près, le volume de CO$_2$ dans cette pièce à 24 h ?
\item Démontrer que $V'(t) = - 194.4 e^{- 0.06 t}$.
\item Étudier le signe de $V'(t)$ puis en déduire le sens de variation de $V(t)$.
\item Que peut-on dire du volume de CO$_2$ quand $t$ devient grand?
\end{enumerate}
\end{enumerate}
\end{exercise}
\begin{solution}
\begin{enumerate}
\item Volume à 20h: $600000\times 0.006 = 3600$
\item
\begin{enumerate}
\item $t=0$ correspond à 20h.
Donc $V(0) = 3600 = V_0e^{-0.06\times 0} + 360 = V_0 + 360$
Donc $V_0 = 3600 - 360 = 3240$
\item Il faut calculer $V(t)$ pour $t = 4$ donc
\[
V(4) = 2908.67
\]
\item Pas de correction pour cette question.
\item Pas de correction pour cette question.
\item Pas de correction pour cette question.
\end{enumerate}
\end{enumerate}
\end{solution}
\end{document}
%%% Local Variables:
%%% mode: latex
%%% TeX-master: "master"
%%% End:

View File

@ -0,0 +1,121 @@
\documentclass[a4paper,10pt]{article}
\usepackage{myXsim}
\usepackage{tasks}
% Title Page
\title{DM1 \hfill KILINC Suleyman}
\tribe{TST sti2d}
\date{\hfillÀ render pour le jeudi 25 février}
\xsimsetup{
solution/print = true
}
\begin{document}
\maketitle
\begin{exercise}[subtitle={Complexes}]
\begin{enumerate}
\item Mettre le nombre complexe suivant sous forme algébrique $z_1 = \dfrac{2 + 7 i}{-8 + 6 i} $
\item Mettre le complexe suivante sous forme exponentielle $z_2 = \sqrt{2} + \sqrt{2} i$
\item Mettre le complexe suivante sous forme exponentielle $z_3 = 8 \sqrt{3} - 8 i$
\item Calculer le produit $z_4=z_2\times z_3$ donner le résultat sous forme exponentielle puis algébrique.
\item Calculer le quotient $z_5=\frac{z_2}{z_3}$ donner le résultat sous forme exponentielle puis algébrique.
\end{enumerate}
\end{exercise}
\begin{solution}
\begin{enumerate}
\item $z_1 = \frac{13}{50} - \frac{17 i}{25}$
\item $z_3 = 2 e^{\frac{i \pi}{4}}$
\item $z_4 = 32 e^{\frac{i \pi}{12}} = 8 \sqrt{2} + 8 \sqrt{6} + i \left(- 8 \sqrt{2} + 8 \sqrt{6}\right) = 30.9 + 8.28 i$
\item $z_5 = \frac{1}{8} e^{\frac{5 i \pi}{12}} = - \frac{\sqrt{2}}{32} + \frac{\sqrt{6}}{32} + i \left(\frac{\sqrt{2}}{32} + \frac{\sqrt{6}}{32}\right) = 0.0323 + 0.121 i$
\end{enumerate}
\end{solution}
\begin{exercise}[subtitle={Bassin}]
Le tour d'un bassin au niveau du sol présente deux axes de symétrie : laxe des abscisses et la droite déquation $x=4$. Il est obtenu par symétrie de la courbe $\mathcal{C_f}$ sur $\intFF{0}{4}$$f$ est la fonction définie par
\[
f(x) = \left(- x^{2} + 5.8 x - 4.6\right) e^{- x} + 4.6
\]
On admet que sur $\intFF{0}{4}$ la fonction $f$ est positive.
\begin{enumerate}
\item Sur un repère, tracer l'allure de la courbe $\mathcal{C}_f$, les axes de symétries puis compléter pour dessiner la forme du bassin.
\item Montrer que la fonction $f$ admet comme primitive sur $\R$ la fonction $F$ définie par
\[
F(x) = 4.6 x + \left( x^{2} - 3.8 x + 0.8\right) e^{- x}
\]
\item Calculer la quantité $\ds \int_0^4 f(x) \; dx$, vous donnerez le résultat sous forme exacte. Interpréter le résultat et reportez cette quantité sur le graphique.
\item On considère que l'échelle de votre graphique est de 1unité pour 15m. Calculer l'aire du bassin. Vous donnerez un résultat arrondi au $m^2$ près.
\end{enumerate}
\end{exercise}
\begin{solution}
\begin{enumerate}
\item
\begin{tikzpicture}[baseline=(a.north), xscale=1, yscale=0.5]
\tkzInit[xmin=0,xmax=5,xstep=1,
ymin=0,ymax=10,ystep=1]
\tkzGrid
\tkzAxeXY
\tkzFct[domain=0:10,color=red,very thick]%
{ (-x**2 + 5.8*x - 4.6)*exp(-x) + 4.6 };
\end{tikzpicture}
\item Il faut dériver $F(x)$ et vérifier que $F'(x) = f(x)$.
\item $\ds \int_0^4 f(x) \; dx = F(4) - F(0) = \frac{1.6}{e^{4}} + 17.6$
\item La quantité calculée à la question précédente se retrouve 4fois pour former le bassin. Il faut ensuite prendre en compte l'échelle, comme 1unité de longueur correspond à 15m, une unité d'air correspond à $15\times15 = 225m^2$. Ainsi l'aire du bassin est égale à
\[
(\frac{1.6}{e^{4}} + 17.6)\times 4 \times 15^2 = 15866.00000
\]
\end{enumerate}
\end{solution}
\begin{exercise}[subtitle={Bassin}]
Le clinker est un constituant du ciment qui résulte de la cuisson d'un mélange composé de calcaire et d'argile. La fabrication du clinker nécessite des fours à très haute température qui libèrent dans l'air une grande quantité de dioxyde de carbone (CO$_2$).
Dans une cimenterie, la fabrication du clinker s'effectue de 7 h 30 à 20 h, dans une pièce de volume \np{700000}~dm$^3$.
À 20 h, après une journée de travail, le taux volumique de CO$_2$ dans la pièce est de 1.0\,\%.
\begin{enumerate}
\item Justifier que le volume de CO$_2$ présent dans cette pièce à 20 h est de \np{7000}~dm$^3$ .
\item On modélise le volume de CO$_2$ présent dans la pièce par une fonction du temps $t$ écoulé après 20h (exprimé en minutes) qui pour formule $V(t) = V_0e^{-0.07t} + 520$
\begin{enumerate}
\item Démontrer que $V_0$ est égale à \np{6480}.
\item Quel sera, au dm$^3$ près, le volume de CO$_2$ dans cette pièce à 24 h ?
\item Démontrer que $V'(t) = - 453.6 e^{- 0.07 t}$.
\item Étudier le signe de $V'(t)$ puis en déduire le sens de variation de $V(t)$.
\item Que peut-on dire du volume de CO$_2$ quand $t$ devient grand?
\end{enumerate}
\end{enumerate}
\end{exercise}
\begin{solution}
\begin{enumerate}
\item Volume à 20h: $700000\times 0.01 = 7000$
\item
\begin{enumerate}
\item $t=0$ correspond à 20h.
Donc $V(0) = 7000 = V_0e^{-0.07\times 0} + 520 = V_0 + 520$
Donc $V_0 = 7000 - 520 = 6480$
\item Il faut calculer $V(t)$ pour $t = 4$ donc
\[
V(4) = 5417.48
\]
\item Pas de correction pour cette question.
\item Pas de correction pour cette question.
\item Pas de correction pour cette question.
\end{enumerate}
\end{enumerate}
\end{solution}
\end{document}
%%% Local Variables:
%%% mode: latex
%%% TeX-master: "master"
%%% End:

View File

@ -0,0 +1,121 @@
\documentclass[a4paper,10pt]{article}
\usepackage{myXsim}
\usepackage{tasks}
% Title Page
\title{DM1 \hfill M'BAREK HASNAOUI Bilal}
\tribe{TST sti2d}
\date{\hfillÀ render pour le jeudi 25 février}
\xsimsetup{
solution/print = true
}
\begin{document}
\maketitle
\begin{exercise}[subtitle={Complexes}]
\begin{enumerate}
\item Mettre le nombre complexe suivant sous forme algébrique $z_1 = \dfrac{4 + 7 i}{-10 + 7 i} $
\item Mettre le complexe suivante sous forme exponentielle $z_2 = 6 - 6 \sqrt{3} i$
\item Mettre le complexe suivante sous forme exponentielle $z_3 = 6 \sqrt{2} + 6 \sqrt{2} i$
\item Calculer le produit $z_4=z_2\times z_3$ donner le résultat sous forme exponentielle puis algébrique.
\item Calculer le quotient $z_5=\frac{z_2}{z_3}$ donner le résultat sous forme exponentielle puis algébrique.
\end{enumerate}
\end{exercise}
\begin{solution}
\begin{enumerate}
\item $z_1 = \frac{9}{149} - \frac{98 i}{149}$
\item $z_3 = 12 e^{- \frac{i \pi}{3}}$
\item $z_4 = 144 e^{- \frac{i \pi}{12}} = 36 \sqrt{2} + 36 \sqrt{6} + i \left(- 36 \sqrt{6} + 36 \sqrt{2}\right) = 139.0 - 37.3 i$
\item $z_5 = 1 e^{- \frac{7 i \pi}{12}} = - \frac{\sqrt{6}}{4} + \frac{\sqrt{2}}{4} + i \left(- \frac{\sqrt{6}}{4} - \frac{\sqrt{2}}{4}\right) = -0.259 - 0.966 i$
\end{enumerate}
\end{solution}
\begin{exercise}[subtitle={Bassin}]
Le tour d'un bassin au niveau du sol présente deux axes de symétrie : laxe des abscisses et la droite déquation $x=4$. Il est obtenu par symétrie de la courbe $\mathcal{C_f}$ sur $\intFF{0}{4}$$f$ est la fonction définie par
\[
f(x) = \left(- x^{2} + 2.2 x - 9.7\right) e^{- x} + 9.7
\]
On admet que sur $\intFF{0}{4}$ la fonction $f$ est positive.
\begin{enumerate}
\item Sur un repère, tracer l'allure de la courbe $\mathcal{C}_f$, les axes de symétries puis compléter pour dessiner la forme du bassin.
\item Montrer que la fonction $f$ admet comme primitive sur $\R$ la fonction $F$ définie par
\[
F(x) = 9.7 x + \left( x^{2} - 0.2 x + 9.5\right) e^{- x}
\]
\item Calculer la quantité $\ds \int_0^4 f(x) \; dx$, vous donnerez le résultat sous forme exacte. Interpréter le résultat et reportez cette quantité sur le graphique.
\item On considère que l'échelle de votre graphique est de 1unité pour 15m. Calculer l'aire du bassin. Vous donnerez un résultat arrondi au $m^2$ près.
\end{enumerate}
\end{exercise}
\begin{solution}
\begin{enumerate}
\item
\begin{tikzpicture}[baseline=(a.north), xscale=1, yscale=0.5]
\tkzInit[xmin=0,xmax=5,xstep=1,
ymin=0,ymax=10,ystep=1]
\tkzGrid
\tkzAxeXY
\tkzFct[domain=0:10,color=red,very thick]%
{ (-x**2 + 2.2*x - 9.7)*exp(-x) + 9.7 };
\end{tikzpicture}
\item Il faut dériver $F(x)$ et vérifier que $F'(x) = f(x)$.
\item $\ds \int_0^4 f(x) \; dx = F(4) - F(0) = \frac{24.7}{e^{4}} + 29.3$
\item La quantité calculée à la question précédente se retrouve 4fois pour former le bassin. Il faut ensuite prendre en compte l'échelle, comme 1unité de longueur correspond à 15m, une unité d'air correspond à $15\times15 = 225m^2$. Ainsi l'aire du bassin est égale à
\[
(\frac{24.7}{e^{4}} + 29.3)\times 4 \times 15^2 = 26777.00000
\]
\end{enumerate}
\end{solution}
\begin{exercise}[subtitle={Bassin}]
Le clinker est un constituant du ciment qui résulte de la cuisson d'un mélange composé de calcaire et d'argile. La fabrication du clinker nécessite des fours à très haute température qui libèrent dans l'air une grande quantité de dioxyde de carbone (CO$_2$).
Dans une cimenterie, la fabrication du clinker s'effectue de 7 h 30 à 20 h, dans une pièce de volume \np{300000}~dm$^3$.
À 20 h, après une journée de travail, le taux volumique de CO$_2$ dans la pièce est de 0.6\,\%.
\begin{enumerate}
\item Justifier que le volume de CO$_2$ présent dans cette pièce à 20 h est de \np{1800}~dm$^3$ .
\item On modélise le volume de CO$_2$ présent dans la pièce par une fonction du temps $t$ écoulé après 20h (exprimé en minutes) qui pour formule $V(t) = V_0e^{-0.1t} + 430$
\begin{enumerate}
\item Démontrer que $V_0$ est égale à \np{1370}.
\item Quel sera, au dm$^3$ près, le volume de CO$_2$ dans cette pièce à 22 h ?
\item Démontrer que $V'(t) = - 137.0 e^{- 0.1 t}$.
\item Étudier le signe de $V'(t)$ puis en déduire le sens de variation de $V(t)$.
\item Que peut-on dire du volume de CO$_2$ quand $t$ devient grand?
\end{enumerate}
\end{enumerate}
\end{exercise}
\begin{solution}
\begin{enumerate}
\item Volume à 20h: $300000\times 0.006 = 1800$
\item
\begin{enumerate}
\item $t=0$ correspond à 20h.
Donc $V(0) = 1800 = V_0e^{-0.1\times 0} + 430 = V_0 + 430$
Donc $V_0 = 1800 - 430 = 1370$
\item Il faut calculer $V(t)$ pour $t = 2$ donc
\[
V(2) = 1551.66
\]
\item Pas de correction pour cette question.
\item Pas de correction pour cette question.
\item Pas de correction pour cette question.
\end{enumerate}
\end{enumerate}
\end{solution}
\end{document}
%%% Local Variables:
%%% mode: latex
%%% TeX-master: "master"
%%% End:

View File

@ -0,0 +1,121 @@
\documentclass[a4paper,10pt]{article}
\usepackage{myXsim}
\usepackage{tasks}
% Title Page
\title{DM1 \hfill MERCIER Almandin}
\tribe{TST sti2d}
\date{\hfillÀ render pour le jeudi 25 février}
\xsimsetup{
solution/print = true
}
\begin{document}
\maketitle
\begin{exercise}[subtitle={Complexes}]
\begin{enumerate}
\item Mettre le nombre complexe suivant sous forme algébrique $z_1 = \dfrac{2 + 4 i}{-6 + 6 i} $
\item Mettre le complexe suivante sous forme exponentielle $z_2 = 4 \sqrt{3} - 4 i$
\item Mettre le complexe suivante sous forme exponentielle $z_3 = - 10 \sqrt{2} + 10 \sqrt{2} i$
\item Calculer le produit $z_4=z_2\times z_3$ donner le résultat sous forme exponentielle puis algébrique.
\item Calculer le quotient $z_5=\frac{z_2}{z_3}$ donner le résultat sous forme exponentielle puis algébrique.
\end{enumerate}
\end{exercise}
\begin{solution}
\begin{enumerate}
\item $z_1 = \frac{1}{6} - \frac{i}{2}$
\item $z_3 = 8 e^{- \frac{i \pi}{6}}$
\item $z_4 = 160 e^{\frac{7 i \pi}{12}} = - 40 \sqrt{6} + 40 \sqrt{2} + i \left(40 \sqrt{2} + 40 \sqrt{6}\right) = -41.4 + 155.0 i$
\item $z_5 = \frac{2}{5} e^{- \frac{11 i \pi}{12}} = - \frac{\sqrt{6}}{10} - \frac{\sqrt{2}}{10} + i \left(- \frac{\sqrt{6}}{10} + \frac{\sqrt{2}}{10}\right) = -0.386 - 0.104 i$
\end{enumerate}
\end{solution}
\begin{exercise}[subtitle={Bassin}]
Le tour d'un bassin au niveau du sol présente deux axes de symétrie : laxe des abscisses et la droite déquation $x=4$. Il est obtenu par symétrie de la courbe $\mathcal{C_f}$ sur $\intFF{0}{4}$$f$ est la fonction définie par
\[
f(x) = \left(- x^{2} + 1.4 x - 4.9\right) e^{- x} + 4.9
\]
On admet que sur $\intFF{0}{4}$ la fonction $f$ est positive.
\begin{enumerate}
\item Sur un repère, tracer l'allure de la courbe $\mathcal{C}_f$, les axes de symétries puis compléter pour dessiner la forme du bassin.
\item Montrer que la fonction $f$ admet comme primitive sur $\R$ la fonction $F$ définie par
\[
F(x) = 4.9 x + \left( x^{2} + 0.6 x + 5.5\right) e^{- x}
\]
\item Calculer la quantité $\ds \int_0^4 f(x) \; dx$, vous donnerez le résultat sous forme exacte. Interpréter le résultat et reportez cette quantité sur le graphique.
\item On considère que l'échelle de votre graphique est de 1unité pour 15m. Calculer l'aire du bassin. Vous donnerez un résultat arrondi au $m^2$ près.
\end{enumerate}
\end{exercise}
\begin{solution}
\begin{enumerate}
\item
\begin{tikzpicture}[baseline=(a.north), xscale=1, yscale=0.5]
\tkzInit[xmin=0,xmax=5,xstep=1,
ymin=0,ymax=10,ystep=1]
\tkzGrid
\tkzAxeXY
\tkzFct[domain=0:10,color=red,very thick]%
{ (-x**2 + 1.4*x - 4.9)*exp(-x) + 4.9 };
\end{tikzpicture}
\item Il faut dériver $F(x)$ et vérifier que $F'(x) = f(x)$.
\item $\ds \int_0^4 f(x) \; dx = F(4) - F(0) = \frac{23.9}{e^{4}} + 14.1$
\item La quantité calculée à la question précédente se retrouve 4fois pour former le bassin. Il faut ensuite prendre en compte l'échelle, comme 1unité de longueur correspond à 15m, une unité d'air correspond à $15\times15 = 225m^2$. Ainsi l'aire du bassin est égale à
\[
(\frac{23.9}{e^{4}} + 14.1)\times 4 \times 15^2 = 13084.00000
\]
\end{enumerate}
\end{solution}
\begin{exercise}[subtitle={Bassin}]
Le clinker est un constituant du ciment qui résulte de la cuisson d'un mélange composé de calcaire et d'argile. La fabrication du clinker nécessite des fours à très haute température qui libèrent dans l'air une grande quantité de dioxyde de carbone (CO$_2$).
Dans une cimenterie, la fabrication du clinker s'effectue de 7 h 30 à 20 h, dans une pièce de volume \np{400000}~dm$^3$.
À 20 h, après une journée de travail, le taux volumique de CO$_2$ dans la pièce est de 0.7\,\%.
\begin{enumerate}
\item Justifier que le volume de CO$_2$ présent dans cette pièce à 20 h est de \np{2800}~dm$^3$ .
\item On modélise le volume de CO$_2$ présent dans la pièce par une fonction du temps $t$ écoulé après 20h (exprimé en minutes) qui pour formule $V(t) = V_0e^{-0.07t} + 360$
\begin{enumerate}
\item Démontrer que $V_0$ est égale à \np{2440}.
\item Quel sera, au dm$^3$ près, le volume de CO$_2$ dans cette pièce à 23 h ?
\item Démontrer que $V'(t) = - 170.8 e^{- 0.07 t}$.
\item Étudier le signe de $V'(t)$ puis en déduire le sens de variation de $V(t)$.
\item Que peut-on dire du volume de CO$_2$ quand $t$ devient grand?
\end{enumerate}
\end{enumerate}
\end{exercise}
\begin{solution}
\begin{enumerate}
\item Volume à 20h: $400000\times 0.006999999999999999 = 2800$
\item
\begin{enumerate}
\item $t=0$ correspond à 20h.
Donc $V(0) = 2800 = V_0e^{-0.07\times 0} + 360 = V_0 + 360$
Donc $V_0 = 2800 - 360 = 2440$
\item Il faut calculer $V(t)$ pour $t = 3$ donc
\[
V(3) = 2337.83
\]
\item Pas de correction pour cette question.
\item Pas de correction pour cette question.
\item Pas de correction pour cette question.
\end{enumerate}
\end{enumerate}
\end{solution}
\end{document}
%%% Local Variables:
%%% mode: latex
%%% TeX-master: "master"
%%% End:

View File

@ -0,0 +1,121 @@
\documentclass[a4paper,10pt]{article}
\usepackage{myXsim}
\usepackage{tasks}
% Title Page
\title{DM1 \hfill MOUFAQ Amine}
\tribe{TST sti2d}
\date{\hfillÀ render pour le jeudi 25 février}
\xsimsetup{
solution/print = true
}
\begin{document}
\maketitle
\begin{exercise}[subtitle={Complexes}]
\begin{enumerate}
\item Mettre le nombre complexe suivant sous forme algébrique $z_1 = \dfrac{6 + 9 i}{-9 + 10 i} $
\item Mettre le complexe suivante sous forme exponentielle $z_2 = 10 \sqrt{3} + 10 i$
\item Mettre le complexe suivante sous forme exponentielle $z_3 = 9 \sqrt{2} + 9 \sqrt{2} i$
\item Calculer le produit $z_4=z_2\times z_3$ donner le résultat sous forme exponentielle puis algébrique.
\item Calculer le quotient $z_5=\frac{z_2}{z_3}$ donner le résultat sous forme exponentielle puis algébrique.
\end{enumerate}
\end{exercise}
\begin{solution}
\begin{enumerate}
\item $z_1 = \frac{36}{181} - \frac{141 i}{181}$
\item $z_3 = 20 e^{\frac{i \pi}{6}}$
\item $z_4 = 360 e^{\frac{5 i \pi}{12}} = - 90 \sqrt{2} + 90 \sqrt{6} + i \left(90 \sqrt{2} + 90 \sqrt{6}\right) = 93.2 + 348.0 i$
\item $z_5 = \frac{10}{9} e^{- \frac{i \pi}{12}} = \frac{5 \sqrt{2}}{18} + \frac{5 \sqrt{6}}{18} + i \left(- \frac{5 \sqrt{6}}{18} + \frac{5 \sqrt{2}}{18}\right) = 1.07 - 0.288 i$
\end{enumerate}
\end{solution}
\begin{exercise}[subtitle={Bassin}]
Le tour d'un bassin au niveau du sol présente deux axes de symétrie : laxe des abscisses et la droite déquation $x=4$. Il est obtenu par symétrie de la courbe $\mathcal{C_f}$ sur $\intFF{0}{4}$$f$ est la fonction définie par
\[
f(x) = \left(- x^{2} + 5.5 x - 9.8\right) e^{- x} + 9.8
\]
On admet que sur $\intFF{0}{4}$ la fonction $f$ est positive.
\begin{enumerate}
\item Sur un repère, tracer l'allure de la courbe $\mathcal{C}_f$, les axes de symétries puis compléter pour dessiner la forme du bassin.
\item Montrer que la fonction $f$ admet comme primitive sur $\R$ la fonction $F$ définie par
\[
F(x) = 9.8 x + \left( x^{2} - 3.5 x + 6.3\right) e^{- x}
\]
\item Calculer la quantité $\ds \int_0^4 f(x) \; dx$, vous donnerez le résultat sous forme exacte. Interpréter le résultat et reportez cette quantité sur le graphique.
\item On considère que l'échelle de votre graphique est de 1unité pour 15m. Calculer l'aire du bassin. Vous donnerez un résultat arrondi au $m^2$ près.
\end{enumerate}
\end{exercise}
\begin{solution}
\begin{enumerate}
\item
\begin{tikzpicture}[baseline=(a.north), xscale=1, yscale=0.5]
\tkzInit[xmin=0,xmax=5,xstep=1,
ymin=0,ymax=10,ystep=1]
\tkzGrid
\tkzAxeXY
\tkzFct[domain=0:10,color=red,very thick]%
{ (-x**2 + 5.5*x - 9.8)*exp(-x) + 9.8 };
\end{tikzpicture}
\item Il faut dériver $F(x)$ et vérifier que $F'(x) = f(x)$.
\item $\ds \int_0^4 f(x) \; dx = F(4) - F(0) = \frac{8.3}{e^{4}} + 32.9$
\item La quantité calculée à la question précédente se retrouve 4fois pour former le bassin. Il faut ensuite prendre en compte l'échelle, comme 1unité de longueur correspond à 15m, une unité d'air correspond à $15\times15 = 225m^2$. Ainsi l'aire du bassin est égale à
\[
(\frac{8.3}{e^{4}} + 32.9)\times 4 \times 15^2 = 29747.00000
\]
\end{enumerate}
\end{solution}
\begin{exercise}[subtitle={Bassin}]
Le clinker est un constituant du ciment qui résulte de la cuisson d'un mélange composé de calcaire et d'argile. La fabrication du clinker nécessite des fours à très haute température qui libèrent dans l'air une grande quantité de dioxyde de carbone (CO$_2$).
Dans une cimenterie, la fabrication du clinker s'effectue de 7 h 30 à 20 h, dans une pièce de volume \np{600000}~dm$^3$.
À 20 h, après une journée de travail, le taux volumique de CO$_2$ dans la pièce est de 0.9\,\%.
\begin{enumerate}
\item Justifier que le volume de CO$_2$ présent dans cette pièce à 20 h est de \np{5400}~dm$^3$ .
\item On modélise le volume de CO$_2$ présent dans la pièce par une fonction du temps $t$ écoulé après 20h (exprimé en minutes) qui pour formule $V(t) = V_0e^{-0.06t} + 340$
\begin{enumerate}
\item Démontrer que $V_0$ est égale à \np{5060}.
\item Quel sera, au dm$^3$ près, le volume de CO$_2$ dans cette pièce à 24 h ?
\item Démontrer que $V'(t) = - 303.6 e^{- 0.06 t}$.
\item Étudier le signe de $V'(t)$ puis en déduire le sens de variation de $V(t)$.
\item Que peut-on dire du volume de CO$_2$ quand $t$ devient grand?
\end{enumerate}
\end{enumerate}
\end{exercise}
\begin{solution}
\begin{enumerate}
\item Volume à 20h: $600000\times 0.009000000000000001 = 5400$
\item
\begin{enumerate}
\item $t=0$ correspond à 20h.
Donc $V(0) = 5400 = V_0e^{-0.06\times 0} + 340 = V_0 + 340$
Donc $V_0 = 5400 - 340 = 5060$
\item Il faut calculer $V(t)$ pour $t = 4$ donc
\[
V(4) = 4320.34
\]
\item Pas de correction pour cette question.
\item Pas de correction pour cette question.
\item Pas de correction pour cette question.
\end{enumerate}
\end{enumerate}
\end{solution}
\end{document}
%%% Local Variables:
%%% mode: latex
%%% TeX-master: "master"
%%% End:

View File

@ -0,0 +1,121 @@
\documentclass[a4paper,10pt]{article}
\usepackage{myXsim}
\usepackage{tasks}
% Title Page
\title{DM1 \hfill NARDINI Kakary}
\tribe{TST sti2d}
\date{\hfillÀ render pour le jeudi 25 février}
\xsimsetup{
solution/print = true
}
\begin{document}
\maketitle
\begin{exercise}[subtitle={Complexes}]
\begin{enumerate}
\item Mettre le nombre complexe suivant sous forme algébrique $z_1 = \dfrac{7 + 5 i}{-4 + 5 i} $
\item Mettre le complexe suivante sous forme exponentielle $z_2 = - 8 \sqrt{2} + 8 \sqrt{2} i$
\item Mettre le complexe suivante sous forme exponentielle $z_3 = \sqrt{3} + i$
\item Calculer le produit $z_4=z_2\times z_3$ donner le résultat sous forme exponentielle puis algébrique.
\item Calculer le quotient $z_5=\frac{z_2}{z_3}$ donner le résultat sous forme exponentielle puis algébrique.
\end{enumerate}
\end{exercise}
\begin{solution}
\begin{enumerate}
\item $z_1 = - \frac{3}{41} - \frac{55 i}{41}$
\item $z_3 = 16 e^{\frac{3 i \pi}{4}}$
\item $z_4 = 32 e^{\frac{11 i \pi}{12}} = - 8 \sqrt{6} - 8 \sqrt{2} + i \left(- 8 \sqrt{2} + 8 \sqrt{6}\right) = -30.9 + 8.28 i$
\item $z_5 = 8 e^{\frac{7 i \pi}{12}} = - 2 \sqrt{6} + 2 \sqrt{2} + i \left(2 \sqrt{2} + 2 \sqrt{6}\right) = -2.07 + 7.73 i$
\end{enumerate}
\end{solution}
\begin{exercise}[subtitle={Bassin}]
Le tour d'un bassin au niveau du sol présente deux axes de symétrie : laxe des abscisses et la droite déquation $x=4$. Il est obtenu par symétrie de la courbe $\mathcal{C_f}$ sur $\intFF{0}{4}$$f$ est la fonction définie par
\[
f(x) = \left(- x^{2} + 2.3 x - 1.7\right) e^{- x} + 1.7
\]
On admet que sur $\intFF{0}{4}$ la fonction $f$ est positive.
\begin{enumerate}
\item Sur un repère, tracer l'allure de la courbe $\mathcal{C}_f$, les axes de symétries puis compléter pour dessiner la forme du bassin.
\item Montrer que la fonction $f$ admet comme primitive sur $\R$ la fonction $F$ définie par
\[
F(x) = 1.7 x + \left( x^{2} - 0.3 x + 1.4\right) e^{- x}
\]
\item Calculer la quantité $\ds \int_0^4 f(x) \; dx$, vous donnerez le résultat sous forme exacte. Interpréter le résultat et reportez cette quantité sur le graphique.
\item On considère que l'échelle de votre graphique est de 1unité pour 15m. Calculer l'aire du bassin. Vous donnerez un résultat arrondi au $m^2$ près.
\end{enumerate}
\end{exercise}
\begin{solution}
\begin{enumerate}
\item
\begin{tikzpicture}[baseline=(a.north), xscale=1, yscale=0.5]
\tkzInit[xmin=0,xmax=5,xstep=1,
ymin=0,ymax=10,ystep=1]
\tkzGrid
\tkzAxeXY
\tkzFct[domain=0:10,color=red,very thick]%
{ (-x**2 + 2.3*x - 1.7)*exp(-x) + 1.7 };
\end{tikzpicture}
\item Il faut dériver $F(x)$ et vérifier que $F'(x) = f(x)$.
\item $\ds \int_0^4 f(x) \; dx = F(4) - F(0) = \frac{16.2}{e^{4}} + 5.4$
\item La quantité calculée à la question précédente se retrouve 4fois pour former le bassin. Il faut ensuite prendre en compte l'échelle, comme 1unité de longueur correspond à 15m, une unité d'air correspond à $15\times15 = 225m^2$. Ainsi l'aire du bassin est égale à
\[
(\frac{16.2}{e^{4}} + 5.4)\times 4 \times 15^2 = 5127.000000
\]
\end{enumerate}
\end{solution}
\begin{exercise}[subtitle={Bassin}]
Le clinker est un constituant du ciment qui résulte de la cuisson d'un mélange composé de calcaire et d'argile. La fabrication du clinker nécessite des fours à très haute température qui libèrent dans l'air une grande quantité de dioxyde de carbone (CO$_2$).
Dans une cimenterie, la fabrication du clinker s'effectue de 7 h 30 à 20 h, dans une pièce de volume \np{600000}~dm$^3$.
À 20 h, après une journée de travail, le taux volumique de CO$_2$ dans la pièce est de 0.9\,\%.
\begin{enumerate}
\item Justifier que le volume de CO$_2$ présent dans cette pièce à 20 h est de \np{5400}~dm$^3$ .
\item On modélise le volume de CO$_2$ présent dans la pièce par une fonction du temps $t$ écoulé après 20h (exprimé en minutes) qui pour formule $V(t) = V_0e^{-0.09t} + 360$
\begin{enumerate}
\item Démontrer que $V_0$ est égale à \np{5040}.
\item Quel sera, au dm$^3$ près, le volume de CO$_2$ dans cette pièce à 22 h ?
\item Démontrer que $V'(t) = - 453.6 e^{- 0.09 t}$.
\item Étudier le signe de $V'(t)$ puis en déduire le sens de variation de $V(t)$.
\item Que peut-on dire du volume de CO$_2$ quand $t$ devient grand?
\end{enumerate}
\end{enumerate}
\end{exercise}
\begin{solution}
\begin{enumerate}
\item Volume à 20h: $600000\times 0.009000000000000001 = 5400$
\item
\begin{enumerate}
\item $t=0$ correspond à 20h.
Donc $V(0) = 5400 = V_0e^{-0.09\times 0} + 360 = V_0 + 360$
Donc $V_0 = 5400 - 360 = 5040$
\item Il faut calculer $V(t)$ pour $t = 2$ donc
\[
V(2) = 4569.76
\]
\item Pas de correction pour cette question.
\item Pas de correction pour cette question.
\item Pas de correction pour cette question.
\end{enumerate}
\end{enumerate}
\end{solution}
\end{document}
%%% Local Variables:
%%% mode: latex
%%% TeX-master: "master"
%%% End:

View File

@ -0,0 +1,121 @@
\documentclass[a4paper,10pt]{article}
\usepackage{myXsim}
\usepackage{tasks}
% Title Page
\title{DM1 \hfill ONAL Yakub}
\tribe{TST sti2d}
\date{\hfillÀ render pour le jeudi 25 février}
\xsimsetup{
solution/print = true
}
\begin{document}
\maketitle
\begin{exercise}[subtitle={Complexes}]
\begin{enumerate}
\item Mettre le nombre complexe suivant sous forme algébrique $z_1 = \dfrac{8 + 7 i}{-5 + 6 i} $
\item Mettre le complexe suivante sous forme exponentielle $z_2 = 7 + 7 \sqrt{3} i$
\item Mettre le complexe suivante sous forme exponentielle $z_3 = \sqrt{3} + i$
\item Calculer le produit $z_4=z_2\times z_3$ donner le résultat sous forme exponentielle puis algébrique.
\item Calculer le quotient $z_5=\frac{z_2}{z_3}$ donner le résultat sous forme exponentielle puis algébrique.
\end{enumerate}
\end{exercise}
\begin{solution}
\begin{enumerate}
\item $z_1 = \frac{2}{61} - \frac{83 i}{61}$
\item $z_3 = 14 e^{\frac{i \pi}{3}}$
\item $z_4 = 28 e^{\frac{i \pi}{2}} = 28 i = 28.0 i$
\item $z_5 = 7 e^{\frac{i \pi}{6}} = \frac{7 \sqrt{3}}{2} + \frac{7 i}{2} = 6.06 + 3.5 i$
\end{enumerate}
\end{solution}
\begin{exercise}[subtitle={Bassin}]
Le tour d'un bassin au niveau du sol présente deux axes de symétrie : laxe des abscisses et la droite déquation $x=4$. Il est obtenu par symétrie de la courbe $\mathcal{C_f}$ sur $\intFF{0}{4}$$f$ est la fonction définie par
\[
f(x) = \left(- x^{2} + 3.6 x - 8.6\right) e^{- x} + 8.6
\]
On admet que sur $\intFF{0}{4}$ la fonction $f$ est positive.
\begin{enumerate}
\item Sur un repère, tracer l'allure de la courbe $\mathcal{C}_f$, les axes de symétries puis compléter pour dessiner la forme du bassin.
\item Montrer que la fonction $f$ admet comme primitive sur $\R$ la fonction $F$ définie par
\[
F(x) = 8.6 x + \left( x^{2} - 1.6 x + 7.0\right) e^{- x}
\]
\item Calculer la quantité $\ds \int_0^4 f(x) \; dx$, vous donnerez le résultat sous forme exacte. Interpréter le résultat et reportez cette quantité sur le graphique.
\item On considère que l'échelle de votre graphique est de 1unité pour 15m. Calculer l'aire du bassin. Vous donnerez un résultat arrondi au $m^2$ près.
\end{enumerate}
\end{exercise}
\begin{solution}
\begin{enumerate}
\item
\begin{tikzpicture}[baseline=(a.north), xscale=1, yscale=0.5]
\tkzInit[xmin=0,xmax=5,xstep=1,
ymin=0,ymax=10,ystep=1]
\tkzGrid
\tkzAxeXY
\tkzFct[domain=0:10,color=red,very thick]%
{ (-x**2 + 3.6*x - 8.6)*exp(-x) + 8.6 };
\end{tikzpicture}
\item Il faut dériver $F(x)$ et vérifier que $F'(x) = f(x)$.
\item $\ds \int_0^4 f(x) \; dx = F(4) - F(0) = \frac{16.6}{e^{4}} + 27.4$
\item La quantité calculée à la question précédente se retrouve 4fois pour former le bassin. Il faut ensuite prendre en compte l'échelle, comme 1unité de longueur correspond à 15m, une unité d'air correspond à $15\times15 = 225m^2$. Ainsi l'aire du bassin est égale à
\[
(\frac{16.6}{e^{4}} + 27.4)\times 4 \times 15^2 = 24934.00000
\]
\end{enumerate}
\end{solution}
\begin{exercise}[subtitle={Bassin}]
Le clinker est un constituant du ciment qui résulte de la cuisson d'un mélange composé de calcaire et d'argile. La fabrication du clinker nécessite des fours à très haute température qui libèrent dans l'air une grande quantité de dioxyde de carbone (CO$_2$).
Dans une cimenterie, la fabrication du clinker s'effectue de 7 h 30 à 20 h, dans une pièce de volume \np{300000}~dm$^3$.
À 20 h, après une journée de travail, le taux volumique de CO$_2$ dans la pièce est de 0.6\,\%.
\begin{enumerate}
\item Justifier que le volume de CO$_2$ présent dans cette pièce à 20 h est de \np{1800}~dm$^3$ .
\item On modélise le volume de CO$_2$ présent dans la pièce par une fonction du temps $t$ écoulé après 20h (exprimé en minutes) qui pour formule $V(t) = V_0e^{-0.08t} + 450$
\begin{enumerate}
\item Démontrer que $V_0$ est égale à \np{1350}.
\item Quel sera, au dm$^3$ près, le volume de CO$_2$ dans cette pièce à 22 h ?
\item Démontrer que $V'(t) = - 108.0 e^{- 0.08 t}$.
\item Étudier le signe de $V'(t)$ puis en déduire le sens de variation de $V(t)$.
\item Que peut-on dire du volume de CO$_2$ quand $t$ devient grand?
\end{enumerate}
\end{enumerate}
\end{exercise}
\begin{solution}
\begin{enumerate}
\item Volume à 20h: $300000\times 0.006 = 1800$
\item
\begin{enumerate}
\item $t=0$ correspond à 20h.
Donc $V(0) = 1800 = V_0e^{-0.08\times 0} + 450 = V_0 + 450$
Donc $V_0 = 1800 - 450 = 1350$
\item Il faut calculer $V(t)$ pour $t = 2$ donc
\[
V(2) = 1600.39
\]
\item Pas de correction pour cette question.
\item Pas de correction pour cette question.
\item Pas de correction pour cette question.
\end{enumerate}
\end{enumerate}
\end{solution}
\end{document}
%%% Local Variables:
%%% mode: latex
%%% TeX-master: "master"
%%% End:

View File

@ -0,0 +1,121 @@
\documentclass[a4paper,10pt]{article}
\usepackage{myXsim}
\usepackage{tasks}
% Title Page
\title{DM1 \hfill RADOUAA Saleh}
\tribe{TST sti2d}
\date{\hfillÀ render pour le jeudi 25 février}
\xsimsetup{
solution/print = true
}
\begin{document}
\maketitle
\begin{exercise}[subtitle={Complexes}]
\begin{enumerate}
\item Mettre le nombre complexe suivant sous forme algébrique $z_1 = \dfrac{4 + 2 i}{-8 + 10 i} $
\item Mettre le complexe suivante sous forme exponentielle $z_2 = 10 - 10 \sqrt{3} i$
\item Mettre le complexe suivante sous forme exponentielle $z_3 = 9 \sqrt{3} + 9 i$
\item Calculer le produit $z_4=z_2\times z_3$ donner le résultat sous forme exponentielle puis algébrique.
\item Calculer le quotient $z_5=\frac{z_2}{z_3}$ donner le résultat sous forme exponentielle puis algébrique.
\end{enumerate}
\end{exercise}
\begin{solution}
\begin{enumerate}
\item $z_1 = - \frac{3}{41} - \frac{14 i}{41}$
\item $z_3 = 20 e^{- \frac{i \pi}{3}}$
\item $z_4 = 360 e^{- \frac{i \pi}{6}} = 180 \sqrt{3} - 180 i = 312.0 - 180.0 i$
\item $z_5 = \frac{10}{9} e^{- \frac{i \pi}{2}} = - \frac{10 i}{9} = - 1.11 i$
\end{enumerate}
\end{solution}
\begin{exercise}[subtitle={Bassin}]
Le tour d'un bassin au niveau du sol présente deux axes de symétrie : laxe des abscisses et la droite déquation $x=4$. Il est obtenu par symétrie de la courbe $\mathcal{C_f}$ sur $\intFF{0}{4}$$f$ est la fonction définie par
\[
f(x) = \left(- x^{2} + 0.6 x - 2.3\right) e^{- x} + 2.3
\]
On admet que sur $\intFF{0}{4}$ la fonction $f$ est positive.
\begin{enumerate}
\item Sur un repère, tracer l'allure de la courbe $\mathcal{C}_f$, les axes de symétries puis compléter pour dessiner la forme du bassin.
\item Montrer que la fonction $f$ admet comme primitive sur $\R$ la fonction $F$ définie par
\[
F(x) = 2.3 x + \left( x^{2} + 1.4 x + 3.7\right) e^{- x}
\]
\item Calculer la quantité $\ds \int_0^4 f(x) \; dx$, vous donnerez le résultat sous forme exacte. Interpréter le résultat et reportez cette quantité sur le graphique.
\item On considère que l'échelle de votre graphique est de 1unité pour 15m. Calculer l'aire du bassin. Vous donnerez un résultat arrondi au $m^2$ près.
\end{enumerate}
\end{exercise}
\begin{solution}
\begin{enumerate}
\item
\begin{tikzpicture}[baseline=(a.north), xscale=1, yscale=0.5]
\tkzInit[xmin=0,xmax=5,xstep=1,
ymin=0,ymax=10,ystep=1]
\tkzGrid
\tkzAxeXY
\tkzFct[domain=0:10,color=red,very thick]%
{ (-x**2 + 0.6*x - 2.3)*exp(-x) + 2.3 };
\end{tikzpicture}
\item Il faut dériver $F(x)$ et vérifier que $F'(x) = f(x)$.
\item $\ds \int_0^4 f(x) \; dx = F(4) - F(0) = \frac{25.3}{e^{4}} + 5.5$
\item La quantité calculée à la question précédente se retrouve 4fois pour former le bassin. Il faut ensuite prendre en compte l'échelle, comme 1unité de longueur correspond à 15m, une unité d'air correspond à $15\times15 = 225m^2$. Ainsi l'aire du bassin est égale à
\[
(\frac{25.3}{e^{4}} + 5.5)\times 4 \times 15^2 = 5367.000000
\]
\end{enumerate}
\end{solution}
\begin{exercise}[subtitle={Bassin}]
Le clinker est un constituant du ciment qui résulte de la cuisson d'un mélange composé de calcaire et d'argile. La fabrication du clinker nécessite des fours à très haute température qui libèrent dans l'air une grande quantité de dioxyde de carbone (CO$_2$).
Dans une cimenterie, la fabrication du clinker s'effectue de 7 h 30 à 20 h, dans une pièce de volume \np{500000}~dm$^3$.
À 20 h, après une journée de travail, le taux volumique de CO$_2$ dans la pièce est de 0.9\,\%.
\begin{enumerate}
\item Justifier que le volume de CO$_2$ présent dans cette pièce à 20 h est de \np{4500}~dm$^3$ .
\item On modélise le volume de CO$_2$ présent dans la pièce par une fonction du temps $t$ écoulé après 20h (exprimé en minutes) qui pour formule $V(t) = V_0e^{-0.06t} + 440$
\begin{enumerate}
\item Démontrer que $V_0$ est égale à \np{4060}.
\item Quel sera, au dm$^3$ près, le volume de CO$_2$ dans cette pièce à 24 h ?
\item Démontrer que $V'(t) = - 243.6 e^{- 0.06 t}$.
\item Étudier le signe de $V'(t)$ puis en déduire le sens de variation de $V(t)$.
\item Que peut-on dire du volume de CO$_2$ quand $t$ devient grand?
\end{enumerate}
\end{enumerate}
\end{exercise}
\begin{solution}
\begin{enumerate}
\item Volume à 20h: $500000\times 0.009000000000000001 = 4500$
\item
\begin{enumerate}
\item $t=0$ correspond à 20h.
Donc $V(0) = 4500 = V_0e^{-0.06\times 0} + 440 = V_0 + 440$
Donc $V_0 = 4500 - 440 = 4060$
\item Il faut calculer $V(t)$ pour $t = 4$ donc
\[
V(4) = 3633.71
\]
\item Pas de correction pour cette question.
\item Pas de correction pour cette question.
\item Pas de correction pour cette question.
\end{enumerate}
\end{enumerate}
\end{solution}
\end{document}
%%% Local Variables:
%%% mode: latex
%%% TeX-master: "master"
%%% End:

View File

@ -0,0 +1,121 @@
\documentclass[a4paper,10pt]{article}
\usepackage{myXsim}
\usepackage{tasks}
% Title Page
\title{DM1 \hfill TAVERNIER Joanny}
\tribe{TST sti2d}
\date{\hfillÀ render pour le jeudi 25 février}
\xsimsetup{
solution/print = true
}
\begin{document}
\maketitle
\begin{exercise}[subtitle={Complexes}]
\begin{enumerate}
\item Mettre le nombre complexe suivant sous forme algébrique $z_1 = \dfrac{10 + 6 i}{-7 + 3 i} $
\item Mettre le complexe suivante sous forme exponentielle $z_2 = 6 - 6 \sqrt{3} i$
\item Mettre le complexe suivante sous forme exponentielle $z_3 = 5 \sqrt{3} - 5 i$
\item Calculer le produit $z_4=z_2\times z_3$ donner le résultat sous forme exponentielle puis algébrique.
\item Calculer le quotient $z_5=\frac{z_2}{z_3}$ donner le résultat sous forme exponentielle puis algébrique.
\end{enumerate}
\end{exercise}
\begin{solution}
\begin{enumerate}
\item $z_1 = - \frac{26}{29} - \frac{36 i}{29}$
\item $z_3 = 12 e^{- \frac{i \pi}{3}}$
\item $z_4 = 120 e^{- \frac{i \pi}{2}} = - 120 i = - 120.0 i$
\item $z_5 = \frac{6}{5} e^{- \frac{i \pi}{6}} = \frac{3 \sqrt{3}}{5} - \frac{3 i}{5} = 1.04 - 0.6 i$
\end{enumerate}
\end{solution}
\begin{exercise}[subtitle={Bassin}]
Le tour d'un bassin au niveau du sol présente deux axes de symétrie : laxe des abscisses et la droite déquation $x=4$. Il est obtenu par symétrie de la courbe $\mathcal{C_f}$ sur $\intFF{0}{4}$$f$ est la fonction définie par
\[
f(x) = \left(- x^{2} + 8.2 x - 3.6\right) e^{- x} + 3.6
\]
On admet que sur $\intFF{0}{4}$ la fonction $f$ est positive.
\begin{enumerate}
\item Sur un repère, tracer l'allure de la courbe $\mathcal{C}_f$, les axes de symétries puis compléter pour dessiner la forme du bassin.
\item Montrer que la fonction $f$ admet comme primitive sur $\R$ la fonction $F$ définie par
\[
F(x) = 3.6 x + \left( x^{2} - 6.2 x - 2.6\right) e^{- x}
\]
\item Calculer la quantité $\ds \int_0^4 f(x) \; dx$, vous donnerez le résultat sous forme exacte. Interpréter le résultat et reportez cette quantité sur le graphique.
\item On considère que l'échelle de votre graphique est de 1unité pour 15m. Calculer l'aire du bassin. Vous donnerez un résultat arrondi au $m^2$ près.
\end{enumerate}
\end{exercise}
\begin{solution}
\begin{enumerate}
\item
\begin{tikzpicture}[baseline=(a.north), xscale=1, yscale=0.5]
\tkzInit[xmin=0,xmax=5,xstep=1,
ymin=0,ymax=10,ystep=1]
\tkzGrid
\tkzAxeXY
\tkzFct[domain=0:10,color=red,very thick]%
{ (-x**2 + 8.2*x - 3.6)*exp(-x) + 3.6 };
\end{tikzpicture}
\item Il faut dériver $F(x)$ et vérifier que $F'(x) = f(x)$.
\item $\ds \int_0^4 f(x) \; dx = F(4) - F(0) = 17.0 - \frac{11.4}{e^{4}}$
\item La quantité calculée à la question précédente se retrouve 4fois pour former le bassin. Il faut ensuite prendre en compte l'échelle, comme 1unité de longueur correspond à 15m, une unité d'air correspond à $15\times15 = 225m^2$. Ainsi l'aire du bassin est égale à
\[
(17.0 - \frac{11.4}{e^{4}})\times 4 \times 15^2 = 15112.00000
\]
\end{enumerate}
\end{solution}
\begin{exercise}[subtitle={Bassin}]
Le clinker est un constituant du ciment qui résulte de la cuisson d'un mélange composé de calcaire et d'argile. La fabrication du clinker nécessite des fours à très haute température qui libèrent dans l'air une grande quantité de dioxyde de carbone (CO$_2$).
Dans une cimenterie, la fabrication du clinker s'effectue de 7 h 30 à 20 h, dans une pièce de volume \np{100000}~dm$^3$.
À 20 h, après une journée de travail, le taux volumique de CO$_2$ dans la pièce est de 0.8\,\%.
\begin{enumerate}
\item Justifier que le volume de CO$_2$ présent dans cette pièce à 20 h est de \np{800}~dm$^3$ .
\item On modélise le volume de CO$_2$ présent dans la pièce par une fonction du temps $t$ écoulé après 20h (exprimé en minutes) qui pour formule $V(t) = V_0e^{-0.01t} + 560$
\begin{enumerate}
\item Démontrer que $V_0$ est égale à \np{240}.
\item Quel sera, au dm$^3$ près, le volume de CO$_2$ dans cette pièce à 22 h ?
\item Démontrer que $V'(t) = - 2.4 e^{- 0.01 t}$.
\item Étudier le signe de $V'(t)$ puis en déduire le sens de variation de $V(t)$.
\item Que peut-on dire du volume de CO$_2$ quand $t$ devient grand?
\end{enumerate}
\end{enumerate}
\end{exercise}
\begin{solution}
\begin{enumerate}
\item Volume à 20h: $100000\times 0.008 = 800$
\item
\begin{enumerate}
\item $t=0$ correspond à 20h.
Donc $V(0) = 800 = V_0e^{-0.01\times 0} + 560 = V_0 + 560$
Donc $V_0 = 800 - 560 = 240$
\item Il faut calculer $V(t)$ pour $t = 2$ donc
\[
V(2) = 795.25
\]
\item Pas de correction pour cette question.
\item Pas de correction pour cette question.
\item Pas de correction pour cette question.
\end{enumerate}
\end{enumerate}
\end{solution}
\end{document}
%%% Local Variables:
%%% mode: latex
%%% TeX-master: "master"
%%% End:

View File

@ -0,0 +1,121 @@
\documentclass[a4paper,10pt]{article}
\usepackage{myXsim}
\usepackage{tasks}
% Title Page
\title{DM1 \hfill ZAHORE Zahiri}
\tribe{TST sti2d}
\date{\hfillÀ render pour le jeudi 25 février}
\xsimsetup{
solution/print = true
}
\begin{document}
\maketitle
\begin{exercise}[subtitle={Complexes}]
\begin{enumerate}
\item Mettre le nombre complexe suivant sous forme algébrique $z_1 = \dfrac{2 + 4 i}{-4 + 9 i} $
\item Mettre le complexe suivante sous forme exponentielle $z_2 = - 2 \sqrt{3} - 2 i$
\item Mettre le complexe suivante sous forme exponentielle $z_3 = 6 + 6 \sqrt{3} i$
\item Calculer le produit $z_4=z_2\times z_3$ donner le résultat sous forme exponentielle puis algébrique.
\item Calculer le quotient $z_5=\frac{z_2}{z_3}$ donner le résultat sous forme exponentielle puis algébrique.
\end{enumerate}
\end{exercise}
\begin{solution}
\begin{enumerate}
\item $z_1 = \frac{28}{97} - \frac{34 i}{97}$
\item $z_3 = 4 e^{- \frac{5 i \pi}{6}}$
\item $z_4 = 48 e^{- \frac{i \pi}{2}} = - 48 i = - 48.0 i$
\item $z_5 = \frac{1}{3} e^{- \frac{7 i \pi}{6}} = - \frac{\sqrt{3}}{6} + \frac{i}{6} = -0.289 + 0.167 i$
\end{enumerate}
\end{solution}
\begin{exercise}[subtitle={Bassin}]
Le tour d'un bassin au niveau du sol présente deux axes de symétrie : laxe des abscisses et la droite déquation $x=4$. Il est obtenu par symétrie de la courbe $\mathcal{C_f}$ sur $\intFF{0}{4}$$f$ est la fonction définie par
\[
f(x) = \left(- x^{2} + 4.8 x - 3.0\right) e^{- x} + 3.0
\]
On admet que sur $\intFF{0}{4}$ la fonction $f$ est positive.
\begin{enumerate}
\item Sur un repère, tracer l'allure de la courbe $\mathcal{C}_f$, les axes de symétries puis compléter pour dessiner la forme du bassin.
\item Montrer que la fonction $f$ admet comme primitive sur $\R$ la fonction $F$ définie par
\[
F(x) = 3.0 x + \left( x^{2} - 2.8 x + 0.2\right) e^{- x}
\]
\item Calculer la quantité $\ds \int_0^4 f(x) \; dx$, vous donnerez le résultat sous forme exacte. Interpréter le résultat et reportez cette quantité sur le graphique.
\item On considère que l'échelle de votre graphique est de 1unité pour 15m. Calculer l'aire du bassin. Vous donnerez un résultat arrondi au $m^2$ près.
\end{enumerate}
\end{exercise}
\begin{solution}
\begin{enumerate}
\item
\begin{tikzpicture}[baseline=(a.north), xscale=1, yscale=0.5]
\tkzInit[xmin=0,xmax=5,xstep=1,
ymin=0,ymax=10,ystep=1]
\tkzGrid
\tkzAxeXY
\tkzFct[domain=0:10,color=red,very thick]%
{ (-x**2 + 4.8*x - 3.0)*exp(-x) + 3.0 };
\end{tikzpicture}
\item Il faut dériver $F(x)$ et vérifier que $F'(x) = f(x)$.
\item $\ds \int_0^4 f(x) \; dx = F(4) - F(0) = \frac{5.0}{e^{4}} + 11.8$
\item La quantité calculée à la question précédente se retrouve 4fois pour former le bassin. Il faut ensuite prendre en compte l'échelle, comme 1unité de longueur correspond à 15m, une unité d'air correspond à $15\times15 = 225m^2$. Ainsi l'aire du bassin est égale à
\[
(\frac{5.0}{e^{4}} + 11.8)\times 4 \times 15^2 = 10702.00000
\]
\end{enumerate}
\end{solution}
\begin{exercise}[subtitle={Bassin}]
Le clinker est un constituant du ciment qui résulte de la cuisson d'un mélange composé de calcaire et d'argile. La fabrication du clinker nécessite des fours à très haute température qui libèrent dans l'air une grande quantité de dioxyde de carbone (CO$_2$).
Dans une cimenterie, la fabrication du clinker s'effectue de 7 h 30 à 20 h, dans une pièce de volume \np{500000}~dm$^3$.
À 20 h, après une journée de travail, le taux volumique de CO$_2$ dans la pièce est de 0.8\,\%.
\begin{enumerate}
\item Justifier que le volume de CO$_2$ présent dans cette pièce à 20 h est de \np{4000}~dm$^3$ .
\item On modélise le volume de CO$_2$ présent dans la pièce par une fonction du temps $t$ écoulé après 20h (exprimé en minutes) qui pour formule $V(t) = V_0e^{-0.05t} + 230$
\begin{enumerate}
\item Démontrer que $V_0$ est égale à \np{3770}.
\item Quel sera, au dm$^3$ près, le volume de CO$_2$ dans cette pièce à 24 h ?
\item Démontrer que $V'(t) = - 188.5 e^{- 0.05 t}$.
\item Étudier le signe de $V'(t)$ puis en déduire le sens de variation de $V(t)$.
\item Que peut-on dire du volume de CO$_2$ quand $t$ devient grand?
\end{enumerate}
\end{enumerate}
\end{exercise}
\begin{solution}
\begin{enumerate}
\item Volume à 20h: $500000\times 0.008 = 4000$
\item
\begin{enumerate}
\item $t=0$ correspond à 20h.
Donc $V(0) = 4000 = V_0e^{-0.05\times 0} + 230 = V_0 + 230$
Donc $V_0 = 4000 - 230 = 3770$
\item Il faut calculer $V(t)$ pour $t = 4$ donc
\[
V(4) = 3316.61
\]
\item Pas de correction pour cette question.
\item Pas de correction pour cette question.
\item Pas de correction pour cette question.
\end{enumerate}
\end{enumerate}
\end{solution}
\end{document}
%%% Local Variables:
%%% mode: latex
%%% TeX-master: "master"
%%% End:

Binary file not shown.

View File

@ -0,0 +1,153 @@
\documentclass[a4paper,10pt]{article}
\usepackage{myXsim}
\usepackage{tasks}
% Title Page
\title{DM1 \hfill \Var{Nom}}
\tribe{TST sti2d}
\date{\hfillÀ render pour le jeudi 25 février}
\xsimsetup{
solution/print = false
}
\begin{document}
\maketitle
%- set I = sympy.I
%- set latex = sympy.latex
%- set sqrt = sympy.sqrt
%- set exp = sympy.functions.exp
%- set integrate = sympy.integrate
\begin{exercise}[subtitle={Complexes}]
\begin{enumerate}
%- set z_num = randint(2, 10) + I*randint(2, 10)
%- set z_denom = -randint(2, 10) + I*randint(2, 10)
%- set z1 = z_num / z_denom
\item Mettre le nombre complexe suivant sous forme algébrique $z_1 = \dfrac{\Var{latex(z_num)}}{\Var{latex(z_denom)}} $
%- set base = choice([(1, sqrt(3)), (sqrt(2), sqrt(2)), (sqrt(3), 1)])
%- set z2 = randint(1, 10)*(choice([1, -1])*base[0] + choice([1, -1])*base[1]*I)
\item Mettre le complexe suivante sous forme exponentielle $z_2 = \Var{latex(z2)}$
%- set base = choice([(1, sqrt(3)), (sqrt(2), sqrt(2)), (sqrt(3), 1)])
%- set z3 = randint(1, 10)*(choice([1, -1])*base[0] + choice([1, -1])*base[1]*I)
\item Mettre le complexe suivante sous forme exponentielle $z_3 = \Var{latex(z3)}$
%- set z4 = z2*z3
\item Calculer le produit $z_4=z_2\times z_3$ donner le résultat sous forme exponentielle puis algébrique.
%- set z5 = z2/z3
\item Calculer le quotient $z_5=\frac{z_2}{z_3}$ donner le résultat sous forme exponentielle puis algébrique.
\end{enumerate}
\end{exercise}
\begin{solution}
\begin{enumerate}
\item $z_1 = \Var{latex(sympy.re(z1) + sympy.im(z1)*I)}$
\item $z_3 = \Var{latex(sympy.Abs(z2))} e^{\Var{latex(I*sympy.arg(z2))}}$
\item $z_4 = \Var{latex(sympy.Abs(z4))} e^{\Var{latex(I*(sympy.arg(z2) + sympy.arg(z3)))}} = \Var{latex(sympy.re(z4) + sympy.im(z4)*I)} = \Var{latex(sympy.N(sympy.re(z4), 3)+ sympy.N(sympy.im(z4), 3)*I)}$
\item $z_5 = \Var{latex(sympy.Abs(z5))} e^{\Var{latex(I*(sympy.arg(z2) - sympy.arg(z3)))}} = \Var{latex(sympy.re(z5) + sympy.im(z5)*I)} = \Var{latex(sympy.N(sympy.re(z5), 3)+ sympy.N(sympy.im(z5), 3)*I)}$
\end{enumerate}
\end{solution}
\begin{exercise}[subtitle={Bassin}]
%- set a = round(random()*10, 1)
%- set b = round(random()*10, 1)
%- set x = sympy.symbols("x")
%- set f = -(x**2 - a*x + b)*exp(-x) + b
%- set F = integrate(f, x)
Le tour d'un bassin au niveau du sol présente deux axes de symétrie : laxe des abscisses et la droite déquation $x=4$. Il est obtenu par symétrie de la courbe $\mathcal{C_f}$ sur $\intFF{0}{4}$$f$ est la fonction définie par
\[
f(x) = \Var{latex(f)}
\]
On admet que sur $\intFF{0}{4}$ la fonction $f$ est positive.
\begin{enumerate}
\item Sur un repère, tracer l'allure de la courbe $\mathcal{C}_f$, les axes de symétries puis compléter pour dessiner la forme du bassin.
\item Montrer que la fonction $f$ admet comme primitive sur $\R$ la fonction $F$ définie par
\[
F(x) = \Var{latex(F) | replace("1.0", "")}
\]
\item Calculer la quantité $\ds \int_0^4 f(x) \; dx$, vous donnerez le résultat sous forme exacte. Interpréter le résultat et reportez cette quantité sur le graphique.
\item On considère que l'échelle de votre graphique est de 1unité pour 15m. Calculer l'aire du bassin. Vous donnerez un résultat arrondi au $m^2$ près.
\end{enumerate}
\end{exercise}
\begin{solution}
\begin{enumerate}
\item
\begin{tikzpicture}[baseline=(a.north), xscale=1, yscale=0.5]
\tkzInit[xmin=0,xmax=5,xstep=1,
ymin=0,ymax=10,ystep=1]
\tkzGrid
\tkzAxeXY
\tkzFct[domain=0:10,color=red,very thick]%
{ \Var{f} };
\end{tikzpicture}
\item Il faut dériver $F(x)$ et vérifier que $F'(x) = f(x)$.
%- set surf = integrate(f, (x, 0, 4))
\item $\ds \int_0^4 f(x) \; dx = F(4) - F(0) = \Var{latex(surf)}$
\item La quantité calculée à la question précédente se retrouve 4fois pour former le bassin. Il faut ensuite prendre en compte l'échelle, comme 1unité de longueur correspond à 15m, une unité d'air correspond à $15\times15 = 225m^2$. Ainsi l'aire du bassin est égale à
\[
(\Var{latex(surf)})\times 4 \times 15^2 = \Var{round(sympy.N(surf*4*15**2, 10), 0)}
\]
\end{enumerate}
\end{solution}
\begin{exercise}[subtitle={Bassin}]
%- set Vinit = randint(1, 10)*100000
%- set tx = round((random()+1)/2, 1)
Le clinker est un constituant du ciment qui résulte de la cuisson d'un mélange composé de calcaire et d'argile. La fabrication du clinker nécessite des fours à très haute température qui libèrent dans l'air une grande quantité de dioxyde de carbone (CO$_2$).
Dans une cimenterie, la fabrication du clinker s'effectue de 7 h 30 à 20 h, dans une pièce de volume \np{\Var{Vinit}}~dm$^3$.
À 20 h, après une journée de travail, le taux volumique de CO$_2$ dans la pièce est de \Var{tx}\,\%.
\begin{enumerate}
%- set v20 = int(Vinit*tx/100)
\item Justifier que le volume de CO$_2$ présent dans cette pièce à 20 h est de \np{\Var{v20}}~dm$^3$ .
%- set q = round(random()/10, 2)
%- set c = randint(20, 60)*10
%- set v0 = int(v20 - c)
%- set t = sympy.symbols("t")
%- set V = v0*exp(- q*t) + c
%- set Vp = V.diff()
\item On modélise le volume de CO$_2$ présent dans la pièce par une fonction du temps $t$ écoulé après 20h (exprimé en minutes) qui pour formule $V(t) = V_0e^{-\Var{q}t} + \Var{c}$
\begin{enumerate}
\item Démontrer que $V_0$ est égale à \np{\Var{v0}}.
%- set decal = randint(1, 4)
\item Quel sera, au dm$^3$ près, le volume de CO$_2$ dans cette pièce à \Var{20+decal} h ?
\item Démontrer que $V'(t) = \Var{latex(Vp)}$.
\item Étudier le signe de $V'(t)$ puis en déduire le sens de variation de $V(t)$.
\item Que peut-on dire du volume de CO$_2$ quand $t$ devient grand?
\end{enumerate}
\end{enumerate}
\end{exercise}
\begin{solution}
\begin{enumerate}
\item Volume à 20h: $\Var{Vinit}\times \Var{tx/100} = \Var{v20}$
\item
\begin{enumerate}
\item $t=0$ correspond à 20h.
Donc $V(0) = \Var{v20} = V_0e^{-\Var{q}\times 0} + \Var{c} = V_0 + \Var{c}$
Donc $V_0 = \Var{v20} - \Var{c} = \Var{v0}$
\item Il faut calculer $V(t)$ pour $t = \Var{decal}$ donc
\[
V(\Var{decal}) = \Var{round(V.subs(t, str(decal)), 2)}
\]
\item Pas de correction pour cette question.
\item Pas de correction pour cette question.
\item Pas de correction pour cette question.
\end{enumerate}
\end{enumerate}
\end{solution}
\end{document}
%%% Local Variables:
%%% mode: latex
%%% TeX-master: "master"
%%% End: